INSIGHTSIAS IA SIMPLIFYING IAS EXAM PREPARATION

RTM COMPILATIONS PRELIMS 2021

March 2021

www.insightsactivelearn.com | www.insightsonindia.com

Revision Through MCQs (RTM) Compilation (March 2021)

Telegram: https://t.me/insightsIAStips 2 Youtube: https://www.youtube.com/channel/UCpoccbCX9GEIwaiIe4HLjwA

Revision Through MCQs (RTM) Compilation (March 2021)

Table of Contents RTM- REVISION THROUGH MCQS – 1st-Mar-2021 ...... 4 RTM- REVISION THROUGH MCQS – 2nd-Mar-2021 ...... 9 RTM- REVISION THROUGH MCQS – 3rd-Mar-2021 ...... 14 RTM- REVISION THROUGH MCQS – 4th-Mar-2021 ...... 20 RTM- REVISION THROUGH MCQS – 5th-Mar-2021 ...... 24 RTM- REVISION THROUGH MCQS – 6th-Mar-2021 ...... 28 RTM- REVISION THROUGH MCQS – 8th-Mar-2021 ...... 33 RTM- REVISION THROUGH MCQS – 9th-Mar-2021 ...... 38 RTM- REVISION THROUGH MCQS – 10th-Mar-2021 ...... 44 RTM- REVISION THROUGH MCQS – 11th-Mar-2021 ...... 49 RTM- REVISION THROUGH MCQS – 12th-Mar-2021 ...... 55 RTM- REVISION THROUGH MCQS – 13th-Mar-2021 ...... 61 RTM- REVISION THROUGH MCQS – 15th-Mar-2021 ...... 66 RTM- REVISION THROUGH MCQS – 16th-Mar-2021 ...... 71 RTM- REVISION THROUGH MCQS – 17th-Mar-2021 ...... 76 RTM- REVISION THROUGH MCQS – 18th-Mar-2021 ...... 80 RTM- REVISION THROUGH MCQS – 19th-Mar-2021 ...... 86 RTM- REVISION THROUGH MCQS – 20th-Mar-2021 ...... 90 RTM- REVISION THROUGH MCQS – 22th-Mar-2021 ...... 95 RTM- REVISION THROUGH MCQS – 23th-Mar-2021 ...... 101 RTM- REVISION THROUGH MCQS – 24th-Mar-2021 ...... 107 RTM- REVISION THROUGH MCQS – 25th-Mar-2021 ...... 112 RTM- REVISION THROUGH MCQS – 26th-Mar-2021 ...... 116 RTM- REVISION THROUGH MCQS – 27th-Mar-2021 ...... 121 RTM- REVISION THROUGH MCQS – 30th-Mar-2021 ...... 126 RTM- REVISION THROUGH MCQS – 31th-Mar-2021 ...... 132

Telegram: https://t.me/insightsIAStips 3 Youtube: https://www.youtube.com/channel/UCpoccbCX9GEIwaiIe4HLjwA

Revision Through MCQs (RTM) Compilation (March 2021)

RTM- REVISION THROUGH MCQS – 1st-Mar-2021

1. ‘City Innovation Exchange’ (CiX) is launched by which of the following? (a) NITI Aayog (b) Prime Minister's Office (c) Securities and Exchange Board of (d) Ministry of Housing and Urban Affairs Ans: (d) Explanation: • The Ministry of Housing and Urban Affairs has launched a City Innovation Exchange (CiX). • What is it? o The CiX will connect cities to innovators across the national ecosystem to design innovative solutions for their pressing challenges. o The platform will ease the discovery, design & validation of solutions through a robust, transparent and user-centric process that will reduce barriers for innovators and cities to discover fitting solutions. o Built on the concept of ‘open innovation’, the platform will help in the flow of ideas ‘outside in and inside out, enhancing the skills and capacity required to deliver smart urban governance. Refer: https://www.insightsonindia.com/2021/03/01/city-innovation-exchange-cix/ 2. Which of the following best describes/ describe the objectives of ‘National Bamboo Mission’ of the Government of India? 1. To rejuvenate the under developed bamboo industry in India. 2. To increase the area under bamboo plantation in non-forest Government and private lands. 3. To generate employment opportunities for skilled and unskilled persons, especially unemployed youths. Select the correct answer using the code given below. (a) 1 only (b) 2 and 3 only (c) 3 only (d) 1, 2 and 3 Ans: (d) Explanation: • With a view to harness the potential of bamboo crop, Department of Agriculture & Cooperation (DAC), Ministry of Agriculture & Farmers Welfare is implementing a 100% Centrally Sponsored Scheme called Mission for Integrated Development of Horticulture (MIDH) in which National Bamboo Mission (NBM) is being implemented as a sub scheme.

• More>> Refer: https://www.insightsonindia.com/2021/03/01/north-east-cane-and-bamboo-development- council-necbdc/

Telegram: https://t.me/insightsIAStips 4 Youtube: https://www.youtube.com/channel/UCpoccbCX9GEIwaiIe4HLjwA

Revision Through MCQs (RTM) Compilation (March 2021)

3. With reference to Asian Infrastructure Investment Bank (AIIB), consider the following statements 1. AIIB has more than 80 member nations. 2. India is the largest shareholder in AIIB. 3. AIIB does not have any members from outside Asia. Which of the statements given above is / are correct? (a) 1 only (b) 2 and 3 only (c) 1 and 2 only (d) 1, 2 and 3 Ans: (a) Explanation: What is AIIB? • Asian Infrastructure Investment Bank (AIIB) is a multilateral development bank with a mission to improve social and economic outcomes in Asia and beyond. • The Parties (57 founding members) to agreement comprise the Membership of the Bank. • It is headquartered in Beijing. • It commenced operations in January 2016. • Aim: o By investing in sustainable infrastructure and other productive sectors today, it aims to connect people, services and markets that over time will impact the lives of billions and build a better future. • Membership: o There are more than 100 members now. o Fourteen of the G-20 nations are AIIB members including France, Germany, Italy and the United Kingdom. • Voting Rights: o is the largest shareholder with 26.61 % voting shares in the bank followed by India (7.6%), Russia (6.01%) and Germany (4.2 %). o The regional members hold 75% of the total voting power in the Bank. Refer: https://www.insightsonindia.com/2021/03/01/asian-infrastructure-investment-bank-aiib-4/ 4. With reference to India’s satellite launch vehicle, consider the following statements: 1. PSLVs launch the satellites useful for Earth resources monitoring whereas GSLVs are designed mainly to launch communication satellites. 2. Satellites launched by PSLV appear to remain permanently fixed in the same position in the sky, as viewed from a particular location on Earth. 3. GSLV MK III is a four-staged launch vehicle with the first and third stages using solid rocket motors; and the second and fourth stages using liquid rocket engines. Which of the statements given above is/are correct? (a) 1 only (b) 2 and 3 (c) 1 and 2 (d) 3 only Ans: (a) Explanation: • PSLV is designed mainly to deliver the “earth-observation” or “remote-sensing” satellites with lift-off mass of up to about 1750 Kg to Sun-Synchronous circular polar orbits of 600- 900 Km altitude. The GSLV is designed mainly to deliver the communication-satellites to the highly elliptical (typically 250 x 3.6000 Km) Geosynchronous Transfer Orbit (GTO). The satellite in GTO is further raised to its final destination, viz., Geo-synchronous Earth orbit (GEO) of about 3.6000 Km altitude (and zero degree inclination on equatorial plane) by firing its in-built on-board engines. Due to their geo-synchronous nature, the satellites in these orbits appear to remain permanently fixed in the same position in the sky, as

Telegram: https://t.me/insightsIAStips 5 Youtube: https://www.youtube.com/channel/UCpoccbCX9GEIwaiIe4HLjwA

Revision Through MCQs (RTM) Compilation (March 2021)

viewed from a particular location on Earth, thus avoiding the need of a tracking ground antenna and hence are useful for the communication applications. • Source: http://www.thehindu.com/sci-tech/science/what-is-the-difference-between- gslv-and-pslv/article6742299.ece • GSLV Mk 3 is a three-stage heavy lift launch vehicle developed by ISRO. The vehicle has two solid strap-one, a core liquid booster and a cryogenic upper stage. • Source: https://www.isro.gov.in/launchers/gslv-mk-3 Refer: https://www.insightsonindia.com/2021/03/01/pslv-c51-launch/ 5. Consider the following statements about ‘Technical Education Quality Improvement Programme’ (TEQIP): 1. In the year 2002, the Ministry of Human Resource and Development (MHRD) launched the TEQIP scheme. 2. The project commenced with the Asian Development Bank (ADB) assistance. 3. The programme aims to overhaul the quality of technical education in the Low Income States and Special Category States (SCS) in India. Which of the given above statements is/are correct? (a) 1 and 2 only (b) 2 and 3 only (c) 1 and 3 only (d) 1, 2 and 3 Ans: (c) Explanation: About TEQIP: • In the year 2002, the ministry of Human Resource and Development launched the TEQIP scheme. • The project commenced with the World Bank assistance. • The programme aims to overhaul the quality of technical education in the Low Income States and Special Category States (SCS) in India. Refer: https://www.insightsonindia.com/2021/03/01/technical-education-quality-improvement- programme-teqip/ 6. The ‘National Science Day’ is celebrated every year on 28th of February in India, to commemorate the discovery of the: (a) Photoelectric Effect (b) Quantum theory (c) Nuclear Reactor (d) Raman Effect Ans: (d) Explanation: • 28th February is celebrated as National Science Day (NSD) in India. • NSD is celebrated to commemorate discovery of the ‘Raman Effect’, which led to Sir C.V. Raman winning the Noble Prize. • The first National Science Day was celebrated on February 28, 1987. • Theme: “Future of STI: Impacts on Education, Skills, and Work”. • What is Raman Effect? o A phenomenon in spectroscopy discovered by the eminent physicist Sir Chandrasekhara Venkata Raman in 1928. o Raman Effect is a change in the wavelength of light that occurs when a light beam is deflected by molecules. o When a beam of light traverses a dust-free, transparent sample of a chemical compound, a small fraction of the light emerges in directions other than that of the incident (incoming) beam.

Telegram: https://t.me/insightsIAStips 6 Youtube: https://www.youtube.com/channel/UCpoccbCX9GEIwaiIe4HLjwA

Revision Through MCQs (RTM) Compilation (March 2021)

o Most of this scattered light is of unchanged wavelength. A small part, however, has wavelengths different from that of the incident light; its presence is a result of the Raman Effect. Refer: https://www.insightsonindia.com/2021/03/01/national-science-day-2/ 7. Consider the following statements regarding Snow Leopard: 1. It is listed as endangered on the IUCN Red List of Threatened Species. 2. The Union Government had launched SECURE Himalaya project for the protection of Snow Leopard. 3. In India, it can be found only in the western half of the Himalayas, stretching from Jammu and Kashmir to the western India-Nepal border. Which of the above statements is/are correct? (a) 1 and 2 (b) 2 only (c) 1 and 3 (d) 1, 2 and 3 Ans: (b) Explanation: • S1: Snow leopard is listed as Vulnerable under the IUCN Red List. • S3: In India, the snow leopard habitat is spread over the northern Himalayan mountains of Jammu and Kashmir, Himachal Pradesh, Uttarakhand, Arunachal Pradesh and Sikkim. • S2: The Union Government had launched SECURE Himalaya, a six-year project to ensure conservation of locally and globally significant biodiversity, land and forest resources in high Himalayan ecosystem spread over four states viz. Himachal Pradesh, Jammu and Kashmir, Uttarakhand and Sikkim. Refer: https://www.insightsonindia.com/2021/03/01/study-on-snow-leopard-in-himachal-pradesh/ 8. Consider the following statements about Jal Shakti Abhiyan: 1. It is a time-bound, mission-mode campaign. 2. The focus of the campaign is on water stressed districts and blocks. 3. The campaign will run through citizen participation. Which of the given above statements is/are correct? (a) 1 and 2 only (b) 2 and 3 only (c) 1 and 3 only (d) 1, 2 and 3 Ans: (d) Explanation: Jal Shakti Abhiyan: • It is a time-bound, mission-mode campaign. • Launched in 2019, it covered 256 water stressed districts across the country. • The campaign will run through citizen participation. • It is also a mass movement to bring all the stakeholders under one ambit of water conservation drive. • It is a collaborative effort of various Ministries of the Government of India and State Governments, being coordinated by the Department of Drinking Water and Sanitation. • Jal Shakti Abhiyan ‘catch the rain’: o To be initiated by the Jal Shakti Ministry. o It will be a 100-day campaign for the sake of cleaning up the water sources around us and conserving rainwater. Refer: facts for prelims: https://www.insightsonindia.com/2021/03/01/insights-daily-current- affairs-pib-summary-1-march-2021/

Telegram: https://t.me/insightsIAStips 7 Youtube: https://www.youtube.com/channel/UCpoccbCX9GEIwaiIe4HLjwA

Revision Through MCQs (RTM) Compilation (March 2021)

9. Maguri Motapung Beel Wetland’ is located in: (a) Arunachal Pradesh (b) (c) (d) Ans: (b) Explanation: • Maguri Motapung Beel is less than 10 km south of the more famous Dibru-Saikhowa National Park and part of the Dibru-Saikhowa Biosphere Reserve. • The wetland derives its name from ‘Magur’, local word for the catfish Clarius batrachus, once found here in abundance. Motapung is a village nearby, and Beel is the Assamese word for wetland. Refer: Facts for Prelims: https://www.insightsonindia.com/2020/07/20/insights-daily-current- affairs-pib-summary-20-july-2020/ 10. ‘Dibru-Saikhowa National Park’ is located in: (a) Arunachal Pradesh (b) Assam (c) Meghalaya (d) Manipur Ans: (b) Explanation: • Dibru-Saikhowa National Park is a national park in Assam, India, located in Dibrugarh and Tinsukia districts. • It was designated a Biosphere Reserve in July 1997 • The park is bounded by the Brahmaputra and Lohit Rivers in the north and Dibru river in the south. • It mainly consists of moist mixed semi-evergreen forests, moist mixed deciduous forests, canebrakes and grasslands. • It is the largest salix swamp forest in north-eastern India, with a tropical monsoon climate with a hot and wet summer and cool and usually dry winter. Refer: Facts for Prelims: https://www.insightsonindia.com/2020/07/20/insights-daily-current- affairs-pib-summary-20-july-2020/

Telegram: https://t.me/insightsIAStips 8 Youtube: https://www.youtube.com/channel/UCpoccbCX9GEIwaiIe4HLjwA

Revision Through MCQs (RTM) Compilation (March 2021)

RTM- REVISION THROUGH MCQS – 2nd-Mar-2021

11. ‘The Opportunity Index 2021’ report is published by which of the following? (a) IBM Corporation (b) LinkedIn Corporation (c) Infosys (d) Tata Consultancy Services Ans: (b) Explanation: • It is a new report by LinkedIn. • The survey, conducted in January, saw participation from more than 10,000 respondents across the Asia Pacific (APAC) region. It covered 2,285 respondents in India. • Key findings: o 9 in 10 or 89 per cent of women were negatively impacted by the coronavirus pandemic. o About 85%, or four in five working women in India believe they have missed out on a raise, promotion, or work offer because of their gender. This average stands at 60% for the Asia Pacific (APAC) region. o More women in India have experienced the impact of gender on career development when compared to the APAC region. Refer: https://www.insightsonindia.com/2021/03/02/the-opportunity-index-2021-report/ 12. Which of the following best describes/ describe the objectives of ‘Make in India’ initiative of the Government of India? 1. promotion of investment from domestic and foreign sources 2. to create 100 million additional manufacturing jobs in the economy by 2022 3. To make use of more agile implementation models Select the correct answer using the code below: (a) 1 and 3 only (b) 2 only (c) 2 and 3 only (d) 1, 2 and 3 Ans: (b) Explanation: About the Make in India campaign: • Launched by Prime Minister Modi in 2014 to incentivise production in India. • It seeks to encourage manufacturing in India and galvanize the economy with dedicated investments in manufacturing and services. • "Make in India" had three stated objectives: o To increase the manufacturing sector’s growth rate to 12-14% per annum in order to increase the sector’s share in the economy. o To create 100 million additional manufacturing jobs in the economy by 2022. o To ensure that the manufacturing sector’s contribution to GDP is increased to 25% by 2022 (revised to 2025) from the current 15-16%. Refer: https://www.insightsonindia.com/2021/03/02/the-us-2021-presidents-trade-agenda-and- 2020-annual-report/ 13. Consider the following statements: 1. The Indian Computer Emergency Response Team (CERT-In) is the nodal agency to deal with cyber security threats like hacking and phishing. 2. The CERT-In is affiliated to the Ministry of Home Affairs, Government of India. Which of the given above statements is/are correct? (a) 1 only (b) 2 only (c) Both 1 and 2

Telegram: https://t.me/insightsIAStips 9 Youtube: https://www.youtube.com/channel/UCpoccbCX9GEIwaiIe4HLjwA

Revision Through MCQs (RTM) Compilation (March 2021)

(d) Neither 1 nor 2 Ans: (a) Explanation: • The Indian Computer Emergency Response Team (CERT-In) is an office within the Ministry of Electronics and Information Technology of the Government of India. • It is the nodal agency to deal with cyber security threats like hacking and phishing. It strengthens security-related defence of the Indian Internet domain. Refer: https://www.insightsonindia.com/2021/03/02/chinese-cyber-attack-foiled-power-ministry/ 14. The ‘BN Srikrishna Committee’ report was in news recently, is associated with which of the following? (a) Armed Forces (Special Powers) Act (b) Digital Payments (c) Development Finance Bank (d) Data Protection Framework Ans: (d) Explanation:

• Refer: https://www.insightsonindia.com/2021/03/02/whatsapps-new-policy-being-examined-govt/ 15. Consider the following statements regarding Purchasing Manager’s Index (PMI): 1. It is compiled and released by Confederation of Indian Industry (CII). 2. A PMI reading above 50 represents an expansion in business activity when compared to the previous year. Which of the given above statements is/are correct? (a) 1 only (b) 2 only (c) Both 1 and 2 (d) Neither 1 nor 2 Ans: (d) Explanation: • PMI or a Purchasing Managers’ Index (PMI) is an indicator of business activity — both in the manufacturing and services sectors. • S1: Purchasing Manager’s Index (PMI) is released and compiled by the Institute for Supply Management (ISM), USA • The PMI is usually released at the start of the month, much before most of the official data on industrial output, manufacturing and GDP growth becomes available • It is calculated separately for the manufacturing and services sectors and then a composite index is constructed.

Telegram: https://t.me/insightsIAStips 10 Youtube: https://www.youtube.com/channel/UCpoccbCX9GEIwaiIe4HLjwA

Revision Through MCQs (RTM) Compilation (March 2021)

• S2: The headline PMI is a number from 0 to 100. A PMI above 50 represents an expansion when compared to the previous month. A PMI reading under 50 represents a contraction, and a reading at 50 indicates no change. Refer: Facts for Prelims: https://www.insightsonindia.com/2021/03/02/insights-daily-current- affairs-pib-summary-2-march-2021/ 16. Consider the following statements: 1. Indian Council of Medical Research (ICMR) is neither a statutory body nor a regulatory body. 2. According to the Clinical Trial Rules, only the ICMR can initiate action when investigators or sponsor of vaccines trail fail to comply with the Drugs and Cosmetics Act or the Rules. Which of the given above statements is/are correct? (a) 1 only (b) 2 only (c) Both 1 and 2 (d) Neither 1 nor 2 Ans: (a) Explanation: • Source: S1: ICMR is neither a statutory body nor a regulatory body. It is only the apex body in India for formulating, coordinating and promoting biomedical research. It doesn’t have the power to directly initiate punitive action. • S2: According to the Clinical Trial Rules, only the Drugs Controller General of India (DCGI) can initiate action when investigators fail to comply with the Drugs and Cosmetics Act or the Rules. And in such cases, the DCGI may – after giving a show-cause notice – issue a warning, reject the trial’s results, suspend or permanently cancel the permission granted and/or debar the investigator or sponsor from conducting any trials in future. Refer: https://www.insightsonindia.com/2020/11/19/icmr-against-indiscriminate-use-of-plasma- therapy/ 17. The ‘Joint Comprehensive Plan of Action’ (JCPOA) was in news recently, is related to which of the following? (a) India and the European Union (EU) civil nuclear cooperation agreement (b) Indo-US nuclear deal (c) Iran nuclear deal (d) India-Japan civil nuclear agreement Ans: (c) Explanation: • On 20 July 2015, the Security Council unanimously adopted resolution 2231 (2015) endorsing the the Joint Comprehensive Plan of Action (JCPOA). • Resolution 2231 provides for the termination of the provisions of previous Security Council resolutions on the Iranian nuclear issue and establishes specific restrictions that apply to all States without exception. • What was the iran nuclear deal? o Iran agreed to rein in its nuclear programme in a 2015 deal struck with the US, UK, Russia, China, France and Germany. o Under the Joint Comprehensive Plan of Action (JCPoA) Tehran agreed to significantly cut its stores of centrifuges, enriched uranium and heavy-water, all key components for nuclear weapons. o The JCPOA established the Joint Commission, with the negotiating parties all represented, to monitor implementation of the agreement. Refer: https://www.insightsonindia.com/2020/11/19/will-return-to-nuclear-deal-if-u-s-sanctions- are-lifted-iran/ 18. Consider the following: 1. Birds 2. Dust blowing

Telegram: https://t.me/insightsIAStips 11 Youtube: https://www.youtube.com/channel/UCpoccbCX9GEIwaiIe4HLjwA

Revision Through MCQs (RTM) Compilation (March 2021)

3. Rain 4. Wind blowing Which of the above spread plant diseases? (a) 1 and 3 only (b) 3 and 4 only (c) 1, 2 and 4 only (d) 1, 2, 3 and 4 Ans: (d) Explanation: • Nematodes, snails, birds, and wild and domestic animals often help dissemination of plant diseases. • The spores of many parasitic fungi are disseminated by air currents from diseased to disease-free host Pathogens like, bacteria are often disseminated by splashing of raindrops, as in case of Citrus canker disease. • Soil and field operation also disseminate the diseases as they result in dust blowing. Refer: http://www.biologydiscussion.com/plants/plant-diseases/dissemination-of-plant-diseases- 16-mechanismsbotany/58671 19. With reference to Election Commission of India's Model Code of Conduct (MCC), consider the following statements: 1. The government bodies are not to participate in any recruitment process during the electoral process. 2. The election code in force hinders the ruling party leaders from launching new welfare programs. 3. The candidates and political parties must inform the local police for distributing liquor on polling day and during the forty eight hours preceding it. Which of the given above statements is/are correct? (a) 1 and 2 only (b) 2 and 3 only (c) 1 and 3 only (d) 1, 2 and 3 Ans: (a) Explanation: The main points of the code of conduct (link) are: • S1: The government may not lay any new ground for projects or public initiatives once the Model Code of Conduct comes into force. • S2: Government bodies are not to participate in any recruitment process during the electoral process. • The election campaign rallies and road shows must not hinder the road traffic. • S3: Candidates are asked to refrain from distributing liquor to voters. It is a widely known fact in India that during election campaigning, liquor may be distributed to the voters. • Rest you can find it here: https://eci.gov.in/mcc/ Refer: https://www.insightsonindia.com/2020/11/27/one-nation-one-election-2/ 20. Consider the following pairs : Terms sometimes seen in news Context / Topic 1. Hayabusa2 project Asteroid sample-return mission 2. Feluda test CRISPR gene-editing technology 3. Tokamak Digital Crypto currency Which of the pairs given above is/are correctly matched? (a) 1 only (b) 2 and 3 only (c) 1 and 2 only (d) 1, 2 and 3

Telegram: https://t.me/insightsIAStips 12 Youtube: https://www.youtube.com/channel/UCpoccbCX9GEIwaiIe4HLjwA

Revision Through MCQs (RTM) Compilation (March 2021)

Ans: (c) Explanation: • Hayabusa2 project: o It is an asteroid sample-return mission operated by the Japanese space agency, JAXA. o It was launched on 3 December 2014 and rendezvoused with Ryugu on 27 June 2018. o It carried multiple science payloads for remote sensing, sampling, and four small rovers that will investigate the asteroid surface to inform the environmental and geological context of the samples collected. • Feluda is the acronym for FNCAS9 Editor Linked Uniform Detection Assay. o It is an accurate and low-cost paper-based test strip to detect Covid-19 and can deliver a result in 45 minutes. o It has been approved for commercial launch by the Drugs Controller General of India. o Developed by the Council of Scientific and Industrial Research (CSIR) and Tata Group. o How it works? ▪ It uses indigenously developed CRISPR gene-editing technology to identify and target the genetic material of SARS-CoV2, the virus that causes Covid-19. o https://www.insightsonindia.com/2020/10/12/what-is-the-feluda-test-for- covid-19/ • What is a Tokamak? o The tokamak is an experimental machine designed to harness the energy of fusion. o Inside a tokamak, the energy produced through the fusion of atoms is absorbed as heat in the walls of the vessel. o Just like a conventional power plant, a fusion power plant will use this heat to produce steam and then electricity by way of turbines and generators. o First developed by Soviet research in the late 1960s, the tokamak has been adopted around the world as the most promising configuration of magnetic fusion device. ITER will be the world’s largest tokamak—twice the size of the largest machine currently in operation, with ten times the plasma chamber volume. o https://www.insightsonindia.com/2020/07/30/international-thermonuclear- experimental-reactor-iter/ Refer: https://www.insightsonindia.com/2020/11/30/japans-hayabusa2-spacecraft-carrying- asteroid-soil-samples-nears-earth/

Telegram: https://t.me/insightsIAStips 13 Youtube: https://www.youtube.com/channel/UCpoccbCX9GEIwaiIe4HLjwA

Revision Through MCQs (RTM) Compilation (March 2021)

RTM- REVISION THROUGH MCQS – 3rd-Mar-2021

21. As per the Election Commission of India, a political party shall be eligible to be recognized as a National party if: 1. It secures at least four percent of the valid votes polled in any four states. 2. It wins at least six seats in the House of the People from any State or States. 3. It wins at least two percent seats in the House of the People and these members are elected from at least three different States. Select the correct answer using the code below: (a) 1 and 2 only (b) 3 only (c) 2 and 3 only (d) 1, 2 and 3 Ans: (b) Explanation: • Registration of political parties: • Registration of Political parties is governed by the provisions of Section 29A of the Representation of the People Act, 1951. • A party seeking registration under the said Section with the Election Commission has to submit an application to the Commission within the said period following the date of its formation as per guidelines prescribed by the Election Commission of India in exercise of the powers conferred by Article 324 of the Commission of India and Section 29A of the Representation of the People Act, 1951. • To be eligible for a ‘National Political Party of India: o S1: It secures at least six percent of the valid votes polled in any four or more states, at a general election to the House of the People or, to the State Legislative Assembly. o S2: In addition, it wins at least four seats in the House of the People from any State or States. o S3: It wins at least two percent seats in the House of the People (i.e., 11 seats in the existing House having 543 members), and these members are elected from at least three different States. • To be eligible for a ‘State Political Party: o It secures at least six percent of the valid votes polled in the State at a general election, either to the House of the People or to the Legislative Assembly of the State concerned o In addition, it wins at least two seats in the Legislative Assembly of the State concerned. o It wins at least three percent (3%) of the total number of seats in the Legislative Assembly of the State, or at least three seats in the Assembly, whichever is more. Refer: https://www.insightsonindia.com/2021/03/03/registration-of-political-parties-2/

22. Consider the following statements about the Union Public Service Commission (UPSC): 1. The agency's charter is granted by Part XIV of the Constitution of India. 2. The President of India appoints the Chairman and other members of Union Public Service Commission. 3. The commission reports directly to the Minister of Personnel, Public Grievances and Pensions and can advise the Government through him. Which of the given above statements is/are correct? (a) 1 and 2 only (b) 2 and 3 only (c) 1 and 3 only

Telegram: https://t.me/insightsIAStips 14 Youtube: https://www.youtube.com/channel/UCpoccbCX9GEIwaiIe4HLjwA

Revision Through MCQs (RTM) Compilation (March 2021)

(d) 1, 2 and 3 Ans: (a) Explanation: • S1: Articles 315 to 323 of Part XIV of the constitution, titled as Services under the Union and the States, provide for a Public Service Commission for the Union and for each state. Accordingly, as per Art. 315, at Union level, Union Public Service Commission is envisaged by it. • S2: As per Art. 316, the Chairman and other members of Union Public Service Commission shall be appointed by the President. In case the office of the Chairman becomes vacant his duties shall be performed by one of the other members of the Commission as the President may appoint for the purpose. • S3: The commission is mandated by the Constitution for appointments to the services of the Union and All India Services. It is also required to be consulted by the Government in matters relating to the appointment, transfer, promotion and disciplinary matters. The commission reports directly to the President and can advise the Government through him. Although, such advice is not binding on the Government. Refer: https://www.insightsonindia.com/2021/03/03/lateral-entry-into-bureaucracy-reason- process-and-the-controversy/ 23. Which of the following state government has recently launched the country’s maiden Engineering Research & Development (ER&D) Policy? (a) Maharashtra (b) Telangana (c) Andhra Pradesh (d) Karnataka Ans: (d) Explanation: • Karnataka has launched the country’s maiden Engineering Research & Development (ER&D) Policy. • Significance of ER&D: o According to industry apex body Nasscom, ER&D has the potential to become a $100-billion industry in the country in the next five years. o The ER&D sector in the country is the fastest growing industry with a CAGR of 12.8%. Meanwhile, the global engineering research and development industry is expected to reach a spend of $2 trillion by 2025. • Highlights of the new policy: o It seeks to raise contribution to the sector in the country to 45% in the next five years. o It has the potential to create over 50,000 jobs in the ER&D space in five years. o The policy aims to prepare the State to make use of the future opportunities emanating from this sector. o It has identified five key focus sectors such as aerospace and defence; auto, auto components and EV; biotechnology, pharma and medical devices; semiconductors, telecom, ESDM; and software products. Refer: https://www.insightsonindia.com/2021/03/03/karnataka-engineering-research-policy/ 24. With reference to the International Union for Conservation of Nature and Natural Resources (IUCN) and the Convention on International Trade in Endangered Species of Wild Fauna and Flora (CITES), which of the following statements is/are correct? 1. IUCN is an organ of the United Nations and CITES is an international agreement between governments 2. IUCN runs thousands of field projects around the world to better manage natural environments. 3. CITES is legally binding on the States that have joined it, but this Convention does not take the place of national laws.

Telegram: https://t.me/insightsIAStips 15 Youtube: https://www.youtube.com/channel/UCpoccbCX9GEIwaiIe4HLjwA

Revision Through MCQs (RTM) Compilation (March 2021)

Select the correct answer using the code given below. a) 1 only b) 2 and 3 only c) 1 and 2 only d) 1, 2 and 3 Ans: (b) Explanation: • IUCN is a voluntary organization, not an agency of United Nations. Some facts about IUCN: • Founded in 1948 as the world’s first global environmental organisation • Today the largest professional global conservation network • A leading authority on the environment and sustainable development • A neutral forum for governments, NGOs, scientists, business and local communities to find practical solutions to conservation and development challenges • Thousands of field projects and activities around the world • CITES was drafted as a result of a resolution adopted in 1963 at a meeting of members of the International Union for Conservation of Nature (IUCN). States (countries) adhere voluntarily to the agreement. • States that have agreed to be bound by the Convention (‘joined’ CITES) are known as Parties. • Although CITES is legally binding on the Parties – in other words they have to implement the Convention – it does not take the place of national laws. Rather it provides a framework to be respected by each Party, which has to adopt its own domestic legislation to ensure that CITES is implemented at the national level. • Sources: https://www.iucn.org/about/ & https://cites.org/eng/disc/what.php Refer: https://www.insightsonindia.com/2021/03/03/world-wildlife-day-2/ 25. Consider the following statements regarding Convention on International Trade in Endangered Species of Wild Fauna and Flora (CITES): 1. It was founded in 1976 as a strategic alliance of the World Wide Fund for Nature (WWF) and the International Union for the Conservation of Nature (IUCN). 2. The Elephant Trade Information system (ETIS) is the CITES-mandated tool that tracks illegal trade in elephant ivory and other elephant products. Which of the given above statements is/are correct? (a) 1 only (b) 2 only (c) Both 1 and 2 (d) Neither 1 nor 2 Ans: (b) Explanation: About CITES: • The Convention on International Trade in Endangered Species of Wild Fauna and Flora (CITES) is an international regulatory treaty between 183 party states. • Formed in 1973 and regulates the international trade in over 35,000 wild species of plants and animals. • The focus of the convention is not solely on the protection of species. It also promotes controlled trade that is not detrimental to the sustainability of wild species. • The Elephant Trade Information System, commonly known as ETIS, is the CITES- mandated tool that tracks illegal trade in elephant ivory and other elephant products. o Managed and operated by TRAFFIC on behalf of the CITES Parties, ETIS has been designed to establish trends in illicit elephant product trade and changes in trends over time, and to assess whether or not such trends are related to CITES decisions for elephant conservation.

Telegram: https://t.me/insightsIAStips 16 Youtube: https://www.youtube.com/channel/UCpoccbCX9GEIwaiIe4HLjwA

Revision Through MCQs (RTM) Compilation (March 2021)

o The objectives for ETIS are also common to the other CITES programme, Monitoring Illegal Killing of Elephants (MIKE), which tracks poaching of elephants in the wild through a site-based system encompassing Africa and Asia. Refer: https://www.insightsonindia.com/2021/03/03/world-wildlife-day-2/ 26. Consider the following statements about National Board for Wildlife (NBWL): 1. It is a “Statutory Organization” constituted under the Wildlife Protection Act, 1972. 2. No alternation of boundaries in national parks and wildlife sanctuaries can be done without approval of the NBWL. 3. The NBWL is chaired by the Union Environment Minister. Which of the given above statements is/are correct? (a) 1 and 3 only (b) 1 and 2 only (c) 2 only (d) 1, 2 and 3 Ans: (b) Explanation: About National Board for Wildlife: • It is a “Statutory Organization” constituted under the Wildlife Protection Act, 1972. • Its roles is “advisory” in nature and advises the Central Government on framing policies and measures for conservation of wildlife in the country. • Primary function of the Board is to promote the conservation and development of wildlife and forests. • It has power to review all wildlife-related matters and approve projects in and around national parks and sanctuaries. • No alternation of boundaries in national parks and wildlife sanctuaries can be done without approval of the NBWL. • Composition: The NBWL is chaired by the Prime Minister. It has 47 members including the Prime Minister. Among these, 19 members are ex-officio members. Other members include three Members of Parliament (two from Lok Sabha and one from Rajya Sabha), five NGOs and 10 eminent ecologists, conservationists and environmentalists. Refer: https://www.insightsonindia.com/2021/03/03/what-is-species-recovery-programme/ 27. Consider the following statements: 1. Caracal is naturally found in India only. 2. Asiatic lion is naturally found in India only. 3. One-horned rhinoceros is naturally found in India only. Which of the statements given above is / are correct? (a) 1 only (b) 2 only (c) 1 and 2 only (d) 1, 2 and 3 Ans: (b) Explanation: • S1: The caracal is a medium-sized wild cat native to Africa, the Middle East, Central Asia, and India. • S2: The Asiatic lion’s range is restricted to the Gir National Park and environs in the Indian state of Gujarat. • S3: The one-horned rhinoceros is native to the Indian subcontinent (not only India). The Indian rhinoceros once ranged throughout the entire stretch of the Indo-Gangetic Plain, but excessive hunting and agricultural development reduced their range drastically to 11 sites in northern India and southern Nepal. • About Caracal: o Besides India, the caracal is found in several dozen countries across Africa, the Middle East, Central and South Asia.

Telegram: https://t.me/insightsIAStips 17 Youtube: https://www.youtube.com/channel/UCpoccbCX9GEIwaiIe4HLjwA

Revision Through MCQs (RTM) Compilation (March 2021)

o While it flourishes in parts of Africa, its numbers in Asia are declining. o The wildcat has long legs, a short face, long canine teeth, and distinctive ears — long and pointy, with tufts of black hair at their tips. o The iconic ears are what give the animal its name — caracal comes from the Turkish karakulak, meaning ‘black ears’. In India, it is called siya gosh, a Persian name that translates as ‘black Ear’.

Refer: https://www.insightsonindia.com/2021/03/03/what-is-species-recovery-programme/

28. Consider the following statements: 1. Surcharge is a form of tax levied or collected by the government for the development or welfare of a particular service or sector. 2. Cess is charged over and above direct and indirect taxes. 3. Currently, the cess and surcharge collected by the Centre are not part of the tax devolution. Which of the given above statements is/are correct? (a) 1 and 2 only (b) 2 and 3 only (c) 1 and 3 only (d) 1, 2 and 3 Ans: (b) Explanation: • What is cess? o S1: It is a form of tax levied or collected by the government for the development or welfare of a particular service or sector. o S2: It is charged over and above direct and indirect taxes. o Cess collected for a particular purpose cannot be used for or diverted to other purposes. o It is not a permanent source of revenue for the government, and it is discontinued when the purpose levying it is fulfilled. o S3: Currently, the cess and surcharge collected by the Centre are not part of the tax devolution. o Examples: o Education Cess, Swachh Bharat Cess, Krishi Kalyan Cess etc. • What is Surcharge? o ‘Surcharge’ is an additional charge or tax levied on an existing tax. o Unlike a cess, which is meant to raise revenue for a temporary need, surcharge is usually permanent in nature. o It is levied as a percentage on the income tax payable as per normal rates. In case no tax is due for a financial year, then no surcharge is levied. o The revenue earned via surcharge is solely retained by the Centre and, unlike other tax revenues, is not shared with States. o Collections from surcharge flow into the Consolidated Fund of India.

Refer: facts for prelims: https://www.insightsonindia.com/2021/03/03/insights-daily-current- affairs-pib-summary-3-march-2021/

29. Consider the following statements about Himalayan serow: 1. It is listed as endangered on the IUCN Red List of Threatened Species. 2. It is listed under Schedule I of The Wildlife Protection Act, 1972. 3. It has been confirmed as the newest creature to be spotted in Meghalaya. Which of the given above statements is/are correct? (a) 1 and 3 only

Telegram: https://t.me/insightsIAStips 18 Youtube: https://www.youtube.com/channel/UCpoccbCX9GEIwaiIe4HLjwA

Revision Through MCQs (RTM) Compilation (March 2021)

(b) 2 only (c) 1 and 2 only (d) 1, 2 and 3 Ans: (b) Explanation: Himalayan serow: • It is a Himalayan mammal, somewhere between a goat and an antelope. • It has been confirmed as the newest creature to be spotted in Assam. • It was spotted in the 950-sq.km Manas Tiger Reserve on December 3. • Categorised as ‘vulnerable’ in the IUCN Red List of Threatened Species. • It is listed under Schedule I of The Wildlife Protection Act, 1972, which provides absolute protection.

Refer: facts for prelims: https://www.insightsonindia.com/2021/03/03/insights-daily-current- affairs-pib-summary-3-march-2021/

30. With reference to India’s Manas National Park or Manas Wildlife Sanctuary, consider the following statements: 1. Manas is famous for its population of the wild water buffalo. 2. The Manas river is a major left bank tributary of Brahmaputra River, which passes through the heart of the national park. 3. There is only one forest village in the core of the national park. Which of the given above statements is/are correct? (a) 1 and 3 only (b) 1 only (c) 2 and 3 only (d) 1, 2 and 3 Ans: (a) Explanation: • S3: There is only one forest village, Pagrang, in the core of the national park. Apart from this village 56 more villages surround the park. Many more fringe villages are directly or indirectly dependent on the park. • S2: The name of the park is originated from the Manas River, which is named after the serpent goddess Manasa. The Manas river is a major right bank tributary of Brahmaputra River, which passes through the heart of the national park. • S1: The park is known for its rare and endangered endemic wildlife such as the Assam roofed turtle, hispid hare, golden langur and pygmy hog. Manas is famous for its population of the wild water buffalo. • It is a national park, UNESCO Natural World Heritage site, a Project Tiger reserve, an elephant reserve and a biosphere reserve in Assam, India.

Refer: facts for prelims: https://www.insightsonindia.com/2021/03/03/insights-daily-current- affairs-pib-summary-3-march-2021/

Telegram: https://t.me/insightsIAStips 19 Youtube: https://www.youtube.com/channel/UCpoccbCX9GEIwaiIe4HLjwA

Revision Through MCQs (RTM) Compilation (March 2021)

RTM- REVISION THROUGH MCQS – 4th-Mar-2021

31. Consider the following statements about the National Population Register (NPR) in India: 1. It is a register of usual residents of the country. 2. It includes both Indian citizens as well as a foreign citizen. 3. It is being prepared under provisions of the Citizenship Act 1955. Which of the given above statements is/are correct? (a) 1 and 2 only (b) 1 only (c) 2 and 3 only (d) 1, 2 and 3 Ans: (d) Explanation: • S1 & S2: The NPR is a register of usual residents of the country. It is mandatory for every usual resident of India to register in the NPR. It includes both Indian citizens as well as a foreign citizen. • The objective of the NPR is to create a comprehensive identity database of every usual resident in the country. • The first National Population Register was prepared in 2010 and updating this data was done during 2015 by conducting door to door survey. • S3: It is being prepared at the local (Village/sub-Town), sub-District, District, State and National level under provisions of the Citizenship Act 1955 and the Citizenship (Registration of Citizens and issue of National Identity Cards) Rules, 2003. • What is the meaning of usual resident? o According to the Citizenship (Registration of Citizens and issue of National Identity Cards) Rules, 2003, a usual resident is a person who has resided in a local area for the past 6 months or more or a person who intends to reside in that area for the next 6 months or more. Refer: https://www.insightsonindia.com/2021/03/04/national-population-register/ 32. Consider the following statements: 1. The Registrar General and Census Commissioner of India (RGCCI) was founded by Ministry of Statistics and Programme Implementation (MoSPI) in 1961. 2. The responsibility of conducting the decennial Census rests with the Office of the Registrar General and Census Commissioner, India. Which of the given above statements is/are correct? (a) 1 only (b) 2 only (c) Both 1 and 2 (d) Neither 1 nor 2 Ans: (b) Explanation: • S1: Registrar General and Census Commissioner of India, founded in 1961 by Government of India Ministry of Home Affairs. • S2: The responsibility of conducting the decennial Census rests with the Office of the Registrar General and Census Commissioner, India under Ministry of Home Affairs, Government of India. Refer: https://www.insightsonindia.com/2021/03/04/national-population-register/ 33. The report “Freedom in the World 2021: Democracy under Siege” was recently released by: (a) Transparency International (b) Economist Intelligence Unit (c) International Institute for Democracy and Electoral Assistance

Telegram: https://t.me/insightsIAStips 20 Youtube: https://www.youtube.com/channel/UCpoccbCX9GEIwaiIe4HLjwA

Revision Through MCQs (RTM) Compilation (March 2021)

(d) None of the above Ans: (d) Explanation: • The report “Freedom in the World 2021: Democracy under Siege” was recently released by US think-tank Freedom House. • Key findings: o Freedoms in India have reduced resulting in India being classified as ‘partly free’. o India’s score was 67, a drop from 71/100 from last year downgrading it from the free category last year. • Reasons for the downgrade: The government and its State-level allies continued to crack down on critics during the year. o The private media are vigorous and diverse, and investigations and scrutiny of politicians do occur. However, attacks on press freedom have escalated dramatically under the Modi government, and reporting has become significantly less ambitious in recent years. o Security, defamation, sedition and contempt of court laws have been used to quiet critical media voices. o Revelations of close relationships between politicians, business executives and lobbyists on one hand and leading media personalities and owners of media outlets, on the other, have dented public confidence in the press. Refer: https://www.insightsonindia.com/2021/03/04/u-s-thinktank-report-classifies-india-as- partly-free/ 34. Among the following, which one is the largest exporter of sugar in the world in the last five years? (a) Brazil (b) Thailand (c) Australia (d) India Ans: (a) Explanation: • Brazil has the highest export volume of sugar of any country, at 19.3 million metric tons as of 2019.2020. The second largest sugar exporter, Thailand, has a little over half of the export volume of Brazil, at about 10.65 million metric tons of sugar. Refer: https://www.insightsonindia.com/2021/03/04/minimum-selling-price-for-sugar/ 35. Who approves the fair and remunerative price of sugarcane in India? (a) Commission for Agricultural Costs and Prices (CACP) (b) Cabinet Committee on Economic Affairs (CCEA) (c) Agricultural produce market committee (APMC) (d) Private Sugar Mills Ans: (b) Explanation: • The Central Government announces Fair and Remunerative Prices which are determined on the recommendation of the Commission for Agricultural Costs and Prices (CACP) and are announced/approved by the Cabinet Committee on Economic Affairs, which is chaired by Prime Minister. Refer: https://www.insightsonindia.com/2021/03/04/minimum-selling-price-for-sugar/ 36. Consider the following statements regarding Anti-Dumping Duty: 1. An anti-dumping duty is a protectionist tariff that a domestic government imposes on foreign imports that it believes are priced below fair market value. 2. In India, Directorate General of Trade Remedies (DGTR), under the Ministry of Commerce & Industry takes the final call to impose the duty. Which of the above statements is/are correct? (a) 1 only

Telegram: https://t.me/insightsIAStips 21 Youtube: https://www.youtube.com/channel/UCpoccbCX9GEIwaiIe4HLjwA

Revision Through MCQs (RTM) Compilation (March 2021)

(b) 2 only (c) Both 1 and 2 (d) Neither 1 nor 2 Ans: (a) Explanation: • S1: An anti-dumping duty is a protectionist tariff that a domestic government imposes on foreign imports that it believes are priced below fair market value. • According to global trade norms, a country is allowed to impose tariffs on such dumped products to provide a level-playing field to domestic manufacturers. The duty is imposed only after a thorough investigation by a quasi-judicial body, such as DGTR in India • S2: In India, Directorate General of Trade Remedies (DGTR), under the Ministry of Commerce & Industry conducts anti-dumping investigations. o The Finance Ministry takes the final call to impose the duty. Refer: https://www.insightsonindia.com/2021/03/04/anti-dumping-duty/ 37. Consider the following statements: 1. The Nag River originates in panchgani hills, Maharashtra. 2. The Nag River Pollution Abatement Project will be implemented by the National River Conservation Directorate (NRCD). 3. NRCD was formed under the Environment (Protection) Act. Which of the given above statements is/are not correct? (a) 1 and 3 only (b) 2 and 3 only (c) 1 and 2 only (d) 1, 2 and 3 Ans: (a) Explanation: here the directive word is not correct!! • Nag River: o The Nag River is a river flowing through the city of Nagpur in Maharashtra, India. o It is known for providing the etymology for the name Nagpur. o Forming a part of the Kanhan-Pench river system, the Nag River originates in Lava hills near wadi. • Context: • The Nag River Pollution Abatement Project has been approved at a cost of Rs. 2,117.54 crores. • It will be implemented by the National River Conservation Directorate (NRCD). More>> Refer: facts for prelims: https://www.insightsonindia.com/2021/03/04/insights-daily-current- affairs-pib-summary-4-march-2021/ 38. With reference to ‘Star Rating Of Garbage Free Cities Initiative’, consider the following statements: 1. This initiative was launched by the NITI Aayog. 2. The Star Rating is supported by a robust verification mechanism by Quality Council of India. Which of the given above statements is/are correct? (a) 1 Only (b) 2 Only (c) Both 1 and 2 (d) Neither 1 nor 2 Ans: (d) Explanation: • Star Rating Of Garbage Free Cities , an initiative launched by the Ministry Of Housing And Urban Affairs

Telegram: https://t.me/insightsIAStips 22 Youtube: https://www.youtube.com/channel/UCpoccbCX9GEIwaiIe4HLjwA

Revision Through MCQs (RTM) Compilation (March 2021)

• The star rating conditions are based on 25 key parameters across the solid waste management spectrum and has been designed to both help cities assess their progress while encouraging them to move towards a better rating thereby improving their cities’ overall cleanliness and aesthetics. • How cities are give ratings? o The Star Rating is supported by self-assessment and self-verification for achieving a certain star rating. o It also ensures the involvement of citizen groups for a transparent system of self- declaration. o The self-declaration is further verified through an independent third party agency appointed by MoHUA. Refer: https://www.insightsonindia.com/2020/05/20/garbage-free-star-rating-for-the-cities/ 39. Which of the following factors responsible for the intensification of cyclones in Bay of Bengal? 1. COVID19 lockdown impact 2. Increasing sea surface temperature 3. Effect of El-Nino Select the correct answer using the code below: (a) 1 and 2 only (b) 2 and 3 only (c) 1 and 3 only (d) 1, 2 and 3 Ans: (a) Explanation: • Factors responsible for the intensification of cyclones in BoB: • Higher than normal temperatures in the Bay of Bengal (BoB) may be whetting ‘super cyclones’ and the lockdown, indirectly, may have played a role. o Cyclones gain their energy from the heat and moisture generated from warm ocean surfaces. This year, the BoB has posted record summer temperatures a fall-out, as researchers have warned, of global warming from fossil fuel emissions that has been heating up oceans. o Lockdown impact: Reduced particulate matter emissions during the lockdown meant fewer aerosols, such as black carbon, that are known to reflect sunlight and heat away from the surface. • El Nino affects the flow of moisture-bearing winds from the cooler oceans towards India, negatively impact the summer monsoon, which accounts for over 70% of annual rainfall. It remains to be seen how it would actually evolve over the next few months, but if it does retain its strength after the summer, it could mar the prospects of a good monsoon in India. Refer: https://www.insightsonindia.com/2020/05/20/hotter-oceans-spawn-super-cyclones/ 40. Consider the following statements: 1. Calcium Carbide (CaC2) is used to artificially ripen the fruits. 2. Food adulteration has been treated as criminal offence in India. Which of the given above statements is/are correct? (a) 1 Only (b) 2 only (c) Both 1 and 2 (d) Neither 1 nor 2 Ans: (c) Explanation: • Stat2: Source: The High Court has been told by the AAP government that under the penal law, food adulteration is a cognisable offence against which the police can take action, including registration of FIRs.

Telegram: https://t.me/insightsIAStips 23 Youtube: https://www.youtube.com/channel/UCpoccbCX9GEIwaiIe4HLjwA

Revision Through MCQs (RTM) Compilation (March 2021)

• Stat1: The chemical widely used for artificially ripening fruits is calcium carbide (CaC2) which contains arsenic and phosphorus, both of which can prove fatal for human beings. CaC2 is a known carcinogen - an agent having the ability to alter human cells into cancerous cells. Refer: https://www.thehindu.com/news/cities/Delhi/food-adulteration-offence-under-ipc-says- govt/article30995150.ece

RTM- REVISION THROUGH MCQS – 5th-Mar-2021

41. ‘Ease of Living Index’ (EoLI) is released by which of the following? (a) Ministry of Social Justice and Empowerment (b) Ministry of Commerce and Industry (c) Ministry of Jal Shakti (d) Ministry of Housing and Urban Affairs Ans: (d) Explanation: • The Housing and Urban Affairs Ministry has released the final rankings of Ease of Living Index (EoLI) 2020. • What is it? • The Ease of Living Index (EoLI) is an assessment tool that evaluates the quality of life and the impact of various initiatives for urban development. • It provides a comprehensive understanding of participating cities across India based on quality of life, economic-ability of a city, and its sustainability and resilience. Refer: https://www.insightsonindia.com/2021/03/05/ease-of-living-index-eoli/ 42. Which of the following countries with borders touching the Persian Gulf? 1. Bahrain 2. Iran 3. Iraq 4. Qatar 5. Oman Select the correct answer using the code below: (a) 1, 2 and 3 only (b) 2, 4 and 5 only (c) 1, 2, 3 and 4 only (d) 1, 2, 3, 4 and 5 only Ans: (c) Explanation: • Persian Gulf: The lands around the Persian Gulf are shared by eight countries namely, Bahrain, Iran, Iraq, Kuwait, Oman, Qatar, Saudi Arabia, and the United Arab Emirates.

• Refer: https://www.insightsonindia.com/2021/03/05/international-north-south-transport-corridor/

Telegram: https://t.me/insightsIAStips 24 Youtube: https://www.youtube.com/channel/UCpoccbCX9GEIwaiIe4HLjwA

Revision Through MCQs (RTM) Compilation (March 2021)

43. Consider the following statements regarding Ashgabat agreement. 1. The Ashgabat agreement is a multimodal transport agreement for creating an international transport and transit corridor facilitating transportation of goods between Central Asia and Persian Gulf. 2. The objective of the agreement is to synchronize with other transport corridors within the region including the International North–South Transport Corridor (INSTC). 3. India formally joined the agreement in 2010. Which of the above statements is/are correct? (a) 1 and 2 only (b) 2 and 3 only (c) 1 only (d) 1 and 3 only Ans: (a) Explanation: • The Ashgabat agreement is a multimodal transport agreement between the governments of Kazakhstan, Uzbekistan, Turkmenistan, Iran, India and Oman for creating an international transport and transit corridor facilitating transportation of goods between Central Asia and the Persian Gulf. The agreement came into force in April 2016. Ashgabat in Turkmenistan is the depository state for the agreement. • Pakistan has also joined the Agreement from November 2016. India formally joined in February 2018. • The objective of this agreement is to enhance connectivity within Eurasian region and synchronize it with other transport corridors within that region including the International North–South Transport Corridor (INSTC). Refer: https://www.insightsonindia.com/2021/03/05/international-north-south-transport-corridor/ 44. With reference to Major Special Forces (SF) of India, consider the following statements: 1. COBRA is a specialized unit of the Central Reserve Police Force (CRPF) that was formed to counter Naxalism in India. 2. MARCOS is the Special Forces unit of the Indian Air Force (IAF) and is responsible for conducting special operations. Which of the given above statements is/are correct? (a) 1 only (b) 2 only (c) Both 1 and 2 (d) Neither 1 nor 2 Ans: (a) Explanation: • S2: MARCOS (Marine Commandos), is a Special Forces unit that was raised by the Indian Navy in 1987 for direct action, special reconnaissance, amphibious warfare and counter-terrorism. • S1: COBRA (Commando Battalion for Resolute Action) is a specialised unit of the CRPF (Central Reserve Police Force) that was formed to counter Naxalism in India. It’s one of the few Indian Special Forces, that’s exclusively trained in guerrilla warfare. Refer: https://www.insightsonindia.com/2021/03/05/indian-special-forces-sf/ 45. Consider the following pairs: Ethnic group Native to 1. Hazaras Afghanistan 2. Yezidis Myanmar 3. Karen Iraq Which of the following pairs is/are correctly matched? (a) 1 and 2 only (b) 2 and 3 only

Telegram: https://t.me/insightsIAStips 25 Youtube: https://www.youtube.com/channel/UCpoccbCX9GEIwaiIe4HLjwA

Revision Through MCQs (RTM) Compilation (March 2021)

(c) 1 only (d) 1, 2 and 3 Ans: (c) Explanation: • The main ethnic groups living in the seven ethnic minority states of Burma are the Karen, Shan, Mon, Chin, Kachin, Rakhine and Karenni. Other main groups include the Nagas, who live in north Burma and are estimated to number more than 100,000, constituting another complex family of Tibetan-Burmese language subgroups. • The Yezidis is one of many religious minorities in Iraq. They follow Yezidism, a religion combining elements from, among others, Christianity and Islam. • Hazaras are considered to be one of the most oppressed groups in Afghanistan. Refer: facts for prelims: https://www.insightsonindia.com/2021/03/05/insights-daily-current- affairs-pib-summary-5-march-2021/ 46. 'Wagon tragedy' is sometimes mentioned in the news, is associated with: (a) Kakori Conspiracy (b) Chauri Chaura Incident (c) Jallianwala Bagh massacre (d) Mapilla rebellion Ans: (d) Explanation: • Wagon Tragedy: o A noteworthy event of the British suppression was the wagon tragedy when approximately 60 Mappila prisoners on their way to prison, were suffocated to death in a closed railway goods wagon. • What was Mapilla rebellion? o The Mapilla rebellion or Moplah Rebellion (Moplah Riots) of 1921 was the culmination of a series of riots by Moplahs (Muslims of Malabar) in the 19th and early 20th centuries against the British and the Hindu landlords in Malabar (Northern Kerala). o The year 2021 will mark the 100th year anniversary of the uprising. Refer: https://www.insightsonindia.com/2020/09/07/moplah-rioters-not-freedom-fighters-report/ 47. Which one of the following is not a member of Shanghai Cooperation Organisation (SCO)? (a) Tajikistan (b) Turkmenistan (c) Uzbekistan (d) Kazakhstan Ans: (b) Explanation: • SCO comprises eight member states, namely the Republic of India, the Republic of Kazakhstan, the People’s Republic of China, the Kyrgyz Republic, the Islamic Republic of Pakistan, the Russian Federation, the Republic of Tajikistan, and the Republic of Uzbekistan. • SCO counts four observer states, namely the Islamic Republic of Afghanistan, the Republic of Belarus, the Islamic Republic of Iran and the Republic of Mongolia. • SCO has six dialogue partners, namely the Republic of Azerbaijan, the Republic of Armenia, the Kingdom of Cambodia, the Federal Democratic Republic of Nepal, the Republic of Turkey, and the Democratic Socialist Republic of Sri Lanka. Refer: https://www.insightsonindia.com/2020/09/07/shanghai-cooperation-organisation-sco/ 48. Images captured by ISRO's Chandrayaan-1 suggests that moon is rusting along the poles. In the context of this, what may be the possible reasons behind rusting of Moon? 1. atmosphere of its own 2. presence of iron

Telegram: https://t.me/insightsIAStips 26 Youtube: https://www.youtube.com/channel/UCpoccbCX9GEIwaiIe4HLjwA

Revision Through MCQs (RTM) Compilation (March 2021)

3. presence of water and oxygen Select the correct answer using the code below: (a) 1 and 2 (b) 2 only (c) 1 and 3 (d) 1, 2 and 3 Ans: (b) Explanation: Moon may be rusting, shows ISRO’s Chandrayaan 1 images: • We know rusting exists on Earth and Mars, but now scientists have found that there is on the moon as well. The images sent by ISRO’s Chandrayaan 1 orbiter – India’s first mission to the moon, show that the moon may be rusting along the poles. • Why is this so surprising? o Rust, also known as iron oxide, is a reddish compound. It forms when the iron is exposed to water and oxygen. However, moon’s surface is not known for the presence of water and oxygen. Hence, this is surprising. • Possible reason behind this? What is the role of earth’s atmosphere in this phenomenon? o Stat2: For iron to turn rusty red, it needs what’s called an oxidizer — a molecule such as oxygen that removes electrons from a material such as iron. o Stat1 and 3: But, the moon doesn’t have an atmosphere of its own to provide sufficient amounts of oxygen, but it has trace amounts donated by Earth’s atmosphere. This terrestrial oxygen travels to the moon along an elongated extension of the planet’s magnetic field called a “magnetotail.” o At every full moon, the magnetotail blocks 99% of solar wind from blasting the moon, drawing a temporary curtain over the lunar surface, allowing periods of time for rust to form. • But, from where does the moon get water to form rust? o The moon is mostly devoid of water, save for frozen water found in lunar craters on the moon’s far side — far from where most of the hematite was found. But the researchers propose that fast-moving dust particles that bombard the moon might free water molecules locked into the moon’s surface layer, allowing the water to mix with the iron. These dust particles might even be carrying water molecules themselves, and their impact might create heat that could increase the oxidation rate. Refer: https://www.insightsonindia.com/2020/09/07/moon-may-be-rusting-shows-isros- chandrayaan-1-images/ 49. ‘Mission Sahakar 22’, sometimes mentioned in the news, which aims to: (a) improve the co-operative banking system (b) double farmers income (c) enhance the contract farming (d) provide teaser loan to farmers Ans: (b) Explanation: • NCDC is a major financial institution for cooperatives, and has started Mission Sahakar 22, which aims to double farmers’ income by 2022. o It also focusses on strengthening the farmers’ economic condition by augmenting their income and fast track the pace of development in the State. Refer: https://www.insightsonindia.com/2020/09/28/national-cooperative-development-corporation- ncdc/ 50. Consider the following statements about Sandalwood Spike Disease (SSD): 1. The disease is caused by fungal infection. 2. It was first reported in Kodagu (Karnataka) in 1899.

Telegram: https://t.me/insightsIAStips 27 Youtube: https://www.youtube.com/channel/UCpoccbCX9GEIwaiIe4HLjwA

Revision Through MCQs (RTM) Compilation (March 2021)

3. The disease is characterized by extreme reduction in leaf size accompanied by stiffening and reduction of internode length. Which of the given above statements is/are correct? (a) 1 and 2 only (b) 2 and 3 only (c) 1 and 3 only (d) 1, 2 and 3 Ans: (b) Explanation: Sandalwood Spike Disease (SSD): • The disease is caused by phytoplasma — bacterial parasites of plant tissues — which are transmitted by insect vectors. • The disease was first reported in Kodagu in 1899. • The disease is characterized by extreme reduction in leaf size accompanied by stiffening and reduction of internode length. • In advanced stage, the entire shoot gives the appearance of a spike inflorescence. • Context: o India’s sandalwood trees are facing a serious threat with the return of the destructive Sandalwood Spike Disease (SSD). o The infection has resurfaced in Karnataka and Kerala. o The present rapid spread of the infection is largely due to restrictions on green felling in forests, which has allowed vectors to spread the disease to healthy trees. Refer: facts for prelims: https://www.insightsonindia.com/2020/09/28/insights-daily-current- affairs-pib-summary-28-september-2020/

RTM- REVISION THROUGH MCQS – 6th-Mar-2021

51. Consider the following statements about Malaria: 1. Malaria is a mosquito-borne viral infection. 2. There is currently no licensed malaria vaccine in the market. Which of the given above statements is/are correct? (a) 1 only (b) 2 only (c) Both 1 and 2 (d) Neither 1 nor 2 Ans: (b) Explanation: • S1: Malaria is caused by a parasite that commonly infects a certain type of mosquito which feeds on humans. • S2: Although progress has been made in the last 10 years toward developing malaria vaccines, there is currently no licensed malaria vaccine on the market. Refer: https://www.insightsonindia.com/2021/03/06/el-salvador-becomes-first-central-american- country-to-be-declared-malaria-free/ 52. Consider the following statements regarding ‘OPEC plus’ countries: 1. OPEC+ is a group of oil-producing nations, entirely made up non-OPEC members and Russia. 2. The aim of OPEC+ was to undertake production restrictions to help revive a flailing market. Which of the above statements is/are correct? (a) 1 only (b) 2 only (c) Both 1 and 2 (d) Neither 1 nor 2

Telegram: https://t.me/insightsIAStips 28 Youtube: https://www.youtube.com/channel/UCpoccbCX9GEIwaiIe4HLjwA

Revision Through MCQs (RTM) Compilation (March 2021)

Ans: (b) Explanation: What is the Opec+? • Opec+ refers to the alliance of crude producers, who have been undertaking corrections in supply in the oil markets since 2017. • OPEC plus countries include Azerbaijan, Bahrain, Brunei, Kazakhstan, Malaysia, Mexico, Oman, Russia, South Sudan and Sudan. • The Opec and non-Opec producers first formed the alliance at a historic meeting in Algiers in 2016. • The aim was to undertake production restrictions to help resuscitate a flailing market. Refer: https://www.insightsonindia.com/2021/03/06/opec-move-to-hit-recovery/

53. With reference to Organization of the Petroleum Exporting Countries (OPEC), consider the following statements: 1. It is an intergovernmental organization of more than 25 oil producing countries. 2. It is headquartered in Vienna, Austria. 3. Its membership is open to any country that is a substantial exporter of oil and which shares the ideals of the organization. Which of the given above statements is/are correct? (a) 1 and 3 only (b) 2 and 3 only (c) 1 and 2 only (d) 1, 2 and 3 Ans: (b) Explanation: What is OPEC? • The Organization of the Petroleum Exporting Countries (OPEC) was founded in Baghdad, Iraq, with the signing of an agreement in September 1960 by five countries namely Islamic Republic of Iran, Iraq, Kuwait, Saudi Arabia and Venezuela. They were to become the Founder Members of the Organization. • OPEC is a permanent, intergovernmental organization. • OPEC’s objective is to co-ordinate and unify petroleum policies among Member Countries, in order to secure fair and stable prices for petroleum producers; an efficient, economic and regular supply of petroleum to consuming nations; and a fair return on capital to those investing in the industry. • It is headquartered in Vienna, Austria. • OPEC membership is open to any country that is a substantial exporter of oil and which shares the ideals of the organization. • The current OPEC members (13) are the following: Algeria, Angola, Equatorial Guinea, Gabon, Iran, Iraq, Kuwait, Libya, Nigeria, the Republic of the Congo, Saudi Arabia (the De facto leader), the United Arab Emirates and Venezuela. Former OPEC members are Ecuador, Indonesia and Qatar. Refer: https://www.insightsonindia.com/2021/03/06/opec-move-to-hit-recovery/ 54. Consider the following statements: 1. The only approved genetically modified crop in India till date is Bt. Cotton. 2. Genetic Engineering Approval Committee was set up by the Department of Biotechnology to regulate research, testing and commercial release of GM crops, foods and organisms. 3. The one per cent threshold for genetically modified organisms (GMO) in food crops imported into India, set by Food Safety and Standards Authority of India (FSSAI) in a recent order. Which of the given above statements is/are correct? (a) 1 and 2 only (b) 2 and 3 only (c) 1 and 3 only

Telegram: https://t.me/insightsIAStips 29 Youtube: https://www.youtube.com/channel/UCpoccbCX9GEIwaiIe4HLjwA

Revision Through MCQs (RTM) Compilation (March 2021)

(d) 1, 2 and 3 Ans: (c) Explanation: • S3: FSSAI issued an order on February 8 setting the permissible limit for genetically modified organisms (GMO) in imported food crops at 1%. • S2: The task of regulating GMO levels in imported consumables was initially with the Genetic Engineering Appraisal Committee (GEAC) under the Union environment ministry. o The body regulates the use, manufacture, storage, import and export of hazardous microorganisms or genetically-engineered organisms and cells in India. • S1: Bt cotton, the only GM crop that is allowed in India, has two alien genes from the soil bacterium Bacillus thuringiensis (Bt) that allows the crop to develop a protein toxic to the common pest pink bollworm. More>> Refer: https://www.insightsonindia.com/2021/03/06/fssai-guidelines-on-gmo-crops/ 55. With reference to India’s Initiative to Tackle Forest Fire, consider the following statements: 1. National Action Plan on Forest Fires (NAPFF) was launched in 2018 to minimise forest fires. 2. The Forest Fire Prevention and Management Scheme (FPM) is the only centrally funded program specifically dedicated to assist the states in dealing with forest fires. 3. The Ministry of Environment and Forests (MoEF) has constituted a Central Monitoring Committee (CMC) under the chairmanship of Union Environment Minister to monitor the implementation of the National Action Plan on forest fire. Which of the given above statements is/are correct? (a) 1 and 2 only (b) 2 and 3 only (c) 1 and 3 only (d) 1, 2 and 3 Ans: (a) Explanation: India’s Initiative to Tackle Forest Fire: • S1: National Action Plan on Forest Fires (NAPFF): It was launched in 2018 to minimise forest fires by informing, enabling and empowering forest fringe communities and incentivising them to work with the State Forest Departments. • S2: The Forest Fire Prevention and Management Scheme (FPM) is the only centrally funded program specifically dedicated to assist the states in dealing with forest fires. • S3: The Ministry of Environment and Forests (MoEF) has told the National Green Tribunal that it has constituted a Central Monitoring Committee (CMC) under the chairmanship of Secretary to monitor the implementation of the National Action Plan on forest fire in hill states. More>> Refer: https://www.insightsonindia.com/2021/03/06/fsi-report-on-forest-fires/ 56. Consider the following statements: 1. A ramjet is a form of air-breathing jet engine that uses the vehicle’s forward motion to compress incoming air for combustion without a rotating compressor. 2. A ramjet-powered vehicle requires an assisted take-off like a rocket assist to accelerate it to a speed where it begins to produce thrust. 3. Ramjets work most efficiently at hypersonic speeds around Mach 6 and above. Which of the given above statements is/are correct? (a) 1 and 2 only (b) 2 and 3 only (c) 1 and 3 only (d) 1, 2 and 3 Ans: (a) Explanation:

Telegram: https://t.me/insightsIAStips 30 Youtube: https://www.youtube.com/channel/UCpoccbCX9GEIwaiIe4HLjwA

Revision Through MCQs (RTM) Compilation (March 2021)

• A ramjet is a form of air-breathing jet engine that uses the vehicle’s forward motion to compress incoming air for combustion without a rotating compressor. Fuel is injected in the combustion chamber where it mixes with the hot compressed air and ignites. • A ramjet-powered vehicle requires an assisted take-off like a rocket assist to accelerate it to a speed where it begins to produce thrust. • Ramjets work most efficiently at supersonic speeds around Mach 3 (three times the speed of sound) and can operate up to speeds of Mach 6. However, the ramjet efficiency starts to drop when the vehicle reaches hypersonic speeds.

• • A scramjet engine is an improvement over the ramjet engine as it efficiently operates at hypersonic speeds and allows supersonic combustion. Thus it is known as Supersonic Combustion Ramjet, or Scramjet. • A dual mode ramjet (DMRJ) is a type of jet engine where a ramjet transforms into scramjet over Mach 4-8 range, which means it can efficiently operate both in subsonic and supersonic combustor modes. Refer: Facts for Prelims: https://www.insightsonindia.com/2021/03/06/insights-daily-current- affairs-pib-summary-6-march-2021/ 57. With reference to ‘Red Sanders’, sometimes seen in the news, consider the following statements: 1. It is a tree species found in a part of South India. 2. It is one of the most important trees in the tropical rain forest areas of South India. Which of the statements given above is/are correct? (a) 1 only (b) 2 only (c) Both 1 and 2 (d) Neither 1 nor 2 Ans: (a) Explanation: • S1: Red Sanders (a tree) has a highly restrictive distribution in the South Eastern portion of Indian peninsula (Eastern Ghats of South India) to which it is endemic. • S2: It occurs in the forest formation which is classified as “Southern Tropical Dry Deciduous Forests”. • It is generally found at altitudes of 150 – 900 m. It grows on dry, hilly, often rocky ground, and occasionally found on precipitous hill sides also. It prefers lateritic and gravelly soil and cannot tolerate water logging.

Telegram: https://t.me/insightsIAStips 31 Youtube: https://www.youtube.com/channel/UCpoccbCX9GEIwaiIe4HLjwA

Revision Through MCQs (RTM) Compilation (March 2021)

• Refer: facts for prelims: https://www.insightsonindia.com/2021/03/06/insights-daily-current- affairs-pib-summary-6-march-2021/ 58. Consider the following pairs: Geographical Indications State 1. Pochampally Sarees Telangana 2. Uppada Jamdani Sarees Tamil Nadu 3. Chanderi Sarees Which of the given above pairs is/are correctly matched? (a) 1 and 2 only (b) 1 only (c) 2 and 3 only (d) 1, 2 and 3 Ans: (b) Explanation: • Uppada Jamdani Sarees: Andhra Pradesh • Chanderi Sarees: Madhya Pradesh • More>> • Pochampally Ikat: o Pochampally Ikat or resist dyeing involves a sequence of tying and dyeing sections of bundled yarn to a predetermined colour scheme before weaving. o During the Nizam era, they were exported to Burma (Myanmar) and West Asia and east Africa, where they were known as Asia rumals. o The term `Ikat’ stems from the Malay-Indonesian expression, mangikat, meaning to bind, knot or wind. o It has now been granted the Geographical Indication (GI) status. Refer: facts for prelims: https://www.insightsonindia.com/2021/03/06/insights-daily-current- affairs-pib-summary-6-march-2021/ 59. Consider the following statements about Whale shark: 1. It is the largest living species of fish. 2. It is listed as endangered on the IUCN Red List. 3. It is found in open waters of the tropical oceans and is rarely found in water below 21 °C. Which of the given above statements is/are correct? (a) 1 and 2 only (b) 2 and 3 only (c) 1 and 3 only (d) 1, 2 and 3 Ans: (d) Explanation: • Whale shark: • The whale shark is the largest living species of fish and is endangered, according to the International Union for Conservation of Nature.

Telegram: https://t.me/insightsIAStips 32 Youtube: https://www.youtube.com/channel/UCpoccbCX9GEIwaiIe4HLjwA

Revision Through MCQs (RTM) Compilation (March 2021)

• It is protected under the Wildlife Protection Act. • The whale shark has a lifespan of around 130 years and has a unique pattern of dots on its body. • It can grow up to 10 metres in length and weigh around 20 tonnes. • Habitat: The whale shark is found in open waters of the tropical oceans and is rarely found in water below 21 °C (70 °F). Refer: facts for prelims: https://www.insightsonindia.com/2021/03/06/insights-daily-current- affairs-pib-summary-6-march-2021/ 60. Consider the following statements about Chilika Lake: 1. It is an urban lake. 2. It was designated as the first Indian wetland of international importance under the Ramsar Convention. 3. It has been listed as a tentative UNESCO World Heritage site. Which of the given above statements is/are correct? (a) 1 and 2 only (b) 2 and 3 only (c) 1 and 3 only (d) 1, 2 and 3 Ans: (b) Explanation: • Fishing cats: • The first-ever survey of the elusive fishing cat in and around Asia’s biggest brackish water lagoon, the Chilika lake in Odisha, started March 1, 2021. • Fishing cats are almost twice the size of the house cat. • They are generally found in the marshy wetlands of northern and eastern India and on the mangroves of the east coast. • Chilika Lake:- o It is a brackish water lagoon, spread over the Puri, Khurda and Ganjam districts of Odisha state on the east coast of India, at the mouth of the Daya River, flowing into the Bay of Bengal. o It is the largest coastal lagoon in India and the largest brackish water lagoon in the world after The New Caledonian barrier reef. o It has been listed as a tentative UNESCO World Heritage site o In 1981, Chilika Lake was designated the first Indian wetland of international importance under the Ramsar Convention. • Urban lakes: The apparent definition of urban lakes seems to those located entirely within city limits (census town) and directly surrounded by urban developments, with some recreation facilities limited to the shoreline area (parks, playgrounds). Refer: facts for prelims: https://www.insightsonindia.com/2021/03/06/insights-daily-current- affairs-pib-summary-6-march-2021/

RTM- REVISION THROUGH MCQS – 8th-Mar-2021

61. Consider the following statements: 1. Satras are institutional centers associated with the Ekasarana tradition of Vaishnavism. 2. Ekasarana Dharma is a panentheistic religion propagated by Srimanta Sankardeva. 3. Majuli is the centre of Assam’s Vaishnavite culture and Bhaona dance. Which of the given above statements is/are correct? (a) 1 and 2 only (b) 2 and 3 only (c) 1 and 3 only

Telegram: https://t.me/insightsIAStips 33 Youtube: https://www.youtube.com/channel/UCpoccbCX9GEIwaiIe4HLjwA

Revision Through MCQs (RTM) Compilation (March 2021)

(d) 1, 2 and 3 Ans: (d) Explanation: • S1: Sattras are monastic institutions created as part of the 16th century Neo-Vaishnavite reformist movement started by Vaishnavite saint-reformer Srimanta Sankaradeva (1449- 1596). o These Sattras/Thans were established as centres of religious, social and cultural reforms in the 16th century across Assam. o Sattras promulgate Sankardeva’s unique “worship through art” approach with music (borgeet), dance (xattriya) and theatre (bhauna). o Each Sattra has a naamghar (worship hall) as its nucleus and is headed by an influential “Sattradhikar”. Monks, known as bhakats, are inducted into Sattras at a young age. They may or may not be celibate, depending on the kind of Sattra they are inducted into. • S2: Sankardeva propagated a form of Bhakti called eka-sharana-naam-dhrama, and espoused a society based on equality and fraternity, free from caste differences, orthodox Brahmanical rituals and sacrifices. o His teaching focused on prayer and chanting (naam) instead of idol worship. His dharma was based on the four components of deva (god), naam (prayers), bhakats (devotees), and guru (teacher). • S3: Majuli is the centre of Assam’s Vaishnavite culture and Bhaona. Refer: https://www.insightsonindia.com/2021/03/08/what-are-sattras/

62. Consider the following statements: 1. A rare disease is any disease that affects a small percentage of the population. 2. The Union Ministry of Health and Family Welfare has published a national policy for the treatment of 450 ‘rare diseases’. 3. As per the GOI policy, the assistance of Rs 15 lakh will be provided to patients suffering from rare diseases under the Rashtriya Arogya Nidhi scheme. Which of the given above statements is/are correct? (a) 1 and 2 only (b) 2 and 3 only (c) 1 and 3 only (d) 1, 2 and 3 Ans: (d) Explanation: • A rare disease is any disease that affects a small percentage of the population. • Efforts by India towards this: o The Union Ministry of Health and Family Welfare has published a national policy for the treatment of 450 ‘rare diseases’. o The policy intends to kickstart a registry of rare diseases, which will be maintained by the Indian Council of Medical Research (ICMR). o Under the policy, there are three categories of rare diseases — requiring one-time curative treatment, diseases that require long-term treatment but where the cost is low, and those needing long-term treatments with high cost. Some of the diseases in the first category include osteopetrosis and immune deficiency disorders, among others. o Financial assistance: As per the policy, the assistance of Rs 15 lakh will be provided to patients suffering from rare diseases that require a one-time curative treatment under the Rashtriya Arogya Nidhi scheme. The treatment will be limited to the beneficiaries of Pradhan Mantri Jan Arogya Yojana. Refer: https://www.insightsonindia.com/2021/03/08/rare-diseases-2/

Telegram: https://t.me/insightsIAStips 34 Youtube: https://www.youtube.com/channel/UCpoccbCX9GEIwaiIe4HLjwA

Revision Through MCQs (RTM) Compilation (March 2021)

63. With reference to Brahmaputra river, consider the following statements: 1. The river originates from the Kailash ranges of Himalayas. 2. The principal tributaries of the river joining from right are the Lohit, the Dibang and the Subansiri. 3. The River flows by Kaziranga National Park in a braided course for about 53 km. Which of the given above statements is/are correct? (a) 1 and 2 only (b) 2 and 3 only (c) 1 and 3 only (d) 1, 2 and 3 Ans: (c) Explanation: • S1: ….. The river originates from the Kailash ranges of Himalayas at an elevation of 5300 M. After flowing through Tibet it enters India through Arunachal Pradesh and flows through Assam and Bangladesh before it joins Bay of Bengal….more>>> • S2: Tributaries o left: Lhasa River, , Parlung Zangbo, Lohit River, Dhansiri River, Kolong River, the Dibang o right: Kameng River, Manas River, Beki River, Raidak River, Jaldhaka River, Teesta River, • S3: The Brahmaputra River flows by Kaziranga National Park in a braided course for about 53 km……more>>> Refer: https://www.insightsonindia.com/2021/03/08/china-plans-to-build-downstream-dams-on- brahmaputra/ 64. Consider the following statements about the Sub-Mission on Agroforestry (SMAF): 1. This sub-mission is under the National Mission for Sustainable Agriculture (NMSA). 2. At present, the scheme is being implemented in 7 States and 2 UTs. Which of the given above statements is/are correct? (a) 1 only (b) 2 only (c) Both 1 and 2 (d) Neither 1 nor 2 Ans: (a) Explanation: • About the Sub-Mission on Agroforestry (SMAF): o The Department of Agriculture, Cooperation and Farmers Welfare (DAC & FW) has been implementing the Sub-Mission on Agroforestry (SMAF) since 2016-17 as part of the recommendation of the National Agroforestry Policy 2014. o This sub-mission is under the National Mission for Sustainable Agriculture (NMSA). o India was the first country to have such a comprehensive policy which was launched at the World Agroforestry Congress held in Delhi in February 2014. o At present, the scheme is being implemented in 20 States and 2 UTs. • Aim of the mission: o SMAF aims to encourage farmers to plant multi-purpose trees together with the agriculture crops for climate resilience and an additional source of income to the farmers, as well as enhanced feedstock to inter alia wood-based and herbal industry. Refer: https://www.insightsonindia.com/2021/03/08/sub-mission-on-agroforestry-smaf-scheme/ 65. In a recently held referendum, which of the following country voted in favour of banning face coverings in public, including the burka worn by Muslim women? (a) Sweden

Telegram: https://t.me/insightsIAStips 35 Youtube: https://www.youtube.com/channel/UCpoccbCX9GEIwaiIe4HLjwA

Revision Through MCQs (RTM) Compilation (March 2021)

(b) France (c) Netherlands (d) Switzerland Ans: (d) Explanation: • Switzerland to ban on face coverings in public: • In a recently held referendum, Switzerland has narrowly voted in favour of banning face coverings in public, including the burka or niqab worn by Muslim women. • Exceptions: The places where the full facial coverings will be allowed include places of worship and other sacred sites. Besides, it will be allowed for health and safety reasons and also in situations where it is “local custom” such as carnivals. • Background: Swiss people are given a direct say in their own affairs under the country’s system of direct democracy. They are regularly invited to vote on various issues in national or regional referendums. • First European Country to ban: France was the first country in Europe to ban burqas and niqabs in public places in 2011. Refer: facts for prelims: https://www.insightsonindia.com/2021/03/08/insights-daily-current- affairs-pib-summary-8-march-2021/ 66. Feni River flows through which of the following Indian states before entering the Bangladesh? 1. 2. 3. Meghalaya Select the correct answer using the code below (a) 1 only (b) 1 and 2 only (c) 2 and 3 only (d) 1, 2 and 3 Ans: (a) Explanation: • Feni is a river in southeastern Bangladesh. It is a trans-boundary river with an ongoing dispute about water rights. The Feni River originates in South Tripura district and flows through Sabroom town and then enters Bangladesh. • The question of sharing of the waters of the river between India and Pakistan was first discussed in 1958. Through at least 2006 the countries continued to consider possible compromises.

• • Maitri Setu inaugurated: o The bridge ‘Maitri Setu’ has been built over the Feni river which flows between the Indian boundary in Tripura State and Bangladesh.

Telegram: https://t.me/insightsIAStips 36 Youtube: https://www.youtube.com/channel/UCpoccbCX9GEIwaiIe4HLjwA

Revision Through MCQs (RTM) Compilation (March 2021)

o The construction was taken up by the National Highways and Infrastructure Development Corporation Ltd at a project cost of Rs. 133 Crores. o The 1.9 km long bridge joins Sabroom in India with Ramgarh in Bangladesh. Refer: facts for prelims: https://www.insightsonindia.com/2021/03/08/insights-daily-current- affairs-pib-summary-8-march-2021/ 67. Consider the following Statements: 1. The Sixth Schedule of the Constitution deals with the administration of tribal areas in the four northeastern States of Assam, Meghalaya, Tripura and Manipur. 2. Under fifth Schedule, Tribal Advisory council in the States having scheduled areas shall Consist of 20 Members. 3. Under fifth Schedule, the Governor is empowered to declare an area to be a Scheduled Area. Which of the given above statements is/are correct? a) 1 and 2 Only b) 2 Only c) 2 and 3 only d) 3 Only Ans: (b) Explanation: • S1: The Sixth Schedule of the Constitution deals with the administration of tribal areas in the four northeastern States of Assam, Meghalaya, Tripura and Mizoram [Not Manipur]. • S2: According to fifth Schedule, Each state having scheduled areas has to establish a tribes advisory council to advise on welfare and advancement of the scheduled tribes. It is to consist of 20 members, three-fourths of whom are to be the representatives of the scheduled tribes in the state legislative assembly. • S3: Under fifth Schedule, The president [Not the Governor] is empowered to declare an area to be a scheduled area. He can also increase or decrease its area, alter its boundary lines, rescind such designation or make fresh orders for such redesignation on an area in consultation with the governor of the state concerned. Refer: https://www.insightsonindia.com/2020/12/23/hc-orders-composite-floor-test-in-btc/ 68. Who among the following is/are empowered by the Indian Constitution to restrict the territorial extent of Central Legislation with respect to tribal areas? a) Governors of Tripura, Meghalaya and Manipur b) President of India c) Attorney General of India d) Chief Minister Ans: (b) Explanation: • The Governor of Assam may likewise direct that an act of Parliament does not apply to a tribal area (autonomours district) in the state or apply with specified modifications and exceptions. The President enjoys the same power with respect to tribal areas (autonomous districts) in Meghalaya, Tripura and Mizoram. Refer: https://www.insightsonindia.com/2020/12/23/hc-orders-composite-floor-test-in-btc/ 69. Recently, our scientists have discovered a new species of the wild sun rose which has unique features such as a tuberous root, no hair in its leaf axils, a reddish-pink flower, prolate-shaped fruits, and copper brown seeds without lustre. In which part of India has it been discovered? (a) Andaman Islands (b) Western Ghats (c) Tropical rain forests of northeast (d) Eastern Ghats Ans: (d) Explanation: Portulaca laljii:

Telegram: https://t.me/insightsIAStips 37 Youtube: https://www.youtube.com/channel/UCpoccbCX9GEIwaiIe4HLjwA

Revision Through MCQs (RTM) Compilation (March 2021)

• It is a new species of the wild sun rose discovered recently from the Eastern Ghats in India. • Discovered from Prakasam district of Andhra Pradesh. • It has unique features such as a tuberous root, no hair in its leaf axils, a reddish-pink flower, prolate-shaped fruits, and copper brown seeds without lustre. • The succulent nature of tuberous roots allow the plant to survive on rocky crevices.

• Refer: Facts for Prelims: https://www.insightsonindia.com/2020/12/28/insights-daily-current- affairs-pib-summary-28-december-2020/ 70. Which one of the following suggested that the Union should only transfer those subjects into the Concurrent List, which are central to achieving demonstrable national interest? (a) 15th Finance Commission (2020) (b) Punchhi Commission (2010) (c) National Commission to Review the Working of the Constitution (2000) (d) Rajamannar Committee (1969) Ans: (b) Explanation: • The Confederation of Indian Industry (CII) has sought the inclusion of tourism in the concurrent list to enable the Centre and States to effectively regulate the sector as well as frame policies for growth. • Seventh Schedule: o The seventh schedule under Article 246 of the constitution deals with the division of powers between the union and the states. o It contains three lists- Union List, State List and Concurrent List. o The union list details the subjects on which Parliament may make laws while the state list details those under the purview of state legislatures. o The concurrent list on the other hand has subjects in which both Parliament and state legislatures have jurisdiction. However the Constitution provides federal supremacy to Parliament on concurrent list items in case of a conflict. • The commission on Centre-State Relations, headed by Justice M M Punchhi, in 2010 recommended that the Union should only transfer those subjects into the Concurrent List, which are central to achieving demonstrable national interest. Refer: https://www.insightsonindia.com/2020/12/31/seventh-schedule/

RTM- REVISION THROUGH MCQS – 9th-Mar-2021

71. Consider the following statements about the archaeological site Dholavira (Gujarat): 1. Its location is on the Tropic of Cancer. 2. It was first excavated by an archaeologist, Sir John Marshall in 1872.

Telegram: https://t.me/insightsIAStips 38 Youtube: https://www.youtube.com/channel/UCpoccbCX9GEIwaiIe4HLjwA

Revision Through MCQs (RTM) Compilation (March 2021)

3. It has been submitted for nomination of World Heritage Site in 2019-2020. Which of the given above statements is/are correct? (a) 1 and 2 only (b) 2 and 3 only (c) 1 and 3 only (d) 1, 2 and 3 Ans: (c) Explanation: • S1: Dholavira’s location is on the Tropic of Cancer. More>>> • S2: The site was discovered in 1967-68 by J. P. Joshi, of the Archaeological Survey of India (ASI), and is the fifth largest of eight major Harappan sites. • S3: Dholavira: A Harappan City’ has been submitted for nomination of World Heritage Site in 2019-2020. Refer: https://www.insightsonindia.com/2021/03/09/declaration-of-world-heritage-sites-by-unesco/ 72. Consider the following statements: 1. World Heritage Sites are designated by UNESCO for having cultural, historical, scientific or other form of significance. 2. Each World Heritage Site remains part of the legal territory of the UNESCO. 3. At present, India has 38 World Heritage Properties. Which of the given above statements is/are correct? (a) 1 and 2 only (b) 1 and 3 only (c) 2 and 3 only (d) 1, 2 and 3 Ans: (b) Explanation: • What is a World Heritage site? o A World Heritage site is classified as a natural or man-made area or a structure that is of international importance, and a space which requires special protection. o S1: These sites are officially recognised by the UN and the United Nations Educational Scientific and Cultural Organisation, also known as UNESCO. o UNESCO believes that the sites classified as World Heritage are important for humanity, and they hold cultural and physical significance. • Key facts: o The list is maintained by the international World Heritage Programme administered by the UNESCO World Heritage Committee, composed of 21 UNESCO member states which are elected by the General Assembly. o S2: Each World Heritage Site remains part of the legal territory of the state wherein the site is located and UNESCO considers it in the interest of the international community to preserve each site. o To be selected, a World Heritage Site must be an already classified landmark, unique in some respect as a geographically and historically identifiable place having special cultural or physical significance. • S3: At present, India has 38 World Heritage Properties. Besides, India has 42 sites listed under Tentative List which is a pre-requisite condition for inscription as World Heritage Site. Refer: https://www.insightsonindia.com/2021/03/09/declaration-of-world-heritage-sites-by-unesco/ 73. The ‘Indira Sawhney Case’ (1992) was in news recently, is associated with which of the following? (a) Reservation in India (b) Right to Privacy (c) Federalism in India (d) Capital punishment in India

Telegram: https://t.me/insightsIAStips 39 Youtube: https://www.youtube.com/channel/UCpoccbCX9GEIwaiIe4HLjwA

Revision Through MCQs (RTM) Compilation (March 2021)

Ans: (a) Explanation: • In its landmark 1992 decision in Indra Sawhney vs Union of India, the Supreme Court had held that reservations under Article 16(4) could only be provided at the time of entry into government service but not in matters of promotion. • It added that the principle would operate only prospectively and not affect promotions already made and that reservation already provided in promotions shall continue in operation for a period of five years from the date of the judgment. It also ruled that the creamy layer can be and must be excluded. • On June 17, 1995, Parliament, acting in its constituent capacity, adopted the seventy- seventh amendment by which clause (4A) was inserted into Article 16 to enable reservation to be made in promotion for SCs and STs. The validity of the seventy-seventh and eighty-fifth amendments to the Constitution and of the legislation enacted in pursuance of those amendments was challenged before the Supreme Court in the Nagaraj case. • Upholding the validity of Article 16 (4A), the court then said that it is an enabling provision. “The State is not bound to make reservation for the SCs and STs in promotions. But, if it seeks to do so, it must collect quantifiable data on three facets — the backwardness of the class; the inadequacy of the representation of that class in public employment; and the general efficiency of service as mandated by Article 335 would not be affected”. • The court ruled that the constitutional amendments do not abrogate the fundamentals of equality. Refer: https://www.insightsonindia.com/2021/03/09/sc-seeks-states-views-on-50-cap-on-quota/

74. Consider the following statements: 1. The Parliament of India can place a particular law in the Ninth Schedule of the Constitution of India. 2. The validity of a law passed in the Ninth Schedule cannot be examined by any court and no judgment can be made on it. Which of the statements given above is/are correct? (a) 1 only (b) 2 only (c) Both 1 and 2 (d) Neither 1 nor 2 Ans: (a) Explanation: • S1: Once a law is enacted and included in the Ninth Schedule, it gets protection under Article 31-B (validation of certain Acts and Regulations) and is not subject to judicial scrutiny. The Ninth Schedule (Article 31-B) was introduced by the former Prime Minister Jawaharlal Nehru to keep certain laws particularly those on land reforms beyond the scope of judicial review. • S2: The mandate of ninth schedule is to prevent judicial scrutiny but in a landmark ruling in IR Coelho versus State of Tamil Nadu, 2007, the Supreme Court of India ruled that all laws (including those in the Ninth Schedule) would be open to Judicial Review if they violated the basic structure of the constitution. The Supreme Court judgment laid that the laws placed under Ninth Schedule after April 24, 1973. Shall be open to challenge in court if they violated fundamental rights guaranteed under Article 14, 19, 20 and 21 of the Constitution • Context: Tamilnadu’s case: o The state’s Assembly passed the Tamil Nadu Backward Classes, Scheduled Castes and Scheduled Tribes (Reservation of Seats in Educational Institutions

Telegram: https://t.me/insightsIAStips 40 Youtube: https://www.youtube.com/channel/UCpoccbCX9GEIwaiIe4HLjwA

Revision Through MCQs (RTM) Compilation (March 2021)

and Appointments or Posts in the Services under the State) Act, 1993 to keep its reservation limit intact at 69%. o The law was subsequently included into the Ninth Schedule of the Constitution through the 76th Constitution Amendment passed by Parliament in 1994. Refer: https://www.insightsonindia.com/2021/03/09/sc-seeks-states-views-on-50-cap-on-quota/ 75. The Ninth Schedule was introduced in the Constitution of India during the prime ministership of: (a) Jawaharlal Nehru (b) Lal Bahadur Shastri (c) Indira Gandhi (d) Morarji Desai Ans: (a) Explanation: • The first amendment to the Indian Constitution added the Ninth Schedule to it. It was introduced by the Nehru Government, on 10 May 1951 to address judicial decisions and pronouncements especially about the chapter on fundamental rights. • See https://en.wikipedia.org/wiki/Constitution_of_India Refer: https://www.insightsonindia.com/2021/03/09/sc-seeks-states-views-on-50-cap-on-quota/ 76. Consider the following statements about Banks Board Bureau (BBB): 1. The BBB is a not-for-profit organisation registered under Section 8 of the Companies Act 2013. 2. The BBB was the part of Indradhanush Plan of government. 3. It will make recommendations for appointment of whole-time directors as well as non- executive chairpersons of Public Sector Banks (PSBs) and state-owned financial institutions. Which of the given above statements is/are correct? (a) 1 and 2 only (b) 2 and 3 only (c) 1 and 3 only (d) 1, 2 and 3 Ans: (b) Explanation: About BBB: • It was set up in February 2016 as an autonomous body– based on the recommendations of the RBI-appointed Nayak Committee. • It was part of the Indradhanush Plan. • It will make recommendations for appointment of whole-time directors as well as non- executive chairpersons of Public Sector Banks (PSBs) and state-owned financial institutions. • The Ministry of Finance takes the final decision on the appointments in consultation with the Prime Minister’s Office. • Composition: o Banks Board Bureau comprises the Chairman, three ex-officio members i.e Secretary, Department of Public Enterprises, Secretary of the Department of Financial Services and Deputy Governor of the Reserve Bank of India, and five expert members, two of which are from the private sector. Refer: https://www.insightsonindia.com/2021/03/09/banks-board-bureau-bbb/ 77. Consider the following statements: 1. The Van Dhan Scheme is an initiative of the Ministry of Tribal Affairs and TRIFED. 2. The Tribal Co-operative Marketing Federation of India (TRIFED) is a statutory body and was established in 2002. Which of the given above statements is/are correct? (a) 1 only (b) 2 only

Telegram: https://t.me/insightsIAStips 41 Youtube: https://www.youtube.com/channel/UCpoccbCX9GEIwaiIe4HLjwA

Revision Through MCQs (RTM) Compilation (March 2021)

(c) Both 1 and 2 (d) Neither 1 nor 2 Ans: (a) Explanation: • S1: The Van Dhan Scheme is an initiative of the Ministry of Tribal Affairs and TRIFED. It was launched on 14th April, 2018 and seeks to improve tribal incomes through value addition of tribal products. • S2: Tribal Cooperative Marketing Development Federation of India (TRIFED): It is a statutory body and was established in August 1987. It has been registered as a National level Cooperative body by the Government of the country. Refer: https://www.insightsonindia.com/2021/03/09/van-dhan-vikas-kendras-initiative/ 78. With reference to Properties of Microwaves, consider the following statements: 1. Microwaves radiate electromagnetic energy with shorter wavelength. 2. Microwaves are reflected by Ionosphere. 3. Microwaves can pass through glass and plastic. Which of the given above statements is/are correct? (a) 1 and 2 only (b) 2 and 3 only (c) 1 and 3 only (d) 1, 2 and 3 Ans: (c) Explanation: • What are Microwaves? o Microwaves are defined as electromagnetic radiations with a frequency ranging between 300 MHz to 300 GHz while the wavelength ranges from 1 mm to around 30 cm. o They fall between the infrared radiation and radio waves in the electromagnetic spectrum. • Properties of microwaves: o Metal surfaces reflect microwaves. o Microwaves of certain frequencies are absorbed by water. o Microwave transmission is affected by wave effects such as refraction, reflection, interference, and diffraction. o S3: Microwaves can pass through glass and plastic. o S2: Microwaves are not reflected by Ionosphere. o S1: Microwaves are the waves that radiate electromagnetic energy with shorter wavelength. Refer: https://www.insightsonindia.com/2020/12/07/microwave-energy-likely-made-u-s-officials-ill/ 79. The words ‘Shalya tantra’ and ‘Shalakya tantra’ are mentioned in media in reference to: (a) the ancient medical practice in India (b) the earliest dharmashastra texts written in aphoristic (Sutra) style (c) an ancient wind-powered iron smelting technology in India (d) powerful merchant guilds of early medieval south India Ans: (a) Explanation: • The government has notified compulsory surgical procedures for PG students of Ayurveda. • How far is surgery part of Ayurveda? • There are two branches of surgery in Ayurveda: o Shalya Tantra, which refers to general surgery, and Shalakya Tantra which pertains to surgeries related to the eyes, ears, nose, throat and teeth.

Telegram: https://t.me/insightsIAStips 42 Youtube: https://www.youtube.com/channel/UCpoccbCX9GEIwaiIe4HLjwA

Revision Through MCQs (RTM) Compilation (March 2021)

o All postgraduate students of Ayurveda have to study these courses, and some go on to specialise in these, and become Ayurveda surgeons. • The 2016 regulations allow postgraduate students to specialise in Shalya Tantra, Shalakya Tantra, and Prasuti evam Stree Roga (Obstetrics and Gynecology), the three disciplines involving major surgical interventions. • Students of these three disciplines are granted MS (Master in Surgery in Ayurveda) degrees. Refer: https://www.insightsonindia.com/2020/12/05/surgery-as-part-of-ayurveda/ 80. With reference to solar power production in India, consider the following statements: 1. India is the third largest in the world in the manufacture of silicon wafers used in photovoltaic units. 2. The solar power tariffs are determined by the Solar Energy Corporation of India. Which of the statements given above is/are correct? (a) 1 only (b) 2 only (c) Both 1 and 2 (d) Neither 1 nor 2 Ans: (d) Explanation: • S1: Every solar panel which is made in India is assembled while all the material comes from China, Europe and some other countries. India is yet to develop semiconductor clusters. o Read more: https://www.business-standard.com/article/news-ani/blue-wafer- solar-cells-not-to-qualify-under-domestically-made-product-to-get-govt-benefits- govt-clarifies-119102101265_1.html • S2: Central Electricity Regulatory Commission determines solar power tariffs and regulates the tariff of generating companies owned or controlled by the Central Government. • Solar Energy Corporation of India (SECI) has a power-trading license, but it does not set solar power tariffs. o SECI is a company of the Ministry of New and Renewable Energy. o It is established to facilitate the implementation of the National Solar Mission (NSM). o It is the only Central Public Sector Undertaking dedicated to the solar energy sector. o Refer: https://www.seci.co.in/ Refer: https://www.insightsonindia.com/2020/12/05/what-is-indias-sprawling-renewable-energy- park-coming-up-on-its-border-with-pakistan/

Telegram: https://t.me/insightsIAStips 43 Youtube: https://www.youtube.com/channel/UCpoccbCX9GEIwaiIe4HLjwA

Revision Through MCQs (RTM) Compilation (March 2021)

RTM- REVISION THROUGH MCQS – 10th-Mar-2021

81. Consider the following statements about the Konark Sun Temple: 1. It was built by King Narasimhadeva I of Ganga dynasty. 2. It had the tallest shikhara amongst the temples of its time. 3. It was included in UNESCO World Heritage Site in 1984. Which of the given above statements is/are correct? (a) 1 and 2 only (b) 2 and 3 only (c) 1 and 3 only (d) 1, 2 and 3 Ans: (c) Explanation: About the Temple: • Built in the 13th century, the Konark temple was conceived as a gigantic chariot of the Sun God, with 12 pairs of exquisitely ornamented wheels pulled by seven horses. • It was built by King Narasimhadeva I, the great ruler of Ganga dynasty. • The temple was included in UNESCO World Heritage Site in 1984 for its architectural greatness and also for the sophistication and abundance of sculptural work. • The temple is a perfect blend of Kalinga architecture, heritage, exotic beach and salient natural beauty. • It is protected under the National Framework of India by the Ancient Monuments and Archaeological Sites and Remains (AMASR) Act (1958) and its Rules (1959). • The Konark is the third link of Odisha’s Golden Triangle. The first link is Jagannath Puri and the second link is Bhubaneswar (Capital city of Odisha). • This temple was also known as ‘BLACK PAGODA’ due to its dark color and used as a navigational landmark by ancient sailors to Odisha. Similarly, the Jagannath Temple in Puri was called the “White Pagoda”. • It remains a major pilgrimage site for Hindus, who gather here every year for the Chandrabhaga Mela around the month of February. Refer: https://www.insightsonindia.com/2021/03/10/konark-sun-temple-3/

82. Consider the following statements about the Archaeological Survey of India (ASI): 1. It is an Indian government agency attached to the Ministry of Mines. 2. It was founded in 1861 by Alexander Cunningham who also became its first Director-General. 3. It is responsible for archaeological research and the conservation and preservation of cultural monuments in India. Which of the given above statements is/are correct? (a) 1 and 2 only (b) 2 and 3 only (c) 1 and 3 only (d) 1, 2 and 3 Ans: (b) Explanation: • The Archaeological Survey of India is an Indian government agency attached to the Ministry of Culture that is responsible for archaeological research and the conservation and preservation of cultural monuments in the country. It was founded in 1861 by Alexander Cunningham who also became its first Director-General. Refer: https://www.insightsonindia.com/2021/03/10/konark-sun-temple-3/ 83. Consider the following statements about National Social Assistance Programme (NSAP): 1. It is a Centrally Sponsored Scheme under the Ministry of Rural Development. 2. This programme is being implemented in rural areas as well as urban areas.

Telegram: https://t.me/insightsIAStips 44 Youtube: https://www.youtube.com/channel/UCpoccbCX9GEIwaiIe4HLjwA

Revision Through MCQs (RTM) Compilation (March 2021)

3. The annual verification and Social Audit has been introduced under NSAP. Which of the given above statements is/are correct? (a) 1 and 2 only (b) 2 and 3 only (c) 1 and 3 only (d) 1, 2 and 3 Ans: (d) Explanation: • S1 & S2: The National Social Assistance Programme (NSAP) is a welfare programme being administered by the Ministry of Rural Development. This programme is being implemented in rural areas as well as urban areas. NSAP represents a significant step towards the fulfilment of the Directive Principles of State Policy enshrined in the Constitution of India which enjoin upon the State to undertake within its means a number of welfare measures. • S3: Annual verification and Social Audit has been introduced under NSAP. All the States are to complete the Annual verification by 30th June and the Social Audit by 30th September, each year. More>> Refer: https://www.insightsonindia.com/2021/03/10/panel-flags-centres-meagre-pensions/ 84. Consider the following statements: 1. As per the provisions under Environment Protection Act, 1986, the Centre can notify an ecologically important area as Eco-Sensitive Zones. 2. The National Green Tribunal (NGT) is not empowered to hear matters pertaining to issues coming under the ambit of the Environment Protection Act, 1986. Which of the given above statements is/are correct? (a) 1 only (b) 2 only (c) Both 1 and 2 (d) Nether 1 nor 2 Ans: (a) Explanation: • S1: The Environment Protection Act, 1986 does not mention the word “Eco-sensitive Zones”. The section 3(2)(v) of the Act, says that Central Government can restrict areas in which any industries, operations or processes shall not be carried out or shall be carried out subject to certain safeguards • Besides the section 5 (1) of this act says that central government can prohibit or restrict the location of industries and carrying on certain operations or processes on the basis of considerations like the biological diversity of an area, maximum allowable limits of concentration of pollutants for an area, environmentally compatible land use, and proximity to protected areas. • S2: The NGT has the power to hear all civil cases relating to environmental issues and questions that are linked to the implementation of laws listed in Schedule I of the NGT Act. These include the following: o The Water (Prevention and Control of Pollution) Act, 1974; o The Water (Prevention and Control of Pollution) Cess Act, 1977; o The Forest (Conservation) Act, 1980; o The Air (Prevention and Control of Pollution) Act, 1981; o The Environment (Protection) Act, 1986; o The Public Liability Insurance Act, 1991; o The Biological Diversity Act, 2002 Refer: https://www.insightsonindia.com/2021/03/10/sc-questions-delay-in-setting-up-environment- regulator/ 85. The National Green Tribunal can hear cases related to which of the following Acts? 1. The Water (Prevention and Control of Pollution) Act, 1974.

Telegram: https://t.me/insightsIAStips 45 Youtube: https://www.youtube.com/channel/UCpoccbCX9GEIwaiIe4HLjwA

Revision Through MCQs (RTM) Compilation (March 2021)

2. The Environment (Protection) Act, 1986. 3. The Biological Diversity Act, 2002. 4. Wildlife (Protection) Act, 1972. Select the correct answer using the code below: (a) 1, 2 and 3 only (b) 2, 3 and 4 only (c) 1 and 4 only (d) 2 and 3 only Ans: (a) Explanation: • NGT has not been vested with powers to hear any matter relating to the Wildlife (Protection) Act, 1972, the Indian Forest Act, 1927 and various laws enacted by States relating to forests, tree preservation etc. Refer: https://www.insightsonindia.com/2021/03/10/sc-questions-delay-in-setting-up-environment- regulator/ 86. Recently, which of the following country has launched the virus passport? (a) Singapore (b) China (c) India (d) Australia Ans: (b) Explanation: Virus passport: • Launched by China. • Basically, it is a health certificate programme for Chinese international travellers. • The digital certificate shows a user’s vaccination status and virus test results. • Although the certificate is meant for travel in and out of China, it is currently only available for use by Chinese citizens, and it is not yet mandatory. There is also no indication authorities in other countries will use it when Chinese travellers go abroad. Refer: facts for prelims: https://www.insightsonindia.com/2021/03/10/insights-daily-current- affairs-pib-summary-10-march-2021/ 87. Consider the following statements about NISAR (satellite) mission: 1. It is a joint project between NASA and ISRO. 2. The satellite will be the first radar imaging satellite to use dual frequencies. 3. It will be used for remote sensing, to observe and understand natural processes on Earth. Which of the given above statements is/are correct? (a) 1 and 2 only (b) 2 and 3 only (c) 1 and 3 only (d) 1, 2 and 3 Ans: (d) Explanation: • The NASA-ISRO Synthetic Aperture Radar (NISAR) mission is a joint project between NASA and ISRO to co-develop and launch a dual-frequency synthetic aperture radar on an Earth observation satellite. • The satellite will be the first radar imaging satellite to use dual frequencies. It will be used for remote sensing, to observe and understand natural processes on Earth. For example, its left-facing instruments will study the Antarctic cryosphere. • Synthetic Aperture Radar (SAR): o A Synthetic Aperture Radar (SAR) or SAR, is a coherent mostly airborne or spaceborne side looking radar system which utilizes the flight path of the platform to simulate an extremely large antenna, and that generates high- resolution remote sensing imagery.

Telegram: https://t.me/insightsIAStips 46 Youtube: https://www.youtube.com/channel/UCpoccbCX9GEIwaiIe4HLjwA

Revision Through MCQs (RTM) Compilation (March 2021)

• Why in News? o ISRO has completed development of a Synthetic Aperture Radar capable of producing high-resolution images for a joint earth observation satellite mission with the U.S. space agency NASA. The mission is targeted to be launched in 2022. Refer: facts for prelims: https://www.insightsonindia.com/2021/03/10/insights-daily-current- affairs-pib-summary-10-march-2021/ 88. Dustlik II is a joint military exercise between India and which of the following country? (a) Turkmenistan (b) Uzbekistan (c) Jordan (d) Japan Ans: (b) Explanation: Joint Exercise Dustlik: • Second Edition of Joint Exercise Dustlik between Indian Army and Uzbekistan Army is being held in India. • The exercise is based on counter-terrorist operations and aims at enhancing military cooperation. Refer: facts for prelims: https://www.insightsonindia.com/2021/03/10/insights-daily-current- affairs-pib-summary-10-march-2021/ 89. Consider the following statements: 1. The jute plant needs a plain alluvial soil and standing water. 2. The first jute mill in India was set up in Bombay. 3. India is the world’s largest producer of raw jute and jute goods. Which of the given above statements is/are correct? (a) 1 and 3 only (b) 2 and 3 only (c) 1 only (d) 2 only Ans: (a) Explanation: • S2: The word jute was first published by Roxburg, the then Superintendent of Botanical Garden, Shibpur, West Bengal in the year 1795, in his letter addressed to the Board of Directors of East India Company. The first jute mill in India was set up in the year 1855 at Rishra, near Kolkata. • S1: Jute can grow in wide range of soil but fertile loamy alluvial soil is better suitable. • S3: India is the world’s largest producer of raw jute and jute goods. Refer: https://www.insightsonindia.com/2020/10/30/mandatory-packaging-in-jute-materials/ 90. Consider the following statements: 1. National Productivity Council of India (NPC) was set up by NITI Aayog. 2. NPC provides consultancy and training services to the Government and Public & Private sector organizations. Which of the given above statements is/are correct? (a) 1 only (b) 2 only (c) Both 1 and 2 (d) Neither 1 nor 2 Ans: (b) Explanation: National Productivity Council (NPC): • NPC has been granted accreditation conforming to ISO 17020:2012 by National Accreditation Board for Certification Body (NABCB), Quality Council of India (QCI)

Telegram: https://t.me/insightsIAStips 47 Youtube: https://www.youtube.com/channel/UCpoccbCX9GEIwaiIe4HLjwA

Revision Through MCQs (RTM) Compilation (March 2021)

for undertaking inspection and audit work in the area of Food Safety Audit and Scientific Storage of Agricultural Products. • This accreditation is valid for a period of three years. • Implications: This accreditation will enable it to undertake Independent Third-Party Audits of Food Business Operators including Food Storage Warehouses as per Food Safety and Standards (Food Safety Auditing) Regulations, 2018 of FSSAI as well as inspections of Warehouses as per Warehousing Development and Regulatory Authority (WDRA) Rules, 2017. • About NPC: o Established by the Ministry of Commerce and Industry, Government of India in 1958, it is an autonomous, multipartite, non-profit organization. o Besides undertaking research in the area of productivity, NPC has been providing consultancy and training services in areas of Industrial Engineering, Agri- Business, Economic Services, Quality Management, Human Resources Management, Information Technology, Technology Management, Energy Management, Environmental Management etc., to the Government and Public & Private sector organizations. o NPC is a constituent of the Tokyo-based Asian Productivity Organisation (APO), an Intergovernmental Body, of which the Government of India is a founder member. Refer: Facts for Prelims: https://www.insightsonindia.com/2020/10/30/insights-daily-current- affairs-pib-summary-30-october-2020/

Telegram: https://t.me/insightsIAStips 48 Youtube: https://www.youtube.com/channel/UCpoccbCX9GEIwaiIe4HLjwA

Revision Through MCQs (RTM) Compilation (March 2021)

RTM- REVISION THROUGH MCQS – 11th-Mar-2021

91. With reference to ‘Powers and Functions of the Chief Minister’, consider the following statements: 1. He can recommend the dissolution of the legislative assembly to the speaker of the house at any time. 2. He is the crisis manager-in-chief at the political level during emergencies. 3. He allocates and reshuffles the portfolios among ministers. Which of the given above statements is/are correct? (a) 1 and 2 only (b) 2 and 3 only (c) 1 and 3 only (d) 1, 2 and 3 Ans: (b) Explanation: • S1: The Chief Minister enjoys the following powers as the leader of the house: (a) He advises the governor with regard to the summoning and proroguing of the sessions of the state legislature. (b) He can recommend the dissolution of the legislative assembly to the governor at any time. (c) He announces the government policies on the floor of the house. • S2: He is the crisis manager-in-chief at the political level during emergencies. • S3: He allocates and reshuffles the portfolios among ministers. Refer: https://www.insightsonindia.com/2021/03/11/the-chief-minister-appointment-power- function-and-position-2/ 92. With reference to parliamentary devices to control the executive in India, consider following statements: 1. A motion of “No Confidence Motion” against the Government can be introduced only in the Lok Sabha. 2. The Constitution of India does not mention about either a Confidence or a No Confidence Motion. Which of the given above statements is/are incorrect? (a) 1 only (b) 2 only (c) Both 1 and 2 (d) Neither 1 nor 2 Ans: (d) Explanation: here the directive word is incorrect!! • What is a no-confidence motion? o A no-confidence motion is a parliamentary motion which is moved in the Lok Sabha against the entire council of ministers, stating that they are no longer deemed fit to hold positions of responsibility due to their inadequacy in some respect or their failure to carry out their obligations. No prior reason needs to be stated for its adoption in the Lok Sabha. • Procedure to move a “No Confidence Motion”: o A motion of “No Confidence Motion” against the Government can be introduced only in the Lok Sabha under rule 198. o The Constitution of India does not mention about either a Confidence or a No Confidence Motion. Although, Article 75 does specify that the Council of Ministers shall be collectively responsible to the Lok Sabha. o A motion of No Confidence can be admitted when a minimum of 50 members, support the motion in the house. o The Speaker then, once satisfied that the motion is in order, will ask the House if the motion can be adopted.

Telegram: https://t.me/insightsIAStips 49 Youtube: https://www.youtube.com/channel/UCpoccbCX9GEIwaiIe4HLjwA

Revision Through MCQs (RTM) Compilation (March 2021)

o If the motion is passed in the house, the Government is bound to vacate the office. o A no-confidence motion needs a majority vote to pass the House. o If individuals or parties abstain from voting, those numbers will be removed from the overall strength of the House and then the majority will be taken into account. Refer: https://www.insightsonindia.com/2021/03/11/no-trust-vote/ 93. Among the following cities, which one lies on a longitude closest to that of Delhi? (a) Gwalior (b) Bhopal (c) Nagpur (d) Pune Ans: (b) Explanation:

• Refer: https://www.insightsonindia.com/2021/03/11/national-capital-territory-bill/ 94. With reference to Pradhan Mantri Swasthya Suraksha Nidhi (PMSSN), consider the following statements: 1. The fund is a public charitable trust with the Prime Minister as its Chairman. 2. The fund consists entirely of public contributions and does not get any budgetary support. Which of the given above statements is/are correct? (a) 1 only (b) 2 only (c) Both 1 and 2 (d) Neither 1 nor 2 Ans: (d) Explanation: About the Pradhan Mantri Swasthya Suraksha Nidhi:: • S1 and S2: It will be a single non-lapsable reserve fund for share of Health from the proceeds of Health and Education Cess. • The accruals into the PMSSN will be utilised for the flagship schemes of the Health Ministry including Ayushmann Bharat–Pradhan Mantri Jan Arogya Yojana (AB-PMJAY) and National Health Mission and Pradhan Mantri Swasthya Suraksha Yojana (PMSSY) and also disaster preparedness, and responses during health emergencies.

Telegram: https://t.me/insightsIAStips 50 Youtube: https://www.youtube.com/channel/UCpoccbCX9GEIwaiIe4HLjwA

Revision Through MCQs (RTM) Compilation (March 2021)

• In any financial year, the expenditure on such schemes of the Health Ministry would be initially incurred from the PMSSN and thereafter, from Gross Budgetary Support (GBS). Refer: https://www.insightsonindia.com/2021/03/11/pradhan-mantri-swasthya-suraksha-nidhi/ 95. Which of the following nations shares land border with Afghanistan? 1. Tajikistan 2. Turkmenistan 3. Uzbekistan 4. Iran 5. Pakistan Select the correct answer using the code below: (a) 1, 2, 3 and 5 only (b) 2, 3, 4 and 5 only (c) 1, 2, 4 and 5 only (d) 1, 2, 3, 4 and 5 Ans: (d) Explanation:

• Refer: https://www.insightsonindia.com/2021/03/11/what-does-bidens-peace-plan-mean-for- afghanistan/ 96. Consider the following statements about the National Critical Information Infrastructure Protection Centre (NCIIPC): 1. It is an organisation of the Government of India created under the Information Technology Act. 2. It is designated as the National Nodal Agency in respect of Critical Information Infrastructure Protection. 3. It is a unit of the National Technical Research Organisation (NTRO). Which of the given above statements is/are correct? (a) 1 and 2 only (b) 2 and 3 only (c) 1 and 3 only (d) 1, 2 and 3 Ans: (d) Explanation: • National Critical Information Infrastructure Protection Centre (NCIIPC) is an organisation of the Government of India created under Sec 70A of the Information Technology Act, 2000 (amended 2008), through a gazette notification on 16 January 2014.

Telegram: https://t.me/insightsIAStips 51 Youtube: https://www.youtube.com/channel/UCpoccbCX9GEIwaiIe4HLjwA

Revision Through MCQs (RTM) Compilation (March 2021)

• Based in New Delhi, India, it is designated as the National Nodal Agency in respect of Critical Information Infrastructure Protection. • It is a unit of the National Technical Research Organisation (NTRO). Refer: https://www.insightsonindia.com/2021/03/11/telecom-licensing-conditions-amended/ 97. Which one of the following is the best description of ‘INS Karanj’, that was in the news recently? (a) Amphibious warfare ship (b) Nuclear-powered submarine (c) Torpedo launch and recovery vessel (d) Diesel-electric attack submarine Ans: (d) Explanation: INS Karanj: • It is the third Scorpene class conventional diesel-electric submarine. • It was inducted into Indian Navy recently.

• Refer: facts for prelims: https://www.insightsonindia.com/2021/03/11/insights-daily-current- affairs-pib-summary-11-march-2021/ 98. With reference to India’s Five-Year Plans, which of the following statements is/are correct? 1. The Minimum Needs Programme (MNP) was introduced in the first year of the Fifth Five-Year Plan. 2. In the Eighth Five-Year Plan, the gradual opening of the Indian economy was undertaken to correct the burgeoning deficit and foreign debt. 3. Ninth Five Year Plan was launched with the objective of faster, sustainable and more inclusive growth. Which of the statements given above is/are correct? (a) 1 only (b) 2 and 3 only (c) 1 and 2 only (d) 1, 2 and 3 Ans: (c) Explanation: • S3: The main objective of the Ninth Five-Year Plan was to correct historical inequalities and increase the economic growth in the country. Other aspects which constituted the Ninth Five-Year Plan were: o Population control. o Generating employment by giving priority to agriculture and rural development. o Reduction of poverty. o Ensuring proper availability of food and water for the poor. o Availability of primary health care facilities and other basic necessities. o Primary education to all children in the country. o Empowering the socially disadvantaged classes like Scheduled castes, Scheduled tribes and other backward classes. o Developing self-reliance in terms of agriculture. o Acceleration in the growth rate of the economy with the help of stable prices.

Telegram: https://t.me/insightsIAStips 52 Youtube: https://www.youtube.com/channel/UCpoccbCX9GEIwaiIe4HLjwA

Revision Through MCQs (RTM) Compilation (March 2021)

• Twelfth Five Year Plan (2012-2017) was launched with the objective of faster, sustainable and more inclusive growth. • S2: Modernization of industries was a major highlight of the Eighth Plan. Under this plan, the gradual opening of the Indian economy was undertaken to correct the burgeoning deficit and foreign debt. • S1: The Fifth Five-Year Plan laid stress on employment, poverty alleviation (Garibi Hatao), and justice. The plan also focused on self-reliance in agricultural production and defence. o The Minimum Needs Programme (MNP) was introduced in the first year of the Fifth Five-Year Plan (1974–78). The objective of the programme is to provide certain basic minimum needs and thereby improve the living standards of the people. It is prepared and launched by D.P.Dhar. Refer: https://en.wikipedia.org/wiki/Five-Year_Plans_of_India 99. Consider the following statements regarding Transfer payments: 1. Transfer Payments are payments which are made without any counterpart of services received by the payer. 2. For the purpose of calculating gross domestic product (GDP), government spending does not include transfer payments. Which of the statements given above is/are correct? (a) 1 only (b) 2 only (c) Both 1 and 2 (d) Neither 1 nor 2 Ans: (c) Explanation: • Transfer Payments are payments which are made without any counterpart of services received by the payer. • It include Social Security, Medicare, unemployment insurance, welfare programs, and subsidies. • For the purpose of calculating gross domestic product (GDP), government spending does not include transfer payments, which are the reallocation of money from one party to another rather than expenditure on newly produced goods and services. Refer: https://www.investopedia.com/terms/t/transferpayment.asp 100. Which of the following statements is/are correct regarding the Monetary Policy Committee (MPC)? 1. It is a committee constituted by the Reserve Bank of India and led by the Governor of RBI. 2. It consists of five members and they hold office for a period of four years. 3. It determines the policy interest rate required to achieve the inflation target. Which of the given above statements is/are correct? (a) 1 only (b) 2 and 3 only (c) 1 and 3 only (d) 1, 2 and 3 Ans: (c) Explanation: • The Monetary Policy Committee (MPC) is a committee constituted by the Reserve Bank of India and led by the Governor of RBI. It has been instituted by the Central Government of India under Section 45ZB of the RBI Act that was amended in 1934. • Functions: o The MPC is entrusted with the responsibility of deciding the different policy rates including MSF, Repo Rate, Reverse Repo Rate, and Liquidity Adjustment Facility. • Composition of MPC:

Telegram: https://t.me/insightsIAStips 53 Youtube: https://www.youtube.com/channel/UCpoccbCX9GEIwaiIe4HLjwA

Revision Through MCQs (RTM) Compilation (March 2021)

o The committee will have six members. Of the six members, the government will nominate three. No government official will be nominated to the MPC. o The other three members would be from the RBI with the governor being the ex- officio chairperson. Deputy governor of RBI in charge of the monetary policy will be a member, as also an executive director of the central bank. • Selection and term of members: o Selection: The government nominees to the MPC will be selected by a Search- cum-Selection Committee under Cabinet Secretary with RBI Governor and Economic Affairs Secretary and three experts in the field of economics or banking or finance or monetary policy as its members. o Term: Members of the MPC will be appointed for a period of four years and shall not be eligible for reappointment. • How decisions are made? o Decisions will be taken by majority vote with each member having a vote. o RBI governor’s role: The RBI Governor will chair the committee. The governor, however, will not enjoy a veto power to overrule the other panel members, but will have a casting vote in case of a tie. • What is RBI Monetary Policy? o The term ‘Monetary Policy’ is the Reserve Bank of India’s policy pertaining to the deployment of monetary resources under its control for the purpose of achieving GDP growth and lowering the inflation rate. o The Reserve Bank of India Act 1934 empowers the RBI to make the monetary policy. Refer: https://www.insightsonindia.com/2020/04/21/what-monetary-policy-committee-mpc/

Telegram: https://t.me/insightsIAStips 54 Youtube: https://www.youtube.com/channel/UCpoccbCX9GEIwaiIe4HLjwA

Revision Through MCQs (RTM) Compilation (March 2021)

RTM- REVISION THROUGH MCQS – 12th-Mar-2021

101. With reference to space hurricanes, consider the following statements: 1. Space hurricanes are caused by plasma unleashed from the sun as solar wind. 2. The first scientific observation of the effects of a space hurricanes occurred early in the 19th century. 3. In space, astronomers have spotted hurricanes on Mars, and Saturn, and Jupiter. Which of the given above statements is/are correct? (a) 1 and 2 only (b) 2 and 3 only (c) 1 and 3 only (d) 1, 2 and 3 Ans: (c) Explanation: • S1: Space hurricanes are caused by plasma unleashed from the sun as solar wind. These charged particle clouds travel through space and fuel magnetic storms as they interact with magnetic fields. • S2: The phenomenon was discovered by a team of researchers from Shandong University in China, whom had observed the storm over the Arctic region on 20 August 2014, before identifying its nature in 2021. • S3: In space, astronomers have spotted hurricanes on Mars, and Saturn, and Jupiter, which are similar to terrestrial hurricanes in the low atmosphere. More>> Refer: https://www.insightsonindia.com/2021/03/12/what-is-space-hurricane/ 102. With reference to ‘Food Security & Public Distribution’ in India, consider the following statements: 1. The Department of Food and Public Distribution is responsible for management of the food economy in India. 2. Public Distribution System (PDS) is operated under the joint responsibility of the Central and the State/UT Governments. 3. Under the Targeted Public Distribution System (TPDS), the end retail price was fixed by the Central Government after taking into account margin for wholesalers/ retailers, transportation charges and levies local taxes. Which of the given above statements is/are correct? (a) 1 and 2 only (b) 2 and 3 only (c) 1 and 3 only (d) 1, 2 and 3 Ans: (a) Explanation: • S1: The Department of Food and Public Distribution is responsible for management of the food economy of the nation. It undertakes various activities, such as procurement of food items, their storage, movement and delivery to the distributing agencies. More>> • S2: The Public Distribution System (PDS) evolved as a system of management of scarcity through distribution of foodgrains at affordable prices……… PDS is operated under the joint responsibility of the Central and the State/UT Governments. More>> • S3: In June, 1997, the Government of India launched the Targeted Public Distribution System (TPDS) with focus on the poor. Under the PDS, States were required to formulate and implement foolproof arrangements for identification of the poor for delivery of foodgrains and for its distribution in a transparent and accountable manner at the FPS level.,……….. Under the TPDS, the end retail price was fixed by the States/UTs after taking into account margin for wholesalers/ retailers, transportation charges, levies local taxes etc. More>>

Telegram: https://t.me/insightsIAStips 55 Youtube: https://www.youtube.com/channel/UCpoccbCX9GEIwaiIe4HLjwA

Revision Through MCQs (RTM) Compilation (March 2021)

Refer: https://www.insightsonindia.com/2021/03/12/one-nation-one-ration-card-system/ 103. Fifth annual democracy report, titled 'Autocratisation goes viral', has been released by which of the following? (a) Economist Intelligence Unit (b) Varieties of Democracy Institute (c) Freedom House (d) World Economic Forum Ans: (b) Explanation: • Fifth annual democracy report, titled ‘Autocratisation goes viral’, has been released by Sweden’s organisation Varieties of Democracy (V-Dem) Institute. • The report summarises the state of democracies of the world against the backdrop of developments that have taken place over the past decade. Refer: https://www.insightsonindia.com/2021/03/12/annual-democracy-report/ 104. The Reserve Bank of India’s ‘Prompt Corrective Action’ (PCA) Framework is applicable to: 1. Private Sector Banks 2. Co-operative Banks 3. Non-banking financial companies Select the correct answer using the code below: (a) 2 and 3 only (b) 1 only (c) 1 and 3 only (d) 1, 2 and 3 Ans: (b) Explanation: • The Reserve Bank of India put several banks under prompt corrective action or PCA for not maintaining a desirable level of capital which restricts their business activity. Besides capital, PCA is also triggered if bad loans are higher than a minimum threshold and return on assets is lower than the threshold. • The PCA framework is applicable only to commercial banks and not extended to cooperative banks and Non-banking financial companies (NBFCs) Refer: https://www.insightsonindia.com/2021/03/12/prompt-corrective-action-framework/ 105. Consider the following statements about Prompt Corrective Action (PCA) framework: 1. Reserve Bank of India (RBI) issues a Prompt Corrective Action (PCA) framework to maintain sound financial health of banks. 2. It is intended to encourage banks to eschew certain riskier activities and focus on conserving capital. 3. The framework is intended to constrain the performance of normal operations of the banks for the general public. Which of the above statements is/are correct? (a) 1 and 3 only (b) 2 and 3 only (c) 1 and 2 only (d) 1, 2 and 3 Ans: (c) Explanation: • Reserve Bank of India (RBI) has issued a Prompt Corrective Action (PCA) framework to maintain sound financial health of banks. It facilitates banks in breach of risk thresholds for identified areas of monitoring, viz., capital, asset quality (which is tracked in terms of the net Non-Performing Assets ratio) and profitability, to take corrective measures in a timely manner, in order to restore their financial health. Thus, it is intended to encourage banks to eschew certain riskier activities, improve operational efficiency and focus on

Telegram: https://t.me/insightsIAStips 56 Youtube: https://www.youtube.com/channel/UCpoccbCX9GEIwaiIe4HLjwA

Revision Through MCQs (RTM) Compilation (March 2021)

conserving capital to strengthen them. The framework is not intended to constrain the performance of normal operations of the banks for the general public. • What will a bank do if PCA is triggered? o Banks are not allowed to renew or access costly deposits or take steps to increase their fee-based income. o Banks will also have to launch a special drive to reduce the stock of NPAs and contain generation of fresh NPAs. o They will also not be allowed to enter into new lines of business. RBI will also impose restrictions on the bank on borrowings from interbank market. Refer: https://www.insightsonindia.com/2021/03/12/prompt-corrective-action-framework/ 106. Consider the following statements: 1. In medicine, the Active Pharmaceutical Ingredients (API) produces the intended effects to cure the disease. 2. Fixed-dose combination drugs use multiple APIs, while single-dose drugs use just one API. 3. During the early 90s, India was self-reliant in manufacturing APIs. Which of the given above statements is/are correct? (a) 1 and 2 only (b) 2 and 3 only (c) 1 and 3 only (d) 1, 2 and 3 Ans: (d) Explanation: What is an API? • Every medicine is made up of two main ingredients — the chemically active APIs and chemically inactive, excipients, which is a substance that delivers the effect of APIs to one’s system. • API is a chemical compound that is the most important raw material to produce a finished medicine. • In medicine, API produces the intended effects to cure the disease. For instance, Paracetamol is the API for Crocin and it is the API paracetamol that gives relief from body ache and fever. • Fixed-dose combination drugs use multiple APIs, while single-dose drugs like Crocin use just one API. • How India lost its API market to China? o During the early 90s, India was self-reliant in manufacturing APIs. o However, with the rise of China as a producer of API, it captured the Indian market with cheaper products and it eventually led to high economies of scale for China. o China created a low-cost API manufacturing industry. The industry was backed by the low cost of capital followed by aggressive government funding models, tax incentives. o Their cost of operation is one-fourth of India’s cost. Even the cost of finance in China is 6-7 per cent against India’s 13-14 per cent. o So, due to low-profit margins and non-lucrative industry, Indian pharma companies over the years stopped manufacturing APIs. Refer: https://www.insightsonindia.com/2021/03/12/pli-centres-nod-for-33-api-applications/ 107. Consider the following statements about the Lingaraja Temple: 1. It is the largest temple in Bhubaneswar. 2. It is well-known because it has the second earliest linga associated with Shiva discovered so far. 3. It is built in red stone and is a classic example of Kalinga style of architecture. Which of the given above statements is/are correct? (a) 1 and 2 only (b) 2 and 3 only

Telegram: https://t.me/insightsIAStips 57 Youtube: https://www.youtube.com/channel/UCpoccbCX9GEIwaiIe4HLjwA

Revision Through MCQs (RTM) Compilation (March 2021)

(c) 1 and 3 only (d) 1, 2 and 3 Ans: (c) Explanation: • S1: The Lingaraja temple is the largest temple in Bhubaneswar. James Fergusson (1808– 86), a noted critic and historian rated the temple as "one of the finest examples of purely Hindu temple in India". • S2: The Gudimallam Lingam is an ancient linga in the Parasurameswara Swamy Temple of Gudimallam. This is perhaps the second earliest linga associated with Shiva discovered so far, and it has been dated to the 2nd/1st century BC, or the 3rd century BC, or much later o This is "the only sculpture of any importance" to survive from ancient South India before sculpture made under the Pallava dynasty from the 7th century AD onwards, and "its mysteriousness lies in the total absence so far of any object in an even remotely similar manner within many hundreds of miles, and indeed anywhere in South India". o If assigned an early date, the figure on the linga is "one of the earliest surviving and unequivocal images of the god Shiva". • S3: Lingaraja Temple: o It is a temple dedicated to Shiva. o Built by king Jajati Keshari of Soma Vansh. o It is built in red stone and is a classic example of Kalinga style of architecture. o Located to the north of the temple is Bindusagar Lake. o The temple has images of Vishnu, possibly because of the rising prominence of Jagannath sect emanating from the Ganga rulers who built the Jagannath Temple in Puri in the 12th century. Refer: Facts for Prelims: https://www.insightsonindia.com/2021/03/12/insights-daily-current- affairs-pib-summary-12-march-2021/ 108. With reference to minimum support price (MSP) for farmers in India, consider the following statements: 1. It was the first time in the year 1975 that the Minimum Support Price was introduced by the Centre. 2. The MSP is fixed twice a year on the recommendations of the Commission for Agricultural Costs and Prices (CACP). 3. At present, the Centre provides the MSP for 23 crops. Which of the given above statements is/are correct? (a) 1 and 2 only (b) 2 and 3 only (c) 1 and 3 only (d) 1, 2 and 3 Ans: (b) Explanation: • Minimum support price (MSP) is a “minimum price” for any crop that the government considers as remunerative for farmers and hence deserving of “support”. It is also the price that government agencies pay whenever they procure the particular crop. • At the time of Independence, India was staring at a major deficit in terms of cereal production. After the struggling first decade, India decided to go for extensive agricultural reforms. It was the first time in the year 1966-67 that the Minimum Support Price was introduced by the Centre. At Rs 54 per quintal, the MSP for wheat was fixed for the first time.

Telegram: https://t.me/insightsIAStips 58 Youtube: https://www.youtube.com/channel/UCpoccbCX9GEIwaiIe4HLjwA

Revision Through MCQs (RTM) Compilation (March 2021)

• The MSP is fixed twice a year on the recommendations of the Commission for Agricultural Costs and Prices (CACP), which is a statutory body and submits separate reports recommending prices for kharif and rabi seasons. • The CACP submits its recommendations to the government in the form of Price Policy Reports every year, separately for five groups of commodities namely kharif crops, rabi crops, sugarcane, raw jute and copra. • After considering the report and views of the state governments and also keeping in view the overall demand and supply situation in the country, the central government takes the final decision. • Currently, MSP for 23 crops is recommended by CACP, which comprise seven cereals (paddy, wheat, maize, sorghum, pearl millet, barley and ragi), five pulses (gram, tur, moong, urad, lentil), seven oilseeds (groundnut, rapeseed-mustard, soyabean, sesamum, sunflower, safflower, nigerseed), and four commercial crops (copra, sugarcane, cotton and raw jute). • Is MSP legal? o Short answer – No. While Centre has been providing the MSP to the wheat and paddy farmers since mid-60s to tide over the food crisis, the fact remains that the MSP doesn’t have any legal stature. Refer: https://indianexpress.com/article/explained/minimum-support-price-msp-farmers-explained- 6706253/ 109. The Fifteenth Finance Commission (FC) has considered which of the following parameters to arrive at the state’s share in the divisible pool of taxes: 1. Forest cover 2. Area of the state 3. 2011 population census 4. Demographic performance Select the correct answer code: (a) 1, 2 and 3 only (b) 1, 3 and 4 only (c) 2, 3 and 4 only (d) 1, 2 and 4 only Ans: (c) Explanation: • The Finance Commission uses certain criteria when deciding the devolution to states. For example, income distance criterion has been used by the 14th and 15th Finance Commissions. Under this criterion, states with lower per capita income would be given a higher share to maintain equity among states. Another example is Demographic Performance criterion which has been introduced by the 15th Finance

Telegram: https://t.me/insightsIAStips 59 Youtube: https://www.youtube.com/channel/UCpoccbCX9GEIwaiIe4HLjwA

Revision Through MCQs (RTM) Compilation (March 2021)

Commission. The Demographic Performance criterion is to reward efforts made by states in controlling their population. • The 15th Finance Commission used the following criteria while determining the share of states: (i) 45% for the income distance, (ii) 15% for the population in 2011, (iii) 15% for the area, (iv) 10% for forest and ecology (forest cover used in 14th FC), (v) 12.5% for demographic performance, and (vi) 2.5% for tax effort.

Refer: https://fincomindia.nic.in/ShowContent.aspx?uid1=11&uid2=6&uid3=0&uid4=0 110. Consider the following statements about Pradhan Mantri Kisan Samman Nidhi (PM-KISAN) 1. PM Kisan is a Central Sector scheme with 100% funding from Government of India. 2. The entire responsibility of identification of beneficiaries rests with the State / UT Governments. 3. Under the scheme an income support of ₹ 6,000 is provided three times to all landholding farmer’s families. Which of the following statements is/are correct? (a) 1 and 2 only (b) 2 only (c) 1 and 3 only (d) 1, 2 and 3 Ans: (a) Explanation: About Pradhan Mantri Kisan Samman Nidhi: • It is implemented as a central sector scheme by the Government of India. • This scheme was introduced to augment the source of income of many small and marginal farmers. • Under the Scheme an amount of Rs.6000/-per year is transferred directly into the bank accounts of the farmers, subjectto certain exclusion criteria relating to higher income status. • The entire responsibility of identification of beneficiaries rests with the State / UT Governments. • Ambit: o The Scheme initially provided income support to all Small and Marginal Farmers’ families across the country, holding cultivable land upto 2 hectares. Its ambit was later expanded w.e.f. 01.06.2019 to cover all farmer families in the country irrespective of the size of their land holdings. • Exceptions: o Affluent farmers have been excluded from the scheme such as Income Tax payers in last assessment year, professionals like Doctors, Engineers, Lawyers, Chartered Accountants etc and pensioners pensioners drawing at least Rs.10,000/-per month (excluding MTS/Class IV/Group D employees). Refer: https://pmkisan.gov.in/#About

Telegram: https://t.me/insightsIAStips 60 Youtube: https://www.youtube.com/channel/UCpoccbCX9GEIwaiIe4HLjwA

Revision Through MCQs (RTM) Compilation (March 2021)

RTM- REVISION THROUGH MCQS – 13th-Mar-2021

111. With reference to civil disobedience movement, consider the following statements: 1. Gandhiji asked followers not only refuse to cooperate but also to break colonial laws. 2. Like Non-cooperation movement, Gandhiji also called off Civil Disobedience movement midway. Which of the above statements is/are correct? (a) 1 Only (b) 2 Only (c) Both 1 and 2 (d) Neither 1 nor 2 Ans: (c) Explanation: • S1: There were certain aspects in which the Civil Disobedience Movement differed from the Non-Cooperation Movement. o The stated objective this time was complete independence and not just remedying two specific wrongs and a vaguely-worded swaraj. o The methods involved violation of law from the very beginning and not just non-cooperation with foreign rule. o There was a decline in forms of protests involving the intelligentsia, such as lawyers giving up practice, students giving up government schools to join national schools and colleges. • S2: On January 25, 1931, Gandhi and all other members of the Congress Working Committee (CWC) were released unconditionally. The CWC authorised Gandhi to initiate discussions with the viceroy. As a result of these discussions, a pact was signed between the viceroy, representing the British Indian Government, and Gandhi, representing the Indian people, in Delhi on February 14, 1931. o Gandhi on behalf of the Congress agreed— ▪ to suspend the civil disobedience movement, and ▪ to participate in the next Round Table Conference on the constitutional question around the three lynch-pins of federation, Indian responsibility, and reservations and safeguards that may be necessary in India’s interests (covering such areas as defence, external affairs, position of minorities, financial credit of India and discharge of other obligations). Refer: https://www.insightsonindia.com/2021/03/13/padyatra-to-commemorate-salt-march/ 112. Consider the following statements: 1. The Civil Disobedience Movement strengthened relationship between peasants – both poor and rich – with the Congress 2. The industrial working classes participated in the Civil Disobedience Movement in large numbers Which of the above statements is/are correct? (a) 1 Only (b) 2 Only (c) Both 1 and 2 (d) Neither 1 nor 2 Ans: (a) Explanation: • There were certain aspects in which the Civil Disobedience Movement differed from the Non-Cooperation Movement. o There was a decline in forms of protests involving the intelligentsia, such as lawyers giving up practice, students giving up government schools to join national schools and colleges.

Telegram: https://t.me/insightsIAStips 61 Youtube: https://www.youtube.com/channel/UCpoccbCX9GEIwaiIe4HLjwA

Revision Through MCQs (RTM) Compilation (March 2021)

o Muslim participation was nowhere near that in the Non-Cooperation Movement level. o No major labour upsurge coincided with the movement. o The massive participation of peasants and business groups compensated for decline of other features. o The number of those imprisoned was about three times more this time. o The Congress was organisationally stronger. Refer: https://www.insightsonindia.com/2021/03/13/padyatra-to-commemorate-salt-march/ 113. With reference to Medical Termination of Pregnancy (Amendment) Bill, 2020, Consider the following statements: 1. It seeks to extend the upper limit for permitting abortions from 24 weeks to 26 under special circumstances. 2. The Bill proposes requirement of opinion of one registered medical practitioner (RMP) for termination of pregnancy up to 20 weeks of gestation. Which of the given above statements is/are correct? (a) 1 only (b) 2 only (c) Both 1 and 2 (d) Neither 1 nor 2 Ans: (b) Explanation: • Medical Termination of Pregnancy (Amendment) Bill, 2020 is passed by the Lok Sabha and it will be discussed in the Rajya Sabha. • Key changes: o It seeks to extend the upper limit for permitting abortions from 20 weeks to 24 under special circumstances. o The “special categories of women” include rape survivors, victims of incest, the differently abled and minors. o The Bill proposes requirement of opinion of one registered medical practitioner (RMP) for termination of pregnancy up to 20 weeks of gestation. o It also provides for the requirement of opinion of two RMPs for termination of pregnancy of 20 to 24 weeks. o Constitution of a Medical Board: Every state government is required to constitute a Medical Board. These Medical Boards will consist of the following members: (i) a gynaecologist, (ii) a paediatrician, (iii) a radiologist or sonologist, and (iv) any other number of members, as may be notified by the state government. o Under the Bill, if any pregnancy occurs as a result of failure of any device or method used by a woman or her partner to limit the number of children, such an unwanted pregnancy may constitute a grave injury to the mental health of the pregnant woman. Refer: https://www.insightsonindia.com/2021/03/13/plea-to-constitute-district-medical-boards/

114. Consider the following statements: 1. The State Election Commission is a Constitutional Authority. 2. The powers and functions of the State Election Commission are identical to those vested in the Election Commission of India. 3. State Election Commissioner cannot be removed by the Governor, though appointed by him. Which of the given above statements is/are correct? (a) 1 and 3 only (b) 1 only (c) 2 and 3 only (d) 1, 2 and 3

Telegram: https://t.me/insightsIAStips 62 Youtube: https://www.youtube.com/channel/UCpoccbCX9GEIwaiIe4HLjwA

Revision Through MCQs (RTM) Compilation (March 2021)

Ans: (d) Explanation: • State election commissioner is appointed by The Governor. • The powers and functions of the State Election Commission under article 243K AND 243ZA of the Constitution Of India are identical to those vested in the Election Commission Of India constituted under article 324 of the Constitution Of India in their respective domains. • His conditions of service and tenure of office shall also be determined by the governor. • He shall not be removed from the office except in the manner and on the grounds prescribed for the removal of the judge of the state high court. • A judge of a high court can be removed from his office by the president on the recommendation of the parliament. This means that a state election commissioner cannot be removed by the governor, though appointed by him. • The ECI and SECs have a similar mandate; do they also have similar powers? o The provisions of Article 243K of the Constitution, which provides for setting up of SECs, are almost identical to those of Article 324 related to the EC. In other words, the SECs enjoy the same status as the EC. o In 2006, the Supreme Court emphasised the two constitutional authorities enjoy the same powers. o In Kishan Singh Tomar vs Municipal Corporation of the City of Ahmedabad, the Supreme Court directed that state governments should abide by orders of the SECs during the conduct of the panchayat and municipal elections, just like they follow the instructions of the EC during Assembly and Parliament polls. Refer: https://www.insightsonindia.com/2021/03/13/state-election-commissioners/ 115. Consider the following statements about Election Commission 1. The Constitution has prescribed the qualifications of the members of the Election Commission. 2. The Constitution has specified the term of the members of the Election Commission as 6 years or until they attain the age of 65 years. 3. The Constitution has debarred the retiring election commissioners from further appointment by the government. Which of the above statements is/are incorrect? (a) 1 and 2 only (b) 1 and 3 only (c) 2 and 3 only (d) 1, 2 and 3 Ans: (d) Explanation: here the directive word is incorrect!! • Though the constitution has sought to safeguard and ensure the independence and impartiality of the Election Commission, some flaws can be noted, viz., o The Constitution has not prescribed the qualifications (legal, educational, administrative or judicial) of the members of the Election Commission. o The Constitution has not specified the term of the members of the Election Commission. o The Constitution has not debarred the retiring election commissioners from any further appointment by the government Refer: https://www.insightsonindia.com/2021/03/13/state-election-commissioners/ 116. Consider the following statements: 1. Right to Information is a part of fundamental rights under Article 19(1) of the Constitution of India. 2. RTI Act, 2005 replaced the former Freedom of Information Act, 2002. 3. Currently no parties are under the RTI Act and a case has been filed for bringing all political parties under it.

Telegram: https://t.me/insightsIAStips 63 Youtube: https://www.youtube.com/channel/UCpoccbCX9GEIwaiIe4HLjwA

Revision Through MCQs (RTM) Compilation (March 2021)

Which of the given above statements is/are correct? (a) 1 and 2 only (b) 1 and 3 only (c) 2 and 3 only (d) 1, 2 and 3 Ans: (d) Explanation: About the RTI Act, 2005: • It sets out the rules and procedures regarding citizens’ right to information. • It replaced the former Freedom of Information Act, 2002. • This act was enacted in order to consolidate the fundamental right in the Indian constitution ‘freedom of speech’. Since RTI is implicit in the Right to Freedom of Speech and Expression under Article 19 of the Indian Constitution, it is an implied fundamental right. • Key Provisions: o Section 4 of the RTI Act requires suo motu disclosure of information by each public authority. o Section 8 (1) mentions exemptions against furnishing information under RTI Act. o Section 8 (2) provides for disclosure of information exempted under Official Secrets Act, 1923 if larger public interest is served. • Information Commissioners and PIOs: o The Act also provides for appointment of Information Commissioners at Central and State level. o Public authorities have designated some of its officers as Public Information Officer. They are responsible to give information to a person who seeks information under the RTI Act. • Time period: o In normal course, information to an applicant is to be supplied within 30 days from the receipt of application by the public authority. o If information sought concerns the life or liberty of a person, it shall be supplied within 48 hours. o In case the application is sent through the Assistant Public Information Officer or it is sent to a wrong public authority, five days shall be added to the period of thirty days or 48 hours, as the case may be. • Applicability of RTI to: • Private bodies: o Private bodies are not within the Act’s ambit directly. o In a decision of Sarbjit roy vs Delhi Electricity Regulatory Commission, the Central Information Commission also reaffirmed that privatised public utility companies fall within the purview of RTI. • Political parties: o The Central Information Commission (CIC) had held that the political parties are public authorities and are answerable to citizens under the RTI Act. o But in August 2013 the government introduced a Right To Information (Amendment) Bill which would remove political parties from the scope of the law. o Currently no parties are under the RTI Act and a case has been filed for bringing all political parties under it. • Chief Justice of India: o Supreme Court of India on 13 November 2019, upheld the decision of Delhi High Court bringing the office of Chief Justice of India under the purview of Right to Information (RTI) Act. Refer: https://www.insightsonindia.com/2021/03/13/rajasthan-information-commission-penalises- five-officials-for-negligence/ 117. Consider the following statements: 1. Benzene is formed from both natural processes and human activities.

Telegram: https://t.me/insightsIAStips 64 Youtube: https://www.youtube.com/channel/UCpoccbCX9GEIwaiIe4HLjwA

Revision Through MCQs (RTM) Compilation (March 2021)

2. As it contains only carbon and hydrogen atoms, benzene is classed as a hydrocarbon. Which of the given above statements is/are correct? (a) 1 only (b) 2 only (c) Both 1 and 2 (d) Neither 1 nor 2 Ans: (c) Explanation: • The benzene molecule is composed of six carbon atoms joined in a planar ring with one hydrogen atom attached to each. As it contains only carbon and hydrogen atoms, benzene is classed as a hydrocarbon. • Benzene is formed from both natural processes and human activities. Natural sources of benzene include volcanoes and forest fires. Benzene is also a natural part of crude oil, gasoline, and cigarette smoke. Refer: https://www.insightsonindia.com/2021/03/13/bring-down-benzene-emission-at-fuel-outlets- says-panel/ 118. Consider the following statements about Petroleum and Explosives Safety Organisation (PESO): 1. It functions under Ministry of Petroleum and Natural Gas. 2. It was formed to control and administer the usage of explosives, petrol stations in India. 3. The Department is headed by Chief Controller of Explosives and is headquartered at Nagpur in the State of Maharashtra in India Which of the given above statements is/are correct? (a) 1 and 2 only (b) 2 and 3 only (c) 1 and 3 only (d) 1, 2 and 3 Ans: (b) Explanation: Petroleum & Explosives Safety Organization: • It is a department under Department for the Promotion of Industry and Internal Trade under Ministry of Commerce and Industry. • It is a regulatory authority with autonomous status. • It was established during the British India in 1890s as Department of Explosives and later expanded to various other activities. • PESO is entrusted with the responsibilities under the Explosives Act, 1884; Petroleum Act, 1934; Inflammable Substances Act, 1952, Environment (Protection Act), 1986. Refer: https://www.insightsonindia.com/2021/03/13/bring-down-benzene-emission-at-fuel-outlets- says-panel/ 119. Consider the following statements about National Green Tribunal (NGT) 1. It is a statutory body established by a Government of India. 2. The Chairperson of the NGT is appointed by the Central Government of India in accordance with the Chief Justice of India. 3. It has not been vested with powers to hear any matter relating to the Wildlife (Protection) Act, 1972. Which of the given above statements is/are correct? (a) 1 and 3 only (b) 2 Only (c) 2 and 3 only (d) 1, 2 and 3 Ans: (d) Explanation:

Telegram: https://t.me/insightsIAStips 65 Youtube: https://www.youtube.com/channel/UCpoccbCX9GEIwaiIe4HLjwA

Revision Through MCQs (RTM) Compilation (March 2021)

• S1: National Green Tribunal is a statutory body established by a Government Notification using the powers of Section 3 of the NGT Act 2010. • S2: The Chairperson of the National Green Tribunal (NGT) is appointed by the Central Government of India in accordance with the Chief Justice of India. • S3: The NGT has not been vested with powers to hear any matter relating to the Wildlife (Protection) Act, 1972, the Indian Forest Act, 1927 and various laws enacted by States relating to forests, tree preservation etc. Therefore, specific and substantial issues related to these laws cannot be raised before the NGT. Refer: https://www.insightsonindia.com/2021/03/13/bring-down-benzene-emission-at-fuel-outlets- says-panel/ 120. In rural road construction, the use of which of the following is preferred for ensuring environmental sustainability or to reduce carbon footprint? 1. Copper slag 2. Cold mix asphalt technology 3. Geotextiles 4. Hot mix asphalt technology 5. Portland cement Select the correct answer using the given code below (a) 1, 2 and 3 only (b) 2, 3 and 4 only (c) 4 and 5 only (d) 1 and 5 only Ans: (a) Explanation: • S5 is wrong since Portland cement is a major CO2 emitter. Same goes for asphalt mix. Hot Mix Asphalt (HMA) is a combination of approximately 95% stone, sand, or gravel bound together by asphalt cement, a product of crude oil. So, S4 is incorrect as well. • S1, S2 and S3 are correct. Refer: UPSC CSE 2020

RTM- REVISION THROUGH MCQS – 15th-Mar-2021

121. Consider the following statements about the All India Council for Technical Education (AICTE): 1. It was set up in 1945 as a national-level apex advisory body with the vision to promote development of the education system in India. 2. In 1987, it was given a statutory status by an Act of Parliament. 3. It is responsible for proper planning and coordinated development of the technical education and management education system in India. Which of the given above statements is/are correct? (a) 1 and 2 only (b) 2 and 3 only (c) 1 and 3 only (d) 1, 2 and 3 Ans: (d) Explanation: • The All India Council for Technical Education (AICTE) is a statutory body, and a national- level council for technical education, under the Department of Higher Education. • Established in November 1945 first as an advisory body and later on in 1987 given statutory status by an Act of Parliament, AICTE is responsible for proper planning and coordinated development of the technical education and management education system in India.

Telegram: https://t.me/insightsIAStips 66 Youtube: https://www.youtube.com/channel/UCpoccbCX9GEIwaiIe4HLjwA

Revision Through MCQs (RTM) Compilation (March 2021)

Refer: https://www.insightsonindia.com/2021/03/15/maths-physics-chemistry-not-compulsory-for- engineering-admissions-aicte/ 122. The ‘World Press Freedom Index’ is published annually by: (a) International Federation of Journalists (b) International Freedom of Expression Exchange (c) World Justice Project (d) Reporters Without Borders Ans: (d) Explanation: About Press Freedom Index: • Paris-based Reporters Sans Frontieres (RSF), or Reporters Without Borders, a non-profit organisation that works to document attacks on journalists around the world, ranked India 142 out of 180 countries in its annual Press Freedom Index report published on April 22, 2020. • The parameters that are evaluated for these rankings, include pluralism, media independence, environment and self-censorship, legal framework, transparency among others. Refer: https://www.insightsonindia.com/2021/03/15/index-monitoring-cell/ 123. The ‘Index Monitoring Cell’ was in news recently, is set up by which of the following ministry? (a) Ministry of Statistics and Programme Implementation (b) Ministry of Finance (c) Ministry of Information and Broadcasting (d) Ministry of Coal Ans: (c) Explanation: About the Index Monitoring Cell: • Set up by the Information and Broadcasting Ministry in 2020. • Its task is to improve India’s ranking in the World Press Freedom Index and to evolve an objective yardstick to gauge media freedom. • The group will also put in place a mechanism for States to come up with their own rankings of press freedom. • Composition of the IMC: o It will include the director-general of the Press Information Bureau, officials from the Registrar of newspapers of India, Bureau of outreach and communication, and from the press facilitation unit, apart from the secretary of the Press Council of India and Niti Aayog. Refer: https://www.insightsonindia.com/2021/03/15/index-monitoring-cell/ 124. Which of the following nation is not a member of ‘Quadrilateral Security Dialogue’? (a) India (b) South Korea (c) Australia (d) Japan Ans: (b) Explanation: What is Quad grouping? • The quadrilateral security dialogue includes Japan, India, United States and Australia. • All four nations find a common ground of being the democratic nations and common interests of unhindered maritime trade and security. • Genesis: o The grouping traces its genesis to 2004 when the four countries came together to coordinate relief operations in the aftermath of the tsunami. o It then met for the first time in 2007 on the sidelines of the Association of South East Asian Nations (ASEAN) summit. o The intention was to enhance maritime cooperation between the four nations. Refer: https://www.insightsonindia.com/2021/03/15/quad-leaders-for-open-free-indo-pacific/

Telegram: https://t.me/insightsIAStips 67 Youtube: https://www.youtube.com/channel/UCpoccbCX9GEIwaiIe4HLjwA

Revision Through MCQs (RTM) Compilation (March 2021)

125. Consider the following statements about the Commission for Air Quality Management (CAQM): 1. It was established in 2020 under the Environment Protection Act, 1986. 2. The Minister for Environment & Forests is the Chairperson of the Authority, and the Minister of State in the Ministry of Environment & Forests is its Vice-Chairman. 3. It will have the powers to issue directions to all state governments on issues pertaining to air pollution. Which of the given above statements is/are not correct? (a) 1 and 2 only (b) 3 only (c) 2 and 3 only (d) 1, 2 and 3 Ans: (d) Explanation: About the Commission for Air Quality Management (CAQM): • S1: The Commission for Air Quality Management was formed by an ordinance in October 2020, the “Commission for Air Quality Management (CAQM) in National Capital Region and Adjoining Areas Ordinance 2020”. • S2: Composition: o Chairperson: To be chaired by a government official of the rank of Secretary or Chief Secretary. o The Commission will be a statutory authority. o The Commission will supersede bodies such as the central and state pollution control boards of Delhi, Punjab, Haryana, UP and Rajasthan. • S3: It will have the powers to issue directions to these state governments (above four) on issues pertaining to air pollution. • Jurisdiction: o Exclusive jurisdiction over the NCR, including areas in Haryana, Punjab, Uttar Pradesh and Rajasthan, in matters of air pollution, and will be working along with CPCB and ISRO, apart from the respective state governments. Refer: https://www.insightsonindia.com/2021/03/15/commission-for-air-quality-management- caqm-2/ 126. Which of one the following includes a hymn called the ‘Nadistuti Sukta’, which sings praises of the Saraswati as being “perfect mother, unsurpassed river, supreme goddess”? (a) Rig Veda (b) Sama Veda (c) Yajur Veda (d) Atharva Veda Ans: (a) Explanation: About Saraswati: • The river, which had originated from Kapal tirith in the Himalayas in the west of Kailash, was flowing southward to Mansarovar and then taking a turn towards west. • The river flowed through Haryana, Rajasthan and North Gujarat. It also flowed through Pakistan before meeting Western Sea through Rann of Kutch and was approximately 4,000 km in length. • The river had two branches: western and eastern. The Himalayan-born Satluj “of the PAST”, which flowed through the channels of present-day Ghaggar-Patialiwali rivulets, represents the western branch of the ancient river. • On the other hand, Markanda and Sarsuti represented the western branch of Saraswati, known as Tons-Yamuna. • The confluence of the branches was near Shatrana, 25 km south of Patiala. And suddenly, it flows crossing the dessert (Rann of Kutch) and meet gulf of western sea. • Historical evidence: o The Sarasvati River is one of the main Rigvedic rivers mentioned in the scripture Rig Veda and later Vedic and post-Vedic texts.

Telegram: https://t.me/insightsIAStips 68 Youtube: https://www.youtube.com/channel/UCpoccbCX9GEIwaiIe4HLjwA

Revision Through MCQs (RTM) Compilation (March 2021)

o Book 6 of the Rig Veda includes a hymn called the ‘Nadistuti Sukta’, which sings praises of the Saraswati as being “perfect mother, unsurpassed river, supreme goddess”. o For 2000 years, between 6000 and 4000 B.C., the Saraswati flowed as a great river. Refer: https://www.insightsonindia.com/2021/03/15/panel-on-mythical-sarasvati-river/ 127. Consider the following statements about Jal Jeevan Mission: 1. It is a project for safe drinking water for every village and city household in India. 2. The project Contractor is to take care of the maintenance of the entire water network at no additional cost to the Government of India. Which of the given above statements is/are correct? (a) 1 only (b) 2 only (c) Both 1 and 2 (d) Neither 1 nor 2 Ans: (d) Explanation: • Jal Jeevan Mission, is envisioned to provide safe and adequate drinking water through individual household tap connections by 2024 to all households in rural India. • Vision o Every rural household has drinking water supply in adequate quantity of prescribed quality on regular and long-term basis at affordable service delivery charges leading to improvement in living standards of rural communities. • Mission o Jal Jeevan Mission is to assist, empower and facilitate: o States/ UTs in planning of participatory rural water supply strategy for ensuring potable drinking water security on long-term basis to every rural household and public institution, viz. GP building, School, Anganwadi centre, Health centre, wellness centres, etc. o States/ UTs for creation of water supply infrastructure so that every rural household has Functional Tap Connection (FHTC) by 2024 and water in adequate quantity of prescribed quality is made available on regular basis. o States/ UTs to plan for their drinking water security o GPs/ rural communities to plan, implement, manage, own, operate and maintain their own in-village water supply systems o States/ UTs to develop robust institutions having focus on service delivery and financial sustainability of the sector by promoting utility approach o Capacity building of the stakeholders and create awareness in community on significance of water for improvement in quality of life o In making provision and mobilization of financial assistance to States/ UTs for implementation of the mission • Objectives: The broad objectives of the Mission are: o To provide FHTC to every rural household. o To prioritize provision of FHTCs in qualityaffected areas, villages in drought prone and desert areas, Sansad Adarsh Gram Yojana (SAGY) villages, etc. o To provide functional tap connection to Schools, Anganwadi centres, GP buildings, Health centres, wellness centres and community buildings o To monitor functionality of tap connections. o To promote and ensure voluntary ownership among local community by way of contribution in cash, kind and/ or labour and voluntary labour (shramdaan) o To assist in ensuring sustainability of water supply system, i.e. water source, water supply infrastructure, and funds for regular O&M o To empower and develop human resource in the sector such that the demands of construction, plumbing, electrical, water quality management, water

Telegram: https://t.me/insightsIAStips 69 Youtube: https://www.youtube.com/channel/UCpoccbCX9GEIwaiIe4HLjwA

Revision Through MCQs (RTM) Compilation (March 2021)

treatment, catchment protection, O&M, etc. are taken care of in short and long term o To bring awareness on various aspects and significance of safe drinking water and involvement of stakeholders in manner that make water everyone's business Refer: https://www.insightsonindia.com/2021/03/15/framework-for-water-quality-testing- monitoring/ 128. Arrange the following components of India’s forex reserves according to their proportion in ascending order: 1. Foreign currency 2. Gold 3. Special Drawing Rights at IMF Select the correct answer using the code below: (a) 1-2-3 (b) 3-1-2 (c) 2-3-1 (d) 3-2-1 Ans: (a) Explanation: India’s foreign exchange reserves now world’s fourth largest: • India’s foreign exchange reserves surpass Russia’s to become world’s fourth largest. • India’s reserves, enough to cover roughly 18 months of imports, have been bolstered by a rare current-account surplus, rising inflows into the local stock market and foreign direct investment. • India’s foreign currency holdings fell by $4.3 billion to $580.3 billion as of March 5. • China has the largest reserves, followed by Japan and Switzerland on the International Monetary Fund table.

• Refer: facts for prelims: https://www.insightsonindia.com/2021/03/15/insights-daily-current- affairs-pib-summary-15-march-2021/ 129. He lived in the princely kingdom of Travancore in the early decades of the 19th century. He was the first to install a mirror for worshipping in South India. He said, ‘One caste, One religion, One clan, One world, One God’. He was (a) Siva Narayana (b) Narayana Guru (c) Sahodaran Ayyappan (d) Mahatma Ayyankali Ans: (a) Explanation: • Ayya Vaikundar also known as Siva Narayana or Vaikunda Swami is the first and the foremost Purna avatar of Eka-Paran. • Ayya Vaikunda Swamikal (1809-1851) was a great thinker and social reformer of the 19th century.

Telegram: https://t.me/insightsIAStips 70 Youtube: https://www.youtube.com/channel/UCpoccbCX9GEIwaiIe4HLjwA

Revision Through MCQs (RTM) Compilation (March 2021)

• He lived in the princely kingdom of Travancore in the early decades of the 19th century. • Founded ‘Samathwa Samajam’, an early socio-reform movement in India (1836). • He was the first to install a mirror for worshipping in South India. • He was the exponent of a new path of spiritual thoughts named ‘Ayya Vazhi’. • He said, ‘One caste, One religion, One clan, One world, One God’. • He led an agitation known as the Mel Mundu Samaram. Refer: facts for prelims: https://www.insightsonindia.com/2021/03/15/insights-daily-current- affairs-pib-summary-15-march-2021/ 130. Consider the following statements about the “Invest India” agency: 1. It is India’s official agency dedicated to investment promotion and facilitation. 2. It is set up as a joint venture company between the Department of Industrial Policy & Promotion (DIPP) and Confederation of Indian Industry (CII). Which of the given above statements is/are correct? (a) 1 only (b) 2 only (c) Both 1 and 2 (d) Neither 1 nor 2 Ans: (a) Explanation: Atmanirbhar Niveshak Mitra: • To be launched by the Department for Promotion of Industry and Internal Trade (DPIIT), Ministry of Commerce and Industry. • The portal is being developed for handholding and facilitation, information dissemination, and facilitation of domestic investors. • It will give information about approvals, licenses, and clearances required for businesses. • It will also help investors connect to various stakeholders on single platform like Central Ministries, Industry Associations, State Departments. • This Project is under the “Invest India” agency which was set up in 2009 as a non- profit venture under the DPIIT. • Invest India is a non-profit venture under the Department for Promotion of Industry and Internal Trade, Ministry of Commerce and Industry, Government of India. o Operationalized in early 2010, Invest India is set up as a joint venture company between the Department of Industrial Policy & Promotion (DIPP), Ministry of Commerce & Industry (35% equity), Federation of Indian Chambers of Commerce and Industry (FICCI) (51% equity), and State Governments of India (0.5% each). Refer: facts for prelims: https://www.insightsonindia.com/2021/03/15/insights-daily-current- affairs-pib-summary-15-march-2021/

RTM- REVISION THROUGH MCQS – 16th-Mar-2021

131. When a recognised political party splits, which of the following authority has the power to make decision on assigning the symbol? (a) Supreme Court of India (b) High courts of India (c) Election Commission of India (d) Both (a) and (c) Ans: (c) Explanation: How are symbols allotted to political parties? • As per the guidelines, to get a symbol allotted: • A party/candidate has to provide a list of three symbols from the EC’s free symbols list at the time of filing nomination papers.

Telegram: https://t.me/insightsIAStips 71 Youtube: https://www.youtube.com/channel/UCpoccbCX9GEIwaiIe4HLjwA

Revision Through MCQs (RTM) Compilation (March 2021)

• Among them, one symbol is allotted to the party/candidate on a first-come-first-serve basis. • When a recognised political party splits, the Election Commission takes the decision on assigning the symbol. Refer: https://www.insightsonindia.com/2021/03/16/how-election-commission-decides-on-party- symbols-3/ 132. Consider the following statements about Election Commission: 1. Since inception, the Election Commission is a multi-member body consisting of three election commissioners. 2. Election commissioner or a regional commissioner cannot be removed from office except on the recommendation of the chief election commissioner. 3. They determine the territorial areas of the electoral constituencies throughout the country. Which of the above statements is/are correct? (a) 1 and 2 only (b) 3 only (c) 2 and 3 only (d) 1 and 2 only Ans: (c) Explanation: • Since its inception in 1950 and till 15 October 1989, the election commission functioned as a single member body consisting of the Chief Election Commissioner. On 16 October 1989, the president appointed two more election commissioners to cope with the increased work of the election commission on account of lowering of the voting age from 21 to 18 years. Thereafter, the Election Commission functioned as a multimember body consisting of three election commissioners. • Any other election commissioner or a regional commissioner cannot be removed from office except on the recommendation of the chief election commissioner. • EC determine the territorial areas of the electoral constituencies throughout the country on the basis of the Delimitation Commission Act of Parliament. Refer: https://www.insightsonindia.com/2021/03/16/how-election-commission-decides-on-party- symbols-3/ 133. What are the possible actions an Election Commission can take against candidates and political parties? 1. Election Commission cannot bar candidates or leaders from campaigning for specified periods. 2. The EC monitors the adherence of political parties and candidates to the ‘Model Code of Conduct’. 3. It can postpone elections to any constituency, cancel an election already notified, but cannot abrogate or annul an election already held. Which of the above statements is/are incorrect? (a) 1 and 2 only (b) 2 only (c) 1 and 3 only (d) 3 only Ans: (c) Explanation: here the directive word is incorrect!! • What are the possible actions EC can take against candidates and parties? • The EC monitors the adherence of political parties and candidates to the ‘Model Code of Conduct’. • If the violations are also offences under election law and the criminal law of the land, the EC has the power to recommend registration of cases against the offenders. • However, for some violations — such as canvassing for votes during a period when electioneering is barred, making official announcements while the MCC is in force, and

Telegram: https://t.me/insightsIAStips 72 Youtube: https://www.youtube.com/channel/UCpoccbCX9GEIwaiIe4HLjwA

Revision Through MCQs (RTM) Compilation (March 2021)

making appeal to voters on sectarian grounds — the EC has the power to advise or censure candidates, in addition to directing registration of cases. • In some cases, as recent incidents would show, the EC may bar candidates or leaders from campaigning for specified periods. • Asking individuals to leave a constituency or barring entry into certain areas are other powers that the EC may exercise. These powers are not necessarily traceable to any provision in law, but are generally considered inherent because of the sweeping and plenary nature of the EC’s responsibility under the Constitution to ensure free and fair elections. • Its powers extend to postponing elections to any constituency, cancelling an election already notified, and even to abrogate or annul an election already held. Refer: https://www.insightsonindia.com/2021/03/16/how-election-commission-decides-on-party- symbols-3/ 134. Consider the following statements with respect to powers of Election Commission: 1. Election Commission can disqualify candidates who commit electoral malpractices. 2. The Constitution empowers the Election Commission to decide whether a candidate has incurred disqualification by holding an office of profit. Which of the above statements is/are correct? (a) 1 only (b) 2 only (c) Both 1 and 2 (d) Neither 1 nor 2 Ans: (b) Explanation: What are the limitations of the EC’s powers? • The EC does not have the power to disqualify candidates who commit electoral malpractices. At best, it may direct the registration of a case. • The EC also does not have the power to deregister any political party. However, the Constitution empowers the EC to decide whether a candidate has incurred disqualification by holding an office of profit under the appropriate government, or has been declared an insolvent, or acquired the citizenship of a foreign state. • When a question arises whether a candidate has incurred any of these disqualifications, the President of India or Governor has to refer it to the EC. The poll panel’s decision on this is binding. Refer: https://www.insightsonindia.com/2021/03/16/how-election-commission-decides-on-party- symbols-3/ 135. Consider the following statements with reference to 'None Of The Above' (NOTA) on the electronic voting machines (EVMs) and ballot papers: 1. The electronic voting machines (EVMs) have the NOTA option at the end of the candidates' list. 2. The NOTA option was first used in the 2013 assembly elections held in Kerala. 3. NOTA is available only for direct elections not for Rajya Sabha and the Legislative Council. Which of the given above statements is/are correct? (a) 1 and 2 only (b) 2 and 3 only (c) 1 and 3 only (d) 1, 2 and 3 Ans: (c) Explanation: • S1: The NOTA option in the EVM is given at the bottom of the candidates' list. Earlier, in order to cast a negative ballot, a voter had to inform the presiding officer at the polling booth. Now the voter has to just press the NOTA option on the EVM. • S3: NOTA is available only for direct elections such as the Lok Sabha and State assemblies, not for Rajya Sabha and the Legislative Council.

Telegram: https://t.me/insightsIAStips 73 Youtube: https://www.youtube.com/channel/UCpoccbCX9GEIwaiIe4HLjwA

Revision Through MCQs (RTM) Compilation (March 2021)

• S2: NOTA was first used in India in 2009. Chhattisgarh was the first State in the country to give this right to the voters in the local government elections. In the 2014 general elections, the maximum number of NOTA votes were polled in Uttar Pradesh. • On September 27, 2013, the Supreme Court directed the Election Commission to make the necessary provisions in ballot papers and EVMs to provide a ‘None of the Above’ option for voters who come to the polling booth and decide not to vote for any of the candidates in the fray. This was to enable voters to exercise their right not to choose a candidate while maintaining their right to secrecy. Refer: https://www.insightsonindia.com/2021/03/16/nota-none-of-the-above/ 136. With reference to National Population Register (NPR) in India, consider the following statements: 1. It is a Register of usual residents of the country. 2. It is mandatory for every usual resident of India to register in the NPR. 3. It was first collected in 2010 and then updated in 2015. Which of the given above statements is/are correct? (a) 1 and 3 only (b) 1 and 2 only (c) 2 and 3 only (d) 1, 2 and 3 Ans: (d) Explanation: What is National Population Register (NPR)? • It is a Register of usual residents of the country. • It is being prepared at the local (Village/sub-Town), sub-District, District, State and National level under provisions of the Citizenship Act 1955 and the Citizenship (Registration of Citizens and issue of National Identity Cards) Rules, 2003. • It is mandatory for every usual resident of India to register in the NPR. • Objective: To create a comprehensive identity database of every usual resident in the country. • The NPR was first collected in 2010 and then updated in 2015. • Who is a usual resident? • A usual resident is defined for the purposes of NPR as a person who has resided in a local area for the past 6 months or more or a person who intends to reside in that area for the next 6 months or more. Refer: https://www.insightsonindia.com/2021/03/16/national-population-register-2/ 137. Consider the following statements: 1. Jet Streams occur in the Northern Hemisphere only. 2. Only some cyclones develop an eye. 3. The temperature inside the eye of a cyclone is nearly 10 degree Celsius lesser than that of the surroundings. Which of the statements given above is/are correct? (a) 1 only (b) 2 and 3 only (c) 2 only (d) 1 and 3 only Ans: (c) Explanation: • S1: This is obviously incorrect, they occur in upper atmospheres of both hemispheres. • S2: Correct. Extra-tropical cyclones may not always have an eye, whereas mostly mature storms have well developed eye. Rapidly intensifying storms may develop an extremely small, clear, and circular eye, sometimes referred to as a pinhole eye. • S3: It is warmer and not colder for a tropical cyclone. The warmer temperature is what drives the storm.

Telegram: https://t.me/insightsIAStips 74 Youtube: https://www.youtube.com/channel/UCpoccbCX9GEIwaiIe4HLjwA

Revision Through MCQs (RTM) Compilation (March 2021)

Refer: https://www.insightsonindia.com/2021/03/16/national-cyclone-risk-mitigation-project- ncrmp/ 138. Consider the following statements about the Great Indian Bustard: 1. The great Indian bustard is omnivorous. 2. It is listed as endangered on the IUCN Red List. 3. It is protected under Wildlife Protection Act 1972 of India. Which of the given above statements is/are correct? (a) 3 only (b) 1 and 2 only (c) 1 and 3 only (d) 1, 2 and 3 Ans: (c) Explanation: Great Indian Bustards (GIB): • IUCN status: critically endangered. • Listed in Schedule I of the Indian Wildlife (Protection)Act, 1972 and in the CMS Convention and in Appendix I of CITES. • Identified as one of the species for the recovery programme under the Integrated Development of Wildlife Habitats of the Ministry of Environment and Forests. • Project Great Indian Bustard — state of Rajasthan — identifying and fencing off bustard breeding grounds in existing protected areas as well as provide secure breeding enclosures in areas outside protected areas. • Protected areas: Desert National Park Sanctuary — Rajasthan, Rollapadu Wildlife Sanctuary – Andhra Pradesh and Karera Wildlife Sanctuary– Madhya Pradesh. • Habitats in India: • Only two districts in Rajasthan — Jaisalmer and Barmer — have a breeding GIB population in the wild. The bird can also be found in very small numbers in Gujarat, Madhya Pradesh, Karnataka, Maharashtra and Andhra Pradesh.

• Refer: Facts for Prelims: https://www.insightsonindia.com/2021/03/16/insights-daily-current- affairs-pib-summary-16-march-2021/ 139. In which one of the following States is Karera Wildlife Sanctuary located? (a) Arunachal Pradesh (b) Andhra Pradesh (c) Madhya Pradesh (d) Uttar Pradesh Ans: (c) Explanation: • Desert National Park Sanctuary — Rajasthan, Rollapadu Wildlife Sanctuary – Andhra Pradesh and Karera Wildlife Sanctuary– Madhya Pradesh.

Telegram: https://t.me/insightsIAStips 75 Youtube: https://www.youtube.com/channel/UCpoccbCX9GEIwaiIe4HLjwA

Revision Through MCQs (RTM) Compilation (March 2021)

Refer: Facts for Prelims: https://www.insightsonindia.com/2021/03/16/insights-daily-current- affairs-pib-summary-16-march-2021/ 140. Baralacha Pass is a high mountain pass in (a) Karakoram Range (b) Range (c) Zanskar Range (d) Shivalik Range Ans: (c) Explanation: Baralacha Pass: • For the first time ever, the Border Roads Organisation (BRO) has started work on reopening the crucial Baralacha Pass in Himachal Pradesh much before schedule to restore connectivity to Leh in Ladakh. • Baralacha Pass is a high mountain pass in Zanskar range. • Connects Lahaul district in Himachal Pradesh to Leh district in Ladakh, situated along the Leh–Manali Highway. • The pass also acts as a water-divide between the Bhaga river and the Yunam river. Refer: Facts for Prelims: https://www.insightsonindia.com/2021/03/16/insights-daily-current- affairs-pib-summary-16-march-2021/

RTM- REVISION THROUGH MCQS – 17th-Mar-2021

141. The ‘Orunudoi Scheme’ was in news, is launched by which of the following State Government? (a) Assam (b) Meghalaya (c) Sikkim (d) Odisha Ans: (a) Explanation: About the Scheme: • Under the scheme, a monthly assistance of Rs 830 is transferred to women members of marginalised families of Assam. • On account of being a DBT, or a Direct Benefit Transfer scheme, the money is credited directly to the bank account of the woman head of a family because they are “primary caretakers of the household”. • The scheme gives “a choice to the poor and needy households on how they want to spend their money”. Refer: https://www.insightsonindia.com/2021/03/17/orunudoi-scheme/ 142. Consider following statements about the anti-defection law: 1. The anti-defection law is contained in the 10th Schedule of the Constitution. 2. As per the law, if the member voluntarily gives up the membership of the party, he shall be disqualified. 3. The law does not specify a time-period for the Presiding Officer to decide on a disqualification plea. Which of the given above statements is/are correct? (a) 1 and 2 only (b) 2 and 3 only (c) 1 and 3 only (d) 1, 2 and 3 Ans: (d) Explanation: • S1: The anti-defection law is contained in the 10th Schedule of the Constitution. It was enacted by Parliament in 1985. It came into effect on 1st March 1985.

Telegram: https://t.me/insightsIAStips 76 Youtube: https://www.youtube.com/channel/UCpoccbCX9GEIwaiIe4HLjwA

Revision Through MCQs (RTM) Compilation (March 2021)

• S2: There are two grounds on which a member of a legislature can be disqualified. o One, if the member voluntarily gives up the membership of the party, he shall be disqualified. Voluntarily giving up the membership is not the same as resigning from a party. Even without resigning, a legislator can be disqualified if by his conduct the Speaker/Chairman of the concerned House draws a reasonable inference that the member has voluntarily given up the membership of his party. o Second, if a legislator votes in the House against the direction of his party and his action is not condoned by his party, he can be disqualified. These are the two grounds on which a legislator can be disqualified from being a member of the House. • However, there is an exception that was provided in the law to protect the legislators from disqualification. The 10th Schedule says that if there is a merger between two political parties and two-thirds of the members of a legislature party agree to the merger, they will not be disqualified. • S3: The law does not specify a time-period for the Presiding Officer to decide on a disqualification plea. Given that courts can intervene only after the Presiding Officer has decided on the matter, the petitioner seeking disqualification has no option but to wait for this decision to be made. Refer: https://www.insightsonindia.com/2021/03/17/applicability-of-anti-defection-law-for- nominated-mps/ 143. The 10th Schedule was introduced in the Constitution of India during the prime ministership of (a) Indira Gandhi (b) Morarji Desai (c) Rajiv Gandhi (d) Atal Bihari Vajpayee Ans: (c) Explanation: • In 1985, the Rajiv Gandhi government brought a Bill to amend the Constitution and curb defection. The 10th Schedule of the Constitution, which contains the anti- defection law, was added to the Constitution through this amendment. Refer: https://www.insightsonindia.com/2021/03/17/applicability-of-anti-defection-law-for- nominated-mps/ 144. In which one of the following States is Mullaperiyar dam situated? (a) Tamil Nadu (b) Kerala (c) Karnataka (d) Goa Ans: (b) Explanation: Mullaperiyar Dam- what you need to know? • Although the dam is located in Kerala, it is operated by Tamil Nadu following an 1886 lease indenture for 999 years (the Periyar Lake Lease Agreement) that was signed between the Maharaja of Travancore and the Secretary of State for India for the Periyar Irrigation works. • Constructed between 1887 and 1895, the dam redirected the river to flow towards the Bay of Bengal, instead of the Arabian Sea and provide water to the arid rain region of Madurai in Madras Presidency. • The dam is located on the confluence of the Mullayar and Periyar rivers in Kerala’s Idukki district. Refer: https://www.insightsonindia.com/2021/03/17/mullaperiyar-dam-isssue/ 145. The ‘World Air Quality Report’ is released by which of the following? (a) Greenpeace International (b) Germanwatch

Telegram: https://t.me/insightsIAStips 77 Youtube: https://www.youtube.com/channel/UCpoccbCX9GEIwaiIe4HLjwA

Revision Through MCQs (RTM) Compilation (March 2021)

(c) United Nations Environment Programme (d) None of the above Ans: (d) Explanation: • It is a report on air quality released by Swiss organisation, IQAir. • The global cities ranking report is based on PM2.5 data from 106 countries, which is measured by ground-based monitoring stations, most of which are operated by government agencies. Refer: https://www.insightsonindia.com/2021/03/17/the-world-air-quality-report-2020/

146. With reference to ‘National Bank for Financing Infrastructure and Development’, which of the following statements is/are correct? 1. It is a wholly owned subsidiary of Reserve Bank of India. 2. It has a corpus of Rs. 2, 00,000 crore at present. Select the correct answer using the code given below: (a) 1 only (b) 2 only (c) Both 1 and 2 (d) Neither 1 nor 2 Ans: (d) Explanation: • The Union Cabinet has approved a Bill to set up a Development Finance Institution – National Bank for Financing Infrastructure and Development (NaBFID). • S2: NaBFID will be set up with a corpus of ₹20,000 crore and an initial grant of ₹5,000 crore from the government. • S1: Initially, it will be wholly owned by the Government but the Government stake will be lowered to a quarter. • It will also enjoy some tax benefits for an initial 10-year period and some amendments will be carried out in the Indian Stamp Act in this regard. • It will have a professional board and at least 50% of members will be non-official directors. • An eminent person will be appointed chairperson. Refer: https://www.insightsonindia.com/2021/03/17/cabinet-gives-nod-to-bill-for-setting-up-dfi/ 147. Consider the following statements about Chenab River: 1. The Chenab River originates at the Rimo Glacier. 2. The waters of the Chenab were allocated to Pakistan under the terms of the Indus Waters Treaty. Which of the given above statements is/are correct? (a) 1 only (b) 2 only (c) Both 1 and 2 (d) Neither 1 nor 2 Ans: (b) Explanation: • S2: the Indus Water Treaty of 1960 allocates the Chenab River to Pakistan for exploitation. India can use its water for domestic and agricultural uses or for "non- consumptive" uses such as hydro power. • S1: It rises in the upper Himalayas in the Lahaul and Spiti district of Himachal Pradesh state, India, and flows through the Jammu region of Jammu and Kashmir into the plains of Punjab, Pakistan, before flowing into the near the city of Uch Sharif. • World’s highest railway bridge:

Telegram: https://t.me/insightsIAStips 78 Youtube: https://www.youtube.com/channel/UCpoccbCX9GEIwaiIe4HLjwA

Revision Through MCQs (RTM) Compilation (March 2021)

o It is being constructed over the Chenab river in Jammu and Kashmir. o The ₹1,250-crore bridge will be 359 metres above the Chenab river bed. o The rail line is expected to be 35 metres taller than the Eiffel Tower. o Once completed, it will surpass the record of the Beipan river Shuibai railway bridge (275 m) in China. Refer: Facts for Prelims: https://www.insightsonindia.com/2021/03/17/insights-daily-current- affairs-pib-summary-17-march-2021/ 148. Consider the following statements about the International Renewable Energy Agency (IRENA): 1. It is the first international organisation to focus exclusively on renewable energy. 2. It is best known for the publication of its annual World Energy Outlook. 3. The agency is headquartered in Abu Dhabi, United Arab Emirates. Which of the given above statements is/are correct? (a) 1 and 2 only (b) 2 and 3 only (c) 1 and 3 only (d) 1, 2 and 3 Ans: (c) Explanation: About IRENA: • It is an intergovernmental organisation mandated to facilitate cooperation, advance knowledge, and promote the adoption and sustainable use of renewable energy. • S1: It is the first international organisation to focus exclusively on renewable energy, addressing needs in both industrialized and developing countries. • S3: It was founded in 2009 & its statute entered into force on 8 July 2010 and is headquartered in Masdar City, Abu Dhabi. • IRENA is an official United Nations observer. • S2: The International Energy Agency is best known for the publication of its annual World Energy Outlook. Refer: Facts for Prelims: https://www.insightsonindia.com/2021/03/17/insights-daily-current- affairs-pib-summary-17-march-2021/ 149. What is/are the consequence/consequences of a country becoming the member of the ‘Nuclear Suppliers Group’? 1. It will have access to the latest and most efficient nuclear technologies. 2. It automatically becomes a member of “The Treaty on the Non-Proliferation of Nuclear Weapons (NPT)” Which of the statements given above is/are correct? (a) 1 only (b) 2 only (c) Both 1 and 2 (d) Neither 1 nor 2 Ans: (a) Explanation: • S1 and S2: The Nuclear Suppliers Group (NSG) is a group of nuclear supplier countries that seeks to contribute to the non-proliferation of nuclear weapons through the implementation of two sets of Guidelines for nuclear exports and nuclear-related exports. But becoming the member of the ‘Nuclear Suppliers Group’ will NOT automatically make a country a member of “The Treaty on the Non-Proliferation of Nuclear Weapons (NPT). • The Nuclear Suppliers Group (NSG) is a multilateral export control regime and a group of nuclear supplier countries that seek to prevent nuclear proliferation by controlling the export of materials, equipment and technology that can be used to manufacture nuclear weapons.

Telegram: https://t.me/insightsIAStips 79 Youtube: https://www.youtube.com/channel/UCpoccbCX9GEIwaiIe4HLjwA

Revision Through MCQs (RTM) Compilation (March 2021)

• Membership of the NSG enables access to technology for a range of uses from medicine to building nuclear power plants for India from the NSG which is essentially a traders’ cartel. Source/Refer: UPSC CSE 2018: http://www.nuclearsuppliersgroup.org/en/ 150. If a commodity is provided free to the public by the Government, then (a) The opportunity cost is zero. (b) The opportunity cost is ignored. (c) The opportunity cost is transferred from the consumers of the product to the tax-paying public. (d) The opportunity cost is transferred from the consumers of the product to the Government. Ans: (c) Explanation: • Opportunity cost represents the benefits an individual, investor or business misses out on when choosing one alternative over another. • When you have the opportunity to access public services for free, this would always come at a cost of somebody paying for it. In this case, the tax payer bears the opportunity cost. Source: UPSC CSE 2018

RTM- REVISION THROUGH MCQS – 18th-Mar-2021

151. Consider the following statements: 1. As per article 114 of the Constitution of India, the government can withdraw money from the Consolidated Fund only after receiving approval from Parliament. 2. The Appropriation Bill is first passed by the Lok Sabha and then sent to the Rajya Sabha. 3. The defeat of an Appropriation Bill in a parliamentary vote would necessitate resignation of a government. Which of the give above statements is/are correct? (a) 1 and 2 only (b) 2 and 3 only (c) 1 and 3 only (d) 1, 2 and 3 Ans: (d) Explanation: • What is Appropriation Bill? o Appropriation Bill is a money bill that allows the government to withdraw funds from the Consolidated Fund of India to meet its expenses during the course of a financial year. o As per article 114 of the Constitution, the government can withdraw money from the Consolidated Fund only after receiving approval from Parliament. o To put it simply, the Finance Bill contains provisions on financing the expenditure of the government, and Appropriation Bill specifies the quantum and purpose for withdrawing money. • Procedure followed: o The government introduces the Appropriation Bill in the lower house of Parliament after discussions on Budget proposals and Voting on Demand for Grants. o The Appropriation Bill is first passed by the Lok Sabha and then sent to the Rajya Sabha. o The Rajya Sabha has the power to recommend any amendments in this Bill. However, it is the prerogative of the Lok Sabha to either accept or reject the recommendations made by the upper house of Parliament.

Telegram: https://t.me/insightsIAStips 80 Youtube: https://www.youtube.com/channel/UCpoccbCX9GEIwaiIe4HLjwA

Revision Through MCQs (RTM) Compilation (March 2021)

o The unique feature of the Appropriation Bill is its automatic repeal clause, whereby the Act gets repealed by itself after it meets its statutory purpose. • What happens when the bill is defeated? o Since India subscribes to the Westminster system of parliamentary democracy, the defeat of an Appropriation Bill (and also the Finance Bill) in a parliamentary vote would necessitate resignation of a government or a general election. This has never happened in India till date, though. Refer: https://www.insightsonindia.com/2021/03/18/appropriation-bill-gets-the-nod-of-lok-sabha/ 152. In which of the following matters are the powers and status of the Rajya Sabha unequal to that of the Lok Sabha? 1. Initiation of removal of the Vice President 2. Approval of ordinances issued by the President 3. Introduction and passage of financial bills involving expenditure from the Consolidated Fund of India Select the correct answer using the code given below: (a) 1 Only (b) 1 and 2 Only (c) 2 and 3 Only (d) 1, 2 and 3 Ans: (a) Explanation: • Both, the Lok Sabha as well as the Rajya Sabha are involved in the process of election and removal of the Vice President. But the RS alone can initiate the removal of the VP. Statements 2 and 3 are applicable to both RS and LS. • Page 22.20, 22.28 and 22.29; Indian Polity 4th Edition by M Laxmikanth Refer: https://www.insightsonindia.com/2021/03/18/appropriation-bill-gets-the-nod-of-lok-sabha/ 153. Consider the following statements: 1. As per the MMDR Act, 1957 Central Governments have complete powers for making Rules for grant of concessions in respect of extraction of minor minerals. 2. The power to frame policy and legislation relating to minor minerals is entirely delegated to the State Governments. Which of the given above statements is/are correct? (a) 1 only (b) 2 only (c) Both 1 and 2 (d) Neither 1 nor 2 Ans: (b) Explanation: • S2: The power to frame policy and legislation relating to minor minerals is entirely delegated to the State Governments. o While policy and legislation relating to the major minerals are dealt by the Ministry of Mines under Union /Central Government. o Various State Governments have indeed prescribed rules for the grant of mineral concessions in respect of minerals classified as minor minerals under the MMDR Act, 1957. Minor Minerals get specified in the schedule appended in Minor Mineral concession Rules issued by States. o Thus, as opposed to major minerals, the regulatory and administrative jurisdiction of minor minerals falls under the purview of State governments. These include the powers to frame rules, prescribe rates of royalty, contribution to District Mineral Foundation, the procedure for grant of mineral concessions, regulation of their mining, control of illegal mining etc.

Telegram: https://t.me/insightsIAStips 81 Youtube: https://www.youtube.com/channel/UCpoccbCX9GEIwaiIe4HLjwA

Revision Through MCQs (RTM) Compilation (March 2021)

o In the case of major minerals, States substantially regulate and develop minerals subject to provisions of the MMDR Act, and after prior permissions from the central government. • S1: The central government has the power to notify “minor minerals” under section 3 (e) of the MMDR Act, 1957. On the other hand, as per Section 15 of the MMDR Act, 1957 State Governments have complete powers for making Rules for grant of concessions in respect of extraction of minor minerals and levy and collection of royalty on minor minerals. Refer: https://www.insightsonindia.com/2021/03/18/mmdr-amendment-bill-2021/ 154. The Bibek Debroy Committee was in news recently, is related to which of the following? (a) Fiscal Responsibility and Budget Management (b) Privatisation of Indian Railways (c) Data Protection Framework for India (d) Restructuring of Indian Railways (IR) Ans: (d) Explanation: • The Bibek Debroy committee on the proposed restructuring of Indian Railways (IR) has recommended private sector participation in projects and setting up an independent regulator to promote competition in the segment. • “It needs to be understood that this committee does not recommend privatisation of IR," the panel clarifies. "It does, however, endorse private entry, with the provision of an independent regulator,” officials said the committee had said in its final report. • The panel also said that if all its recommendations highlighted for the first five years -- creation of a independent regulator, reorganisation of the Railway Board, reorganisation of Group A railway services, revision of dividend policy and many others -- are implemented and issues related to social costs are addressed, the annual railway budget could be phased out. The Gross Budgetary Support (GBS) to IR could be mentioned as a paragraph in the Union Budget and no more. Refer: https://www.insightsonindia.com/2021/03/18/dont-privatise-railways-says-opposition/ 155. What are the possible actions an Election Commission can take against candidates and political parties? 1. Election Commission cannot bar candidates or leaders from campaigning for specified periods. 2. The EC monitors the adherence of political parties and candidates to the ‘Model Code of Conduct’. 3. It can postpone elections to any constituency, cancel an election already notified, but cannot abrogate or annul an election already held. Which of the above statements is/are incorrect? (e) 1 and 2 only (f) 2 only (g) 1 and 3 only (h) 3 only Ans: (c) Explanation: here the directive word is incorrect!! • What are the possible actions EC can take against candidates and parties? • The EC monitors the adherence of political parties and candidates to the ‘Model Code of Conduct’. • If the violations are also offences under election law and the criminal law of the land, the EC has the power to recommend registration of cases against the offenders. • However, for some violations — such as canvassing for votes during a period when electioneering is barred, making official announcements while the MCC is in force, and making appeal to voters on sectarian grounds — the EC has the power to advise or censure candidates, in addition to directing registration of cases.

Telegram: https://t.me/insightsIAStips 82 Youtube: https://www.youtube.com/channel/UCpoccbCX9GEIwaiIe4HLjwA

Revision Through MCQs (RTM) Compilation (March 2021)

• In some cases, as recent incidents would show, the EC may bar candidates or leaders from campaigning for specified periods. • Asking individuals to leave a constituency or barring entry into certain areas are other powers that the EC may exercise. These powers are not necessarily traceable to any provision in law, but are generally considered inherent because of the sweeping and plenary nature of the EC’s responsibility under the Constitution to ensure free and fair elections. • Its powers extend to postponing elections to any constituency, cancelling an election already notified, and even to abrogate or annul an election already held. Refer: https://www.insightsonindia.com/2021/03/16/how-election-commission-decides-on-party- symbols-3/ 156. Consider the following statements with respect to powers of Election Commission: 1. Election Commission can disqualify candidates who commit electoral malpractices. 2. The Constitution empowers the Election Commission to decide whether a candidate has incurred disqualification by holding an office of profit. Which of the above statements is/are correct? (e) 1 only (f) 2 only (g) Both 1 and 2 (h) Neither 1 nor 2 Ans: (b) Explanation: What are the limitations of the EC’s powers? • The EC does not have the power to disqualify candidates who commit electoral malpractices. At best, it may direct the registration of a case. • The EC also does not have the power to deregister any political party. However, the Constitution empowers the EC to decide whether a candidate has incurred disqualification by holding an office of profit under the appropriate government, or has been declared an insolvent, or acquired the citizenship of a foreign state. • When a question arises whether a candidate has incurred any of these disqualifications, the President of India or Governor has to refer it to the EC. The poll panel’s decision on this is binding. Refer: https://www.insightsonindia.com/2021/03/16/how-election-commission-decides-on-party- symbols-3/ 157. Consider the following statements with reference to 'None Of The Above' (NOTA) on the electronic voting machines (EVMs) and ballot papers: 1. The electronic voting machines (EVMs) have the NOTA option at the end of the candidates' list. 2. The NOTA option was first used in the 2013 assembly elections held in Kerala. 3. NOTA is available only for direct elections not for Rajya Sabha and the Legislative Council. Which of the given above statements is/are correct? (a) 1 and 2 only (b) 2 and 3 only (c) 1 and 3 only (d) 1, 2 and 3 Ans: (c) Explanation: • S1: The NOTA option in the EVM is given at the bottom of the candidates' list. Earlier, in order to cast a negative ballot, a voter had to inform the presiding officer at the polling booth. Now the voter has to just press the NOTA option on the EVM. • S3: NOTA is available only for direct elections such as the Lok Sabha and State assemblies, not for Rajya Sabha and the Legislative Council.

Telegram: https://t.me/insightsIAStips 83 Youtube: https://www.youtube.com/channel/UCpoccbCX9GEIwaiIe4HLjwA

Revision Through MCQs (RTM) Compilation (March 2021)

• S2: NOTA was first used in India in 2009. Chhattisgarh was the first State in the country to give this right to the voters in the local government elections. In the 2014 general elections, the maximum number of NOTA votes were polled in Uttar Pradesh. • On September 27, 2013, the Supreme Court directed the Election Commission to make the necessary provisions in ballot papers and EVMs to provide a ‘None of the Above’ option for voters who come to the polling booth and decide not to vote for any of the candidates in the fray. This was to enable voters to exercise their right not to choose a candidate while maintaining their right to secrecy. Refer: https://www.insightsonindia.com/2021/03/16/nota-none-of-the-above/ 158. Consider the following statements about the Great Indian Bustard: 1. The great Indian bustard is omnivorous. 2. It is listed as endangered on the IUCN Red List. 3. It is protected under Wildlife Protection Act 1972 of India. Which of the given above statements is/are correct? (a) 3 only (b) 1 and 2 only (c) 1 and 3 only (d) 1, 2 and 3 Ans: (c) Explanation: Great Indian Bustards (GIB): • IUCN status: critically endangered. • Listed in Schedule I of the Indian Wildlife (Protection)Act, 1972 and in the CMS Convention and in Appendix I of CITES. • Identified as one of the species for the recovery programme under the Integrated Development of Wildlife Habitats of the Ministry of Environment and Forests. • Project Great Indian Bustard — state of Rajasthan — identifying and fencing off bustard breeding grounds in existing protected areas as well as provide secure breeding enclosures in areas outside protected areas. • Protected areas: Desert National Park Sanctuary — Rajasthan, Rollapadu Wildlife Sanctuary – Andhra Pradesh and Karera Wildlife Sanctuary– Madhya Pradesh. • Habitats in India: • Only two districts in Rajasthan — Jaisalmer and Barmer — have a breeding GIB population in the wild. The bird can also be found in very small numbers in Gujarat, Madhya Pradesh, Karnataka, Maharashtra and Andhra Pradesh.

• Refer: Facts for Prelims: https://www.insightsonindia.com/2021/03/16/insights-daily-current- affairs-pib-summary-16-march-2021/ 159. In which one of the following States is Karera Wildlife Sanctuary located? (a) Arunachal Pradesh (b) Andhra Pradesh

Telegram: https://t.me/insightsIAStips 84 Youtube: https://www.youtube.com/channel/UCpoccbCX9GEIwaiIe4HLjwA

Revision Through MCQs (RTM) Compilation (March 2021)

(c) Madhya Pradesh (d) Uttar Pradesh Ans: (c) Explanation: • Desert National Park Sanctuary — Rajasthan, Rollapadu Wildlife Sanctuary – Andhra Pradesh and Karera Wildlife Sanctuary– Madhya Pradesh. Refer: Facts for Prelims: https://www.insightsonindia.com/2021/03/16/insights-daily-current- affairs-pib-summary-16-march-2021/ 160. Baralacha Pass is a high mountain pass in (a) Karakoram Range (b) Ladakh Range (c) Zanskar Range (d) Shivalik Range Ans: (c) Explanation: Baralacha Pass: • For the first time ever, the Border Roads Organisation (BRO) has started work on reopening the crucial Baralacha Pass in Himachal Pradesh much before schedule to restore connectivity to Leh in Ladakh. • Baralacha Pass is a high mountain pass in Zanskar range. • Connects Lahaul district in Himachal Pradesh to Leh district in Ladakh, situated along the Leh–Manali Highway. • The pass also acts as a water-divide between the Bhaga river and the Yunam river. Refer: Facts for Prelims: https://www.insightsonindia.com/2021/03/16/insights-daily-current- affairs-pib-summary-16-march-2021/

Telegram: https://t.me/insightsIAStips 85 Youtube: https://www.youtube.com/channel/UCpoccbCX9GEIwaiIe4HLjwA

Revision Through MCQs (RTM) Compilation (March 2021)

RTM- REVISION THROUGH MCQS – 19th-Mar-2021

161. Consider the following statements with reference to Electoral Bonds: 1. An electoral bond is like a promissory note that can be bought by any Indian citizen or company incorporated in India. 2. The electoral bonds are available for purchase for 10 days in the beginning of every year. 3. The electoral bonds will not bear the name of the donor. Which of the given above statements is/are correct? (a) 1 and 3 only (b) 2 and 3 only (c) 1 and 2 only (d) 1, 2 and 3 Ans: (a) Explanation: What are electoral bonds? • S1 and S2: : Electoral Bond is a financial instrument for making donations to political parties. The bonds are available for purchase by any person (who is a citizen of India or incorporated or established in India) for a period of ten days each in the months of January, April, July and October as may be specified by the Central Government. • The bonds are issued in multiples of Rs. 1,000, Rs. 10,000, Rs. 1 lakh, Rs. 10 lakh and Rs. 1 crore without any maximum limit. • State Bank of India is authorised to issue and encash these bonds, which are valid for fifteen days from the date of issuance. • These bonds are redeemable in the designated account of a registered political party. • A person being an individual can buy bonds, either singly or jointly with other individuals. • S3: Donor’s name is not mentioned on the bond. Refer: https://www.insightsonindia.com/2021/03/19/sc-to-hear-plea-against-sale-of-electoral- bonds/ 162. Which of the following is/are the restrictions that were done away with after the introduction of the electoral bond scheme? 1. No foreign company could donate to any political party under the Companies Act. 2. A firm could only donate a maximum of 7.5 per cent of its average three year net profit as political donations. 3. Companies had to disclose details of their political donations in their annual statement of accounts. Select the correct answer using the code below: (a) 1 only (b) 2 and 3 only (c) 1 and 3 only (d) 1, 2 and 3 Ans: (d) Explanation: • Restrictions that were done away with after the introduction of the electoral bond scheme: o Earlier, no foreign company could donate to any political party under the Companies Act o A firm could donate a maximum of 7.5 per cent of its average three year net profit as political donations according to Section 182 of the Companies Act o As per the same section of the Act, companies had to disclose details of their political donations in their annual statement of accounts. • The government moved an amendment in the Finance Bill to ensure that this proviso would not be applicable to companies in case of electoral bonds.

Telegram: https://t.me/insightsIAStips 86 Youtube: https://www.youtube.com/channel/UCpoccbCX9GEIwaiIe4HLjwA

Revision Through MCQs (RTM) Compilation (March 2021)

• Thus, Indian, foreign and even shell companies can now donate to political parties without having to inform anyone of the contribution. Refer: https://www.insightsonindia.com/2021/03/19/sc-to-hear-plea-against-sale-of-electoral- bonds/ 163. Consider the following statements about Electoral Bonds: 1. Electoral Bonds can be purchased by only citizen of India. 2. If the Electoral Bond is deposited after expiry of the validity period, no payment is made to any Political Party. 3. The electoral bond deposited by an eligible political party in its account shall be credited on the same day. Which of the above statements is/are correct? (a) 1 and 2 only (b) 2 and 3 only (c) 1 and 3 only (d) 1, 2 and 3 Ans: (d) Explanation: • S1: As per provisions of the scheme, electoral bonds may be purchased by a person, who is a citizen of India or entities incorporated or established in India. • S2: The electoral bonds will be valid for 15 calendar days from the date of issue and no payment will be made to any payee political party if the bond is deposited after expiry of the validity period. • S3: The electoral bond deposited by an eligible political party in its account shall be credited on the same day. Refer: https://www.insightsonindia.com/2021/03/19/sc-to-hear-plea-against-sale-of-electoral- bonds/ 164. Consider the following statements: 1. Ration card is an official document issued by central authority in India to households. 2. Under the National Food Security Act (NFSA), all state governments have to identify eligible households under the Public Distribution System. Which of the given above statements is/are correct? (a) 1 only (b) 2 only (c) Both 1 and 2 (d) Neither 1 nor 2 Ans: (b) Explanation: What are Ration Cards? • Ration card is an official document issued by state governments in India to households. The Household should be eligible to purchase subsidized food grain under the National Food Security Act (NFSA). They also serve as a common form of identification for many Indians. • Under the NFSA, all state governments have to identify eligible households under the Public Distribution System and provide them with ration cards. Refer: https://www.insightsonindia.com/2021/03/19/sc-sought-centres-response-in-ration-cards- cancellation-case/

165. Consider the following statements: 1. The Chambal River is a tributary of the Banas River, itself a tributary of the Yamuna River in central India. 2. The Banas River lies entirely within the state of Rajasthan in western India. Which of the given above statements is/are correct? (a) 1 only

Telegram: https://t.me/insightsIAStips 87 Youtube: https://www.youtube.com/channel/UCpoccbCX9GEIwaiIe4HLjwA

Revision Through MCQs (RTM) Compilation (March 2021)

(b) 2 only (c) Both 1 and 2 (d) Neither 1 nor 2 Ans: (b) Explanation: • The Banas is a river which lies entirely within the state of Rajasthan in western India. It is a tributary of the Chambal River, itself a tributary of the Yamuna, which in turn merges into the Ganga. • What is the Eastern Rajasthan Canal Project (ERCP)? o The Eastern Rajasthan Canal Project aims to harvest surplus water available during the rainy season in rivers in southern Rajasthan such as Chambal and its tributaries including Kunnu, Parvati, Kalisindh. o Besides supplying drinking water in 13 districts, the mega project will also provide irrigation water to an additional 2 lakh hectares of land. o It will also supply water to the Delhi-Mumbai Industrial Corridor and take care of the flood and drought situation in the area. Refer: https://www.insightsonindia.com/2021/03/19/eastern-rajasthan-canal-project-ercp/ 166. If you want to see gharials in their natural habitat, which one of the following is the best place to visit? (a) Bhitarkanika Mangroves (b) Chambal River (c) Pulicat Lake (d) Deepor Beel Ans: (b) Explanation: • Option A: Gharials inhabit deep freshwater habitats, not both freshwater and saltwater habitats as inhabited by crocodile species. So, A is ruled out. Option A is the home of saltwater crocodile. • Option B: Gharials can be naturally found in National Chambal Gharial Wildlife Sanctuary. • The gharial is one of three crocodilians native to India, the other two being the mugger crocodile and the saltwater crocodile. Refer: https://www.insightsonindia.com/2021/03/19/eastern-rajasthan-canal-project-ercp/ 167. Due to some reasons, if photosynthesis in plants suddenly stopped happening, what could be its likely consequence/consequences? 1. most plants would die within short order 2. green algal blooms would cover the entire face of the planet 3. global emissions of CO2 would be greatly reduced Select the correct answer using the code given below: (a) 1 only (b) 2 and 3 only (c) 1 and 2 only (d) 1, 2 and 3 Ans: (a) Explanation: • If photosynthesis came to an abrupt end, most plants would die within short order. Although they could hold out for a few days -- or in some cases, a few weeks -- how long they lived would largely be a factor of how much sugar they had stored within their cells. • The only organisms able to exist under such conditions would be the chemosynthetic bacteria, which can utilize the chemical energy of certain inorganic compounds and thus are not dependent on the conversion of light energy. • S2 and S3 are absurd statements.

Telegram: https://t.me/insightsIAStips 88 Youtube: https://www.youtube.com/channel/UCpoccbCX9GEIwaiIe4HLjwA

Revision Through MCQs (RTM) Compilation (March 2021)

Refer: https://www.insightsonindia.com/2021/03/19/artificial-photosynthesis-to-provide-solutions- for-carbon-capture-and-conversion/ 168. Technical textiles are: (a) the high-tenacity fibres which are lightest and toughest fabrics mainly used in automobile and aerospace industries (b) the toughest fabrics which are much heavier than polyester and used in power industries (c) the toughest fabrics having rigidity mainly used in polyhouse construction (d) the high-tenacity fabrics having fire resistance property Ans: (a) Explanation: What are technical textiles? • Technical textiles are defined as textile materials and products manufactured primarily for their technical performance and functional properties rather than aesthetic and decorative characteristics. • Depending upon their application areas, Technical Textiles products are divided into 12 broad categories: Agrotech, Buildtech, Clothtech, Geotech, Hometech, Indutech, Mobiltech, Meditech, Protech, Sportstech, Oekotech, Packtech. Refer: https://www.insightsonindia.com/2021/03/19/national-technical-textiles-mission-2/ 169. With reference to High Electron Mobility Transistor (HEMT), consider the following statements: 1. They can be used in cellular telecommunications. 2. They can be used for radar imaging and radio astronomy. 3. They can be used in biochemical sensors. Which of the given above statements is/are correct? (a) 1 and 2 only (b) 2 and 3 only (c) 1 and 3 only (d) 1, 2 and 3 Ans: (d) Explanation: What are HEMTs? • High Electron Mobility Transistor (HEMT) is a normally OFF device and can switch currents up to 4A and operates at 600 V. • HEMTs are used in integrated circuits as digital on-off switches. • HEMT transistors are able to operate at higher frequencies than ordinary transistors, up to millimeter wave frequencies, and are used in high-frequency products such as cell phones, satellite television receivers, voltage converters, and radar equipment. • They are widely used in satellite receivers, in low power amplifiers and in the defense industry. • S3: https://link.springer.com/article/10.1007/s12633-020-00937-w Refer: https://www.insightsonindia.com/2021/03/19/high-electron-mobility-transistors-hemts/ 170. The ‘Go Electric’ campaign was in news recently, is launched by: (a) NITI Aayog (b) Ministry of New and Renewable Energy (c) Ministry of Power (d) Ministry of Petroleum and Natural Gas Ans: (c) Explanation: Go Electric Campaign: • Campaign launched by the Ministry of Power. • Objective: Creating awareness among masses on benefits of adopting Electric Vehicles and Electric Cooking appliances such as Induction cook hobs, Electric pressure cooker etc. Refer: facts for prelims: https://www.insightsonindia.com/2021/03/19/insights-daily-current- affairs-pib-summary-19-march-2021/

Telegram: https://t.me/insightsIAStips 89 Youtube: https://www.youtube.com/channel/UCpoccbCX9GEIwaiIe4HLjwA

Revision Through MCQs (RTM) Compilation (March 2021)

RTM- REVISION THROUGH MCQS – 20th-Mar-2021

171. With reference to the Parliament of India, which one of the following is granted when a need has arisen during the current financial year for supplementary expenditure upon some new service not contemplated in the Budget for that year? (a) Excess Grant (b) Exceptional Grants (c) Additional Grant (d) Token Grants Ans: (c) Explanation: • Additional Grant: It is granted when a need has arisen during the current financial year for supplementary or additional expenditure upon some new service not contemplated in the Budget for that year. • Excess Grant: It is granted when money has been spent on any service during a financial year in excess of the amount granted for that year. The demands for excess grants are made after the expenditure has actually been incurred and after the financial year to which it relates, has expired. • Exceptional Grants: It is granted for an exceptional purpose which forms no part of the current service of any financial year. • Token Grant: It is granted when funds to meet proposed expenditure on a new service can be made available by re-appropriation, a demand for the grant of a token sum may be submitted to the vote of the House and, if the House assents to the demand, funds may be so made available. Refer: https://www.insightsonindia.com/2021/03/20/supplementary-demand-for-grants/ 172. With reference to vote-on-account in Parliament of India, consider the following statements: 1. It is passed after the general discussion on budget is over. 2. It only deals with the expenditure side of the government's budget. 3. It is a formality and requires no debate in parliament. Which of the given above statements is/are correct? (a) 1 and 2 only (b) 2 and 3 only (c) 1 and 3 only (d) 1, 2 and 3 Ans: (d) Explanation: • The constitution says that no money can be withdrawn by the government from the Consolidated Fund of India except under appropriation made by law. For that an appropriation bill is passed during the Budget process. However, the appropriation bill may take time to pass through the Parliament and become a law. Meanwhile, the government would need permission to spend even a single penny from April 1 when the new financial year starts. Vote on account is the permission to withdraw money from the Consolidated Fund of India in that period, usually two months. Vote on account is a formality and requires no debate. When elections are scheduled a few months into the new financial year, the government seeks vote on account for four months. Essentially, vote on account is the interim permission of the parliament to the government to spend money. • So vote on account is just an interim permission to spend money as against a full Budget which is an elaborate financial statement of expenditure and receipts including changes in taxes and government policies. It is passed (or granted) after the general discussion on budget is over. In the election year when the elections are scheduled a few months into the new financial year, the government prefers to seek a vote on account instead of

Telegram: https://t.me/insightsIAStips 90 Youtube: https://www.youtube.com/channel/UCpoccbCX9GEIwaiIe4HLjwA

Revision Through MCQs (RTM) Compilation (March 2021)

presenting a full Budget because if the government changes after elections it is not fair to deny it the right to design it. • A vote-on-account only deals with the expenditure side of the government's budget while an Interim Budget is a complete set of accounts, including both expenditure and receipts, akin to a full budget. Refer: https://www.insightsonindia.com/2021/03/20/supplementary-demand-for-grants/ 173. Consider the following statements 1. According to Constitution of India, A person who eligible to vote can be made a minister in a state for six months even if he/she is not a member of the legislature of that state 2. According to the representation of people act, 1951, a person convicted of a criminal offence and sentenced to imprisonment for at least five years is permanently disqualified from contesting an election even after his release from prison. Which of the above given statements is/are correct? (a) 1 only (b) 2 only (c) Both 1 and 2 (d) Neither 1 nor 2 Ans: (a) Explanation: • (If one overthinks the answer could be ‘D’ – overthinking that Voter eligibility is not mentioned in the Constitution and Age 25 is not mentioned. As you know UPSC doesn’t reward overthinking. Here the question is related to checking the concept rather than fact) • S1: Not quite technically true, since one who is eligible to vote isn’t necessarily eligible to be a Minister (for e.g. on grounds of age). But, since UPSC has used the term a voter “can” and not “every voter”, S1 is correct since many voters who meet the eligibility to be a Member of Parliament can be made a Minister. • For e.g. here are few correct and incorrect statements: • CORRECT: A voter CAN become a minister for six months. • CORRECT: A voter eligible to become a Member of Parliament CAN become a minister for six months. • CORRECT: Every voter that is eligible to become a Member of Parliament CAN become a minister for six months. • First statement is only part of a larger correct statement, still true. • INOCRRECT: EVERY voter can become a minister for six months. • S2: Specifically, Section 8 of RPA, 1951 states the grounds for disqualification on conviction of certain offences pursuant to Section 8 of RPA, 1951, in case the candidate is convicted of certain offences specified under sub-section (1) (2) (3) of Sec. 8 and sentenced to indicated fine or imprisonment, then such a candidate is disqualified from being a member of either house of Parliament or State Legislature from the date of such conviction till the stated time. This is not permanent, so S2 is wrong. Refer: UPSC CSE 2020: https://www.insightsonindia.com/2021/03/20/sc-stays-hc-decision- barring-aided-school-teachers-from-contesting-polls/

174. With reference to Insurance Sector in India, consider the following statements: 1. Among the life insurers, Life Insurance Corporation (LIC) is the sole public sector company. 2. There are six public sector insurers in the non-life insurance segment. 3. Currently, the insurance penetration in India is 37 percent of the gross domestic product (GDP) as against the world average which is 61 percent. Which of the given above statements is/are correct? (a) 1 and 2 only (b) 1 only

Telegram: https://t.me/insightsIAStips 91 Youtube: https://www.youtube.com/channel/UCpoccbCX9GEIwaiIe4HLjwA

Revision Through MCQs (RTM) Compilation (March 2021)

(c) 2 and 3 only (d) 1, 2 and 3 Ans: (a) Explanation: • The insurance industry of India has 57 insurance companies 24 are in the life insurance business, while 33 are non-life insurers. Among the life insurers, Life Insurance Corporation (LIC) is the sole public sector company. There are six public sector insurers in the non-life insurance segment. In addition to these, there is a sole national re-insurer, namely General Insurance Corporation of India (GIC Re). Other stakeholders in the Indian Insurance market include agents (individual and corporate), brokers, surveyors and third-party administrators servicing health insurance claims. More>> • According to the economic survey, insurance penetration in India increased to 3.76 per cent in 2019 from the 2.71 per cent in 2001. However, it was much lower than Malaysia, Thailand, and China’s, which clocked 4.72, 4.99 and 4.30 per cent, respectively in 2019. Refer: https://www.insightsonindia.com/2021/03/20/insurance-amendment-bill-2021/ 175. The Horn of Africa is a term used to denote the region containing: 1. Eritrea 2. Ethiopia 3. Somalia 4. Kenya Select the correct answer using the code below: (a) 1, 2 and 3 only (b) 1, 2 and 4 only (c) 2, 3 and 4 only (d) 1, 2, 3 and 4 Ans: (a) Explanation: • The Horn of Africa: o Eritrea o Djibouti o Somalia o Ethiopia

• Refer: https://www.insightsonindia.com/2021/03/20/the-indian-ocean-border-dispute-between- kenya-and-somalia/

Telegram: https://t.me/insightsIAStips 92 Youtube: https://www.youtube.com/channel/UCpoccbCX9GEIwaiIe4HLjwA

Revision Through MCQs (RTM) Compilation (March 2021)

176. Consider the following statements regarding Indian Ocean Commission: 1. It is an intergovernmental organization created in 2006. 2. The Commission has five observers including India and China. 3. It aims for Indian collaboration to develop quality infrastructure in Africa. Which of the above statements is/are correct? (a) 1 and 2 only (b) 2 only (c) 1 and 3 only (d) 1, 2 and 3 Ans: (b) Explanation: About IOC: • It is an intergovernmental organization created in 1982. • It was institutionalized in 1984 by the Victoria Agreement in Seychelles. • It is composed of five African Indian Ocean nations: Comoros, Madagascar, Mauritius, Réunion (an overseas region of France), and Seychelles. • It’s principal mission is to strengthen the ties of friendship between the countries and to be a platform of solidarity for the entire population of the African Indian Ocean region. • The Commission has a Secretariat which is located in Mauritius and headed by a Secretary General. • The Commission has five observers — China, India, EU, Malta and International Organisation of La Francophonie (OIF). Refer: https://www.insightsonindia.com/2021/03/20/the-indian-ocean-border-dispute-between- kenya-and-somalia/ 177. The ‘Global Hunger Index’ (GHI) report is released by which of the following? (a) NGO Welthungerhilfe (b) Amnesty International (c) Walk Free Foundation (d) Both (a) and (c) Ans: (a) Explanation: What is Global Hunger Index? • The report is a peer-reviewed publication released annually by Welthungerhilfe and Concern Worldwide. • It tracks hunger at global, regional and national levels. Refer: https://www.insightsonindia.com/2021/03/20/government-questions-methodology-and-data- accuracy-of-global-hunger-index/ 178. In the context of Indian Oil to launch cost-effective ‘aluminium-air batteries’ for EVs, consider the following statements: 1. The main advantage of the use of aluminium-air batteries is the elimination of carbon monoxide emissions. 2. An aluminum-air battery is considerably lighter than a comparable lithium-ion battery. 3. Aluminium–air batteries are non-rechargeable. Which of the given above statements is/are correct? (a) 1 and 3 only (b) 2 only (c) 2 and 3 only (d) 1, 2 and 3 Ans: (c) Explanation: • S2: Because aluminum is a lightweight metal and the cathode material, oxygen, does not have to be stored in the battery, an aluminum-air battery is considerably lighter than a comparable lithium-ion battery.

Telegram: https://t.me/insightsIAStips 93 Youtube: https://www.youtube.com/channel/UCpoccbCX9GEIwaiIe4HLjwA

Revision Through MCQs (RTM) Compilation (March 2021)

• S3: Aluminium–air batteries are primary cells, i.e., non-rechargeable. Once the aluminium anode is consumed by its reaction with atmospheric oxygen at a cathode immersed in a water-based electrolyte to form hydrated aluminium oxide, the battery will no longer produce electricity. • S1: The total fuel efficiency during the cycle process in Al/air electric vehicles (EVs) can be 15% (present stage) or 20% (projected), comparable to that of internal combustion engine vehicles (ICEs) (13%). S1 is incorrect since it is not possible. Refer: https://www.insightsonindia.com/2021/03/20/what-are-aluminium-air-batteries/ 179. Which of the following types of inventions are not patentable in India? 1. Artificial intelligence led innovations without human intervention. 2. A method of agriculture or horticulture 3. Normal biological processes 4. Inventions relating to atomic energy Select the correct answer code: (a) 1, 2 and 3 only (b) 1, 3 and 4 only (c) 2, 3 and 4 only (d) 1, 2, 3 and 4 Ans: (d) Explanation: What types of inventions are not patentable in India? • An invention may satisfy the condition of novelty, inventiveness and usefulness but it may not qualify for a patent under the following situations: o an invention which is frivolous or which claims anything obviously contrary to well established natural laws; o an invention the primary or intended use or commercial exploitation of which could be contrary to public order or morality or which causes serious prejudice to human, animal or plant life or health or to the environment; o the mere discovery of scientific principle or the formulation of an abstract theory or discovery of any living thing or non-living substance occurring in nature; o the mere discovery of a new form of a known substance which does not result in enhancement of the known efficacy of that substance or the mere discovery of any new property or new use for a known substance or of the mere use of a known process, machine or apparatus unless such known process results in a new product or employs at least one new reactant; • Explanation: For the purposes of this clause, salts, esters, ethers, polymorphs, metabolites, pure form, particle size, isomers, mixtures of isomers, complexes, combinations and other derivatives of known substance shall be considered to be the same substance, unless they differ significantly in properties with regards to efficacy; o a substance obtained by mere admixture resulting only in the aggregation of the properties of the components thereof or a process for producing such substance; o the mere arrangement or re-arrangement or duplication of known devices each functioning independently of one another in a known way; o a method of agriculture or horticulture; o any process for medicinal, surgical, curative, prophylactic (diagnostic, therapeutic) or other treatment of human beings or any process for a similar treatment of animals to render them free of disease or to increase their economic value or that of their products; o plants and animals in whole or any part thereof other than microorganisms but including seeds, varieties and species and essentially biological processes for production or propagation of plants and animals; o a mathematical or business method or a computer program per se or algorithms; o a literary, dramatic, musical or artistic work or any other aesthetic creation whatsoever including cinematographic works and television productions;

Telegram: https://t.me/insightsIAStips 94 Youtube: https://www.youtube.com/channel/UCpoccbCX9GEIwaiIe4HLjwA

Revision Through MCQs (RTM) Compilation (March 2021)

o a mere scheme or rule or method of performing mental act or method of playing game; o a presentation of information; o topography of integrated circuits; o an invention which, in effect, is traditional knowledge or which is an aggregation or duplication of known properties of traditionally known component or components; o inventions relating to atomic energy;

Refer: https://www.insightsonindia.com/2021/03/20/india-japan-agree-for-greater-cooperation-in- patent-verification/ 180. Which of the following schedules of the Constitution have a bearing on the governance of tribals in India? 1. Fifth schedule 2. Sixth Schedule 3. Seventh Schedule 4. Tenth Schedule Select the correct answer code: (a) 1 and 2 only (b) 1, 2 and 3 only (c) 2, 3, 4 only (d) 1, 2, 3 and 4 Ans: (b) Explanation: • 5th and 6th schedule directly concern themselves with tribal governance as all the special provisions made under the constitution fall under it. For e.g. constituting a tribal advisory council; giving tribals control over land and forests etc. • 7th schedule is concerned with the division of legislative power between Centre and States. It concerns with forests; welfare; mining and minerals which have a direct bearing on the tribal community in India. 10th schedule is about anti-defection. • Constitution (Scheduled Castes) Order (Amendment) Bill, 2021: o Passed in Lok Sabha recently. o It seeks to put seven castes under one nomenclature of “Devendrakula Vellalars” with some exceptions for some of the castes in certain districts of Tamil Nadu. o The castes include Devendrakulathan, Kadaiyan, Kalladi, Kudumban, Pallan, Pannadi and Vathiriyan. Refer: facts for prelims: https://www.insightsonindia.com/2021/03/20/insights-daily-current- affairs-pib-summary-20-march-2021/

RTM- REVISION THROUGH MCQS – 22th-Mar-2021

181. With reference to medieval history of India, consider the following statements: 1. The Tomara dynasty ruled parts of present-day Delhi and Haryana in India during 8th-12th century. 2. Prithviraj Chauhan built Lal Kot fort and Anangtal Baoli. Which of the given above statements is/are correct? (a) 1 only (b) 2 only (c) Both 1 and 2 (d) Neither 1 nor 2 Ans: (a)

Telegram: https://t.me/insightsIAStips 95 Youtube: https://www.youtube.com/channel/UCpoccbCX9GEIwaiIe4HLjwA

Revision Through MCQs (RTM) Compilation (March 2021)

Explanation: Who was Anangpal II? • Belonged to the Tomar dynasty that ruled parts of present-day Delhi and Haryana between the 8th and 12th centuries. • Anangpal Tomar II was succeeded by his grandson Prithviraj Chauhan, who was defeated by the Ghurid forces in the Battle of Tarain (present-day Haryana) after which the Delhi Sultanate was established in 1192. • The Tomar dynasty shifted its capital in the 8th century), to Dhillikapuri (Delhi) during the reign of Anangpal II. • He gave Delhi its present name and also repopulated it. • He built Lal Kot fort and Anangtal Baoli. Refer: https://www.insightsonindia.com/2021/03/22/tomar-king-anangpal-ii/ 182. The Supreme Court of India has declared which of the following rights as part of Article 21 (Protection of Life and Personal Liberty)? 1. Right to free legal aid. 2. Right to travel abroad. 3. Right to appropriate life insurance policy. 4. Right to electricity. Select the correct answer using the code below: (a) 1 only (b) 1 and 2 only (c) 1, 2 and 3 only (d) 1, 2, 3 and 4 Ans: (d) Explanation: • The Supreme Court has reaffirmed its judgement in the Menaka case in the subsequent cases. It has declared the following rights as part of Article 21: (1) Right to live with human dignity. (2) Right to decent environment including pollution free water and air and protection against hazardous industries. (3) Right to livelihood. (4) Right to privacy. (5) Right to shelter. (6) Right to health. (7) Right to free education up to 14 years of age. (8) Right to free legal aid. (9) Right against solitary confinement. (10) Right to speedy trial. (11) Right against handcuffing. (12) Right against inhuman treatment. (13) Right against delayed execution. (14) Right to travel abroad. (15) Right against bonded labour. (16) Right against custodial harassment. (17) Right to emergency medical aid. (18) Right to timely medical treatment in government hospital. (19) Right not to be driven out of a state. (20) Right to fair trial. (21) Right of prisoner to have necessities of life. (22) Right of women to be treated with decency and dignity. (23) Right against public hanging. (24) Right to hearing. (25) Right to information. (26) Right to reputation. (27) Right of appeal from a judgement of conviction (28) Right to social security and protection of the family (29) Right to social and economic justice and empowerment (30) Right against bar fetters(31) Right to appropriate life insurance policy (32) Right to sleep (33) Right to freedom from noise pollution (34) Right to electricity. Refer: https://www.insightsonindia.com/2021/03/22/right-to-counsel-in-custody/ 183. Consider the following statements about the Food and Agriculture Organization (FAO): 1. It is a specialized agency of the United Nations that leads international efforts to defeat hunger. 2. One of FAO's strategic goals is the sustainable management of the world's forests. 3. It is headquartered in Paris, France. Which of the given above statements is/are correct? (a) 1 and 2 only (b) 2 and 3 only (c) 1 and 3 only (d) 1, 2 and 3

Telegram: https://t.me/insightsIAStips 96 Youtube: https://www.youtube.com/channel/UCpoccbCX9GEIwaiIe4HLjwA

Revision Through MCQs (RTM) Compilation (March 2021)

Ans: (a) Explanation: About FAO: • It is a specialized agency of the United Nations that leads international efforts to defeat hunger. • Headquarters: Rome, Italy • Founded: 16 October 1945 • Goal of FAO: Their goal is to achieve food security for all and make sure that people have regular access to enough high-quality food to lead active, healthy lives. • Important reports and Programmes (Have a brief overview): o Global Report on Food Crises. o Every two years, FAO publishes the State of the World’s Forests. o FAO and the World Health Organization created the Codex Alimentarius Commission in 1961 to develop food standards, guidelines and texts. o In 1996, FAO organized the World Food Summit. The Summit concluded with the signing of the Rome Declaration, which established the goal of halving the number of people who suffer from hunger by the year 2015. o In 2004 the Right to Food Guidelines were adopted, offering guidance to states on how to implement their obligations on the right to food. o FAO created the International Plant Protection Convention or IPPC in 1952. o FAO is depositary of the International Treaty on Plant Genetic Resources for Food and Agriculture, also called Plant Treaty, Seed Treaty or ITPGRFA, entered into force on 29 June 2004. o The Globally Important Agricultural Heritage Systems (GIAHS) Partnership Initiative was conceptualized in 2002 during World Summit on Sustainable Development in Johannesburg, South Africa. Refer: https://www.insightsonindia.com/2021/03/22/international-day-of-forests/ 184. In which one of the following States is Pin Valley National Park located? (a) Kerala (b) Tamil Nadu (c) Uttarakhand (d) Himachal Pradesh Ans: (d) Explanation: • Pin Valley National Park is a National park of India located in the Spiti Valley in the Lahaul and Spiti district, in the state of Himachal Pradesh. Refer: https://www.insightsonindia.com/2021/03/22/himachal-pradeshs-water-crisis/ 185. Consider the following statements regarding Ken-Betwa river interlinking project. 1. It is the country’s first river interlinking project. 2. Ken and Betwa rivers originate in Madhya Pradesh and are the tributaries of Yamuna. 3. Ken River passes through Panna tiger reserve. Which of the above statements is/are correct? (a) 1 and 2 only (b) 1 and 3 only (c) 2 and 3 only (d) 1, 2 and 3 Ans: (d) Explanation: About Ken- Betwa project: • Conceived as a two-part project, this is the country’s first river interlinking project. • It is perceived as a model plan for similar interstate river transfer missions. • The project aims to transfer surplus water from the Ken river in MP to Betwa in UP to irrigate the drought-prone region spread across the districts of two states mainly Jhansi, Banda, Lalitpur and Mahoba districts of UP and Tikamgarh, Panna and Chhatarpur districts of MP.

Telegram: https://t.me/insightsIAStips 97 Youtube: https://www.youtube.com/channel/UCpoccbCX9GEIwaiIe4HLjwA

Revision Through MCQs (RTM) Compilation (March 2021)

• Key facts: o Ken and Betwa rivers originate in MP and are the tributaries of Yamuna. o Ken meets with Yamuna in Banda district of UP and with Betwa in Hamirpur district of UP. o Rajghat, Paricha and Matatila dams are over Betwa river. o Ken River passes through Panna tiger reserve. Refer: https://www.insightsonindia.com/2021/03/22/ken-betwa-interlinking-project-dam-2/ 186. Consider the following pairs: National Parks/Wild Life Sanctuary State 1. Kuno National Park Rajasthan 2. Bhainsrorgarh Wild Life Sanctuary Madhya Pradesh 3. Gandhi Sagar Wild Life Sanctuary Gujarat Which of the given above pairs is/are not correctly matched? (a) 1 only (b) 1 and 2 (c) 3 only (d) 1, 2 and 3 Ans: (d) Explanation: here the directive word is not correct!! • Gandhi Sagar Wildlife Sanctuary- Madhya Pradesh • Bhainsrorgarh Wildlife Sanctuary- Rajasthan • Kuno National Park- Madhya Pradesh Refer: https://indianexpress.com/article/india/india-imports-african-cheetahs-big-cat-locally- extinction-7237807/ 187. With reference to India’s Madhav National Park, which of the following statements are correct? 1. It is spread over two districts. 2. The National Highway 27 passes through this park. 3. It is one of the natural habitats of Indian leopard. Select the correct answer using the code below: (a) 1 and 2 only (b) 2 and 3 only (c) 1 and 3 only (d) 1, 2 and 3 Ans: (b) Explanation: • S3: The predominant animal species that inhabits the park is the deer, of which the most easily sighted are the graceful little chinkara or Indian gazelle, and the chital. Other species that have their habitat in the park are nilgai, sambar, chausingha or four-horned antelope, blackbuck, sloth bear, Indian leopard and the common langur. • S1 & S2: Madhav National Park is situated in Shivpuri District of Gwalior division in northwest Madhya Pradesh, India. Two national highways pass through the park, the Agra to Bombay former National Highway 3 and the Jhansi to Shivpuri National Highway 27. Refer: https://indianexpress.com/article/india/india-imports-african-cheetahs-big-cat-locally- extinction-7237807/ 188. Consider the following statements: 1. The Asiatic cheetah is classified as a “critically endangered” species by the IUCN Red List and is believed to survive only in Africa. 2. Cheetahs are a keystone species in the savanna. Which of the given above statements is/are correct? (a) 1 only

Telegram: https://t.me/insightsIAStips 98 Youtube: https://www.youtube.com/channel/UCpoccbCX9GEIwaiIe4HLjwA

Revision Through MCQs (RTM) Compilation (March 2021)

(b) 2 only (c) Both 1 and 2 (d) Neither 1 nor 2 Ans: (b) Explanation: • Stat1: The Asiatic cheetah is classified as a “critically endangered” species by the IUCN Red List, and is believed to survive only in Iran. • Stat2: Cheetahs are a keystone species in the savanna. • A keystone species is a species which has a disproportionately large effect on its natural environment relative to its abundance, a concept introduced in 1969 by the zoologist Robert T. Paine. • Such species are described as playing a critical role in maintaining the structure of an ecological community, affecting many other organisms in an ecosystem and helping to determine the types and numbers of various other species in the community. Without keystone species, the ecosystem would be dramatically different or cease to exist altogether. Some keystone species, such as the wolf, are also apex predators. Refer: https://indianexpress.com/article/india/india-imports-african-cheetahs-big-cat-locally- extinction-7237807/ 189. In which one of the following States is Shergarh Wildlife Sanctuary located? (a) Rajasthan (b) Gujarat (c) Madhya Pradesh (d) Punjab Ans: (a) Explanation: • Located in in Rajasthan, Shergarh is situated around 65 km from Baran District. • The sanctuary is known for its rich flora & fauna and amongst its major attractions are tigers, sloth bear, hyenas, deer species like chinkara (Indian gazelle), sambar, chital (spotted deer), and leopards. The place is also considered ideal for wildlife photography and nature photography. • The sanctuary is fringed by Shergarh Fort, an ancient fort that dates back to 790 AD. Every year pilgrims pay homage to this fort as it houses several. Jain and the Brahmanical temples. Refer: https://indianexpress.com/article/india/india-imports-african-cheetahs-big-cat-locally- extinction-7237807/ 190. Consider the following statements about Mukandara Hills National Park: 1. It is located on the eastern bank of the Chambal River. 2. It was once considered for the reintroduction of the Asiatic lion. 3. It was declared one of the Project Tiger reserves in 1973. Which of the given above statements is/are correct? (a) 1 and 2 only (b) 1 and 3 only (c) 2 and 3 only (d) 1, 2 and 3 Ans: (a) Explanation: • Mukundara Hills National Park is mountainous and has a variety of plants, trees and animals. It has grasslands in between and also many dry deciduous trees. There are four rivers that flow in this region, the rivers are Chambal river, Kali river, Ahu river, Ramzan river. • Mukandara Hills National Park was once considered for the reintroduction of the Asiatic lion.

Telegram: https://t.me/insightsIAStips 99 Youtube: https://www.youtube.com/channel/UCpoccbCX9GEIwaiIe4HLjwA

Revision Through MCQs (RTM) Compilation (March 2021)

• It was notified as a Tiger Reserve (2013) under the Wildlife Protection Act, 1972. It is the third park in the country, after Ranthambore and Sariska, where tigers were reintroduced. Refer: https://indianexpress.com/article/india/india-imports-african-cheetahs-big-cat-locally- extinction-7237807/

Telegram: https://t.me/insightsIAStips 100 Youtube: https://www.youtube.com/channel/UCpoccbCX9GEIwaiIe4HLjwA

Revision Through MCQs (RTM) Compilation (March 2021)

RTM- REVISION THROUGH MCQS – 23th-Mar-2021

191. The ‘Vishaka Guidelines’ were mentioned in news sometimes, are related to which of the following? (a) cases of sexual harassment (b) capital punishment (c) securing cyberspace (d) decriminalization of politics Ans: (a) Explanation: • The Vishaka guidelines were a set of guidelines that were intended to protect women at the workplace. These were instituted by the Supreme Court of India in the year 1997. These are procedural in nature and state the method that is to be followed while dealing with cases related to the sexual harassment of women. Refer: https://www.insightsonindia.com/2021/03/23/corrective-voice-from-top-court-against- stereotyping-women/ 192. Consider the following statements: 1. First time use of Electronic Voting Machine (EVMs) occurred in the general election in Kerala in 1982. 2. Voter Verifiable Paper Audit Trail (VVPAT) is an independent system attached with the EVMs that allows the voters to verify that their votes are cast as intended. 3. The election commission has mandated one compulsory verification of slip count with the electronic count for one polling station for every constituency. Which of the given above statements is/are correct? (a) 1 and 2 only (b) 2 and 3 only (c) 1 and 3 only (d) 1, 2 and 3 Ans: (d) Explanation: • S2 & S3: Voter Verifiable Paper Audit Trail is an independent system attached with the Electronic Voting Machines that allows the voters to verify that their votes are cast as intended. When a vote is cast, a slip is printed on the VVPAT printer containing the serial number, name and symbol of the candidate and remains exposed through a transparent window for 7 seconds. Thereafter, this printed slip automatically gets cut and falls in sealed drop box of the VVPAT. o The slips can be counted to verify the electronic result stored in the control unit of the EVM. This can be done under certain circumstances only. If a candidate or their agent requests for recount under section 56 D (4) (b) of the Conduct of Election Rules. The election commission has mandated one compulsory verification of slip count with the electronic count for one polling station for every constituency. • S1: First time use of EVMs occurred in the general election in Kerala in May, 1982; however, the absence of a specific law prescribing its use led to the Supreme Court striking down that election. Subsequently, in 1989, the Parliament amended the Representation of the People Act, 1951 to create a provision for the use of EVMs in the elections (chapter 3). Refer: https://www.insightsonindia.com/2021/03/23/voter-verifiable-paper-audit-trail-vvpat/ 193. Very recently, in which of the following countries having lakhs of people either suffered from severe famine/acute malnutrition or died due to starvation caused by war/ethnic conflicts? (a) Angola and Zambia (b) Morocco and Tunisia

Telegram: https://t.me/insightsIAStips 101 Youtube: https://www.youtube.com/channel/UCpoccbCX9GEIwaiIe4HLjwA

Revision Through MCQs (RTM) Compilation (March 2021)

(c) Venezuela and Colombia (d) Yemen and South Sudan Ans: (d) Explanation: • The world faces one of the largest food crises in 70 years with 20 million people I four countries namely: Yemen, Nigeria, South Sudan and Somalia according to FAO. • How bad is Yemen’s humanitarian situation? o Since the Saudi intervention in 2015, at least 10,000 people have been killed in Yemen, according to the WHO. The widespread damage caused to infrastructure by the coalition airstrikes and lack of supplies of food and medicines due to the blockade have pushed Yemen into a humanitarian catastrophe. o About 12 million people are at the risk of starvation if aid doesn’t reach them fast. The country has also seen a massive cholera outbreak. A child dies every 10 minutes in Yemen from preventable causes, says UNICEF. Refer: https://www.insightsonindia.com/2021/03/23/riyadh-presents-ceasefire-plan-to-yemens- houthis/ 194. With reference to Small Finance Banks in India, consider the following statements: 1. SFBs can provide remittance as well as credit cards. 2. SFBs can accept deposits, but only up to $1 lakh per individual customer. 3. SFBs can lend but the focus will be on small lending. Which of the given above statements is/are correct? (a) 1 and 2 only (b) 2 and 3 only (c) 1 and 3 only (d) 1, 2 and 3 Ans: (c) Explanation: • What are small finance banks? o The small finance bank will primarily undertake basic banking activities of acceptance of deposits and lending to unserved and underserved sections including small business units, small and marginal farmers, micro and small industries and unorganised sector entities. • What they can do? o Take small deposits and disburse loans. o Distribute mutual funds, insurance products and other simple third-party financial products. o Lend 75% of their total adjusted net bank credit to priority sector. o Maximum loan size would be 10% of capital funds to single borrower, 15% to a group. o Minimum 50% of loans should be up to 25 lakhs. • What they cannot do? o Lend to big corporates and groups. o Cannot open branches with prior RBI approval for first five years. o Other financial activities of the promoter must not mingle with the bank. o It cannot set up subsidiaries to undertake non-banking financial services activities. o Cannot be a business correspondent of any bank. • The guidelines they need to follow: o Promoter must contribute minimum 40% equity capital and should be brought down to 30% in 10 years. o Minimum paid-up capital would be Rs 100 cr. o Capital adequacy ratio should be 15% of risk weighted assets, Tier-I should be 7.5%.

Telegram: https://t.me/insightsIAStips 102 Youtube: https://www.youtube.com/channel/UCpoccbCX9GEIwaiIe4HLjwA

Revision Through MCQs (RTM) Compilation (March 2021)

o Foreign shareholding capped at 74% of paid capital, FPIs cannot hold more than 24%. o Priority sector lending requirement of 75% of total adjusted net bank credit. o 50% of loans must be up to Rs 25 lakh.

Refer: https://www.insightsonindia.com/2021/03/23/panel-to-evaluate-applications-for-universal- banks-small-finance-banks/ 195. With reference to Non-Banking Financial Company (NBFC) in India, which of the following statements are correct? 1. NBFCs cannot accept demand deposits 2. NBFCs cannot issue cheques drawn on itself 3. Deposit insurance facility of Deposit Insurance and Credit Guarantee Corporation is not available to depositors of NBFCs Select the correct answer using the code below: (a) 1 and 3 only (b) 2 and 3 only (c) 1 and 2 only (d) 1, 2 and 3 Ans: (d) Explanation: • A Non-Banking Financial Company (NBFC) is a company registered under the Companies Act, 1956 engaged in the business of loans and advances, acquisition of shares/stocks/bonds/debentures/securities issued by Government or local authority or other marketable securities of a like nature, leasing, hire-purchase, insurance business, chit business but does not include any institution whose principal business is that of agriculture activity, industrial activity, purchase or sale of any goods (other than securities) or providing any services and sale/purchase/construction of immovable property. A non-banking institution which is a company and has principal business of receiving deposits under any scheme or arrangement in one lump sum or in installments by way of contributions or in any other manner, is also a non-banking financial company (Residuary non-banking company). • NBFCs lend and make investments and hence their activities are akin to that of banks; however there are a few differences as given below:

Telegram: https://t.me/insightsIAStips 103 Youtube: https://www.youtube.com/channel/UCpoccbCX9GEIwaiIe4HLjwA

Revision Through MCQs (RTM) Compilation (March 2021)

o NBFC cannot accept demand deposits; o NBFCs do not form part of the payment and settlement system and cannot issue cheques drawn on itself; o deposit insurance facility of Deposit Insurance and Credit Guarantee Corporation is not available to depositors of NBFCs, unlike in case of banks. Refer: https://www.insightsonindia.com/2021/03/23/panel-to-evaluate-applications-for-universal- banks-small-finance-banks/ 196. With reference to International Space Station (ISS), consider the following statements: 1. It is in low Earth orbit. 2. It is a multinational collaborative project involving Indian Space Research Organisation (ISRO). 3. It serves as a microgravity and space environment research laboratory. Which of the given above statements is/are correct? (a) 1 and 3 only (b) 1 and 2 only (c) 2 and 3 only (d) 1, 2 and 3 Ans: (a) Explanation: • The International Space Station (ISS) is a modular space station (habitable artificial satellite) in low Earth orbit. It is a multinational collaborative project involving five participating space agencies: NASA (United States), Roscosmos (Russia), JAXA (Japan), ESA (Europe), and CSA (Canada). • The station serves as a microgravity and space environment research laboratory in which scientific research is conducted in astrobiology, astronomy, meteorology, physics, and other fields. • The ISS is suited for testing the spacecraft systems and equipment required for possible future long-duration missions to the Moon and Mars. • Bacteria found on International Space Station named after Indian scientist: o Four species of bacteria have been discovered onboard the International Space Station (ISS), one of which has been named after Indian biodiversity scientist Seyed Ajmal Khan (Methylobacterium ajmalii). o Four strains of bacteria belong to the family o While one strain was identified as Methylorubrum rhodesianum bacteria, the other three strains were previously undiscovered. o These bacteria aid in plant growth. Refer: Facts for Prelims: https://www.insightsonindia.com/2021/03/23/insights-daily-current- affairs-pib-summary-23-march-2021/ 197. Consider the following statements about the Gandhi Peace Prize: 1. It is an annual award instituted by government of India since 1995. 2. The award carries a cash prize of ₹51 lakh and a certificate in case of an institution and ₹5 lakh and a certificate in case of an individual. 3. It is open to all persons regardless of nationality, creed, race or sex. Which of the given above statements is/are correct? (a) 1 and 2 only (b) 2 and 3 only (c) 1 and 3 only (d) 1, 2 and 3 Ans: (c) Explanation: Gandhi Peace Prize:

Telegram: https://t.me/insightsIAStips 104 Youtube: https://www.youtube.com/channel/UCpoccbCX9GEIwaiIe4HLjwA

Revision Through MCQs (RTM) Compilation (March 2021)

• The father of the nation of Bangladesh Sheikh Mujibur Rahman and the former Sultan of Oman, the late Qaboos bin Said Al Said, will be awarded the Gandhi Peace Prize for 2020 and 2019, respectively. • They were selected by the jury, chaired by Prime Minister Narendra Modi and comprising the Chief Justice of India, the leader of the single largest Opposition party in the Lok Sabha, Lok Sabha Speaker and founder of Sulabh International. • About the Gandhi Peace Prize: o Instituted in the year 1995 on the occasion of the 125th birth anniversary of Mahatma Gandhi. o This annual award is given to individuals and institutions for their contributions towards social, economic and political transformation through non-violence and other Gandhian methods. o The award carries a cash prize of Rs 1 crore, a citation and a Plaque as well as an exquisite traditional handicraft/handloom item. o It is open to all persons regardless of nationality, creed, race or sex. Refer: facts for prelims: https://www.insightsonindia.com/2021/03/23/insights-daily-current- affairs-pib-summary-23-march-2021/ 198. Consider the following statements about International Telecommunication Union (ITU): 1. It is a specialized agency of the United Nations. 2. It is one of the oldest international organizations in operation. 3. It is headquartered in Washington D.C., United States of America. Which of the given above statements is/are correct? (a) 1 and 2 only (b) 2 and 3 only (c) 1 and 3 only (d) 1, 2 and 3 Ans: (a) Explanation: • The International Telecommunication Union (ITU) was established to standardize and regulate international radio and telecommunications. It was founded as the International Telegraph Union in Paris on 17 May 1865. Its main tasks include standardization, allocation of the radio spectrum, and organizing interconnection arrangements between different countries to allow international phone calls — in which regard it performs for telecommunications a similar function to what the UPU performs for postal services. It has its headquarters in Geneva, Switzerland, next to the main United Nations campus. • World Summit on Information Society Forum 2021: o It represents one of the world’s largest annual gatherings of the ‘ICT for development’ community. o It is co-organized by the International Telecommunications Union (ITU), UNESCO, UNDP and UNCTAD. o The Forum has proven to be an efficient mechanism for coordination of multi- stakeholder implementation activities, information exchange, creation of knowledge, sharing of best practices. Refer: facts for prelims: https://www.insightsonindia.com/2021/03/23/insights-daily-current- affairs-pib-summary-23-march-2021/ 199. With reference to India’s Ranthambore National Park, which of the following statements are correct? 1. It is located at the junction of the Aravali and Vindhya hill ranges. 2. It is bounded to the north by the Banas River and to the south by the Chambal River. 3. It harbours dry deciduous forests and open grassy meadow. Which of the given above statements is/are correct? (a) 1 and 2 only

Telegram: https://t.me/insightsIAStips 105 Youtube: https://www.youtube.com/channel/UCpoccbCX9GEIwaiIe4HLjwA

Revision Through MCQs (RTM) Compilation (March 2021)

(b) 2 and 3 only (c) 1 and 3 only (d) 1, 2 and 3 Ans: (d) Explanation: • Ranthambore National Park was established initially as Sawai Madhopur Game Sanctuary in 1955 by the Government of India. In 1973, it was declared as one of the Project Tiger reserves in India. It was on 1st November, 1980 that Ranthambore was declared a national park, while the forests located beside it were named Sawai Man Singh Sanctuary & Keladevi Sanctuary. • Located at the junction of the Aravali and Vindhya hill ranges. • It is bounded to the north by the Banas River and to the south by the Chambal River. It is named after the historic Ranthambore Fort, which lies within the park. • Ranthambore National Park harbours dry deciduous forests and open grassy meadow. The flora of the park includes 539 species of flowering plants. Refer: https://www.ranthamborenationalpark.com/ranthambore-national-park-detail.html 200. Consider the following statements with reference to Namdapha National Park: 1. It is a large protected area in Assam of Northeast India. 2. It harbours extensive dipterocarp forests. 3. The Namdapha flying squirrel is endemic to the park and critically endangered. Which of the given above statements is/are correct? (a) 1 and 2 only (b) 2 and 3 only (c) 1 and 3 only (d) 1, 2 and 3 Ans: (b) Explanation: • S1: Namdapha National Park is a large protected area in Arunachal Pradesh of Northeast India. It is a biodiversity hotspot in the Eastern Himalayas. • S2: The national park harbours the northernmost lowland evergreen rainforests in the world at 27°N latitude. It also harbours extensive dipterocarp forests, comprising the northwestern parts of the Mizoram-Manipur-Kachin rain forests ecoregion. It is the fourth largest national park in India. • S3: The Namdapha flying squirrel was first collected in the park and described. It is endemic to the park and critically endangered. It was last recorded in 1981 in a single valley within the park. Refer: https://en.wikipedia.org/wiki/Namdapha_National_Park

Telegram: https://t.me/insightsIAStips 106 Youtube: https://www.youtube.com/channel/UCpoccbCX9GEIwaiIe4HLjwA

Revision Through MCQs (RTM) Compilation (March 2021)

RTM- REVISION THROUGH MCQS – 24th-Mar-2021

201. Consider the following statements: 1. The Part X of the Constitution of India is concerned with the administration of Scheduled Areas and Tribal Areas. 2. The Sixth Schedule consists of provisions for the administration of tribal areas in Assam, Meghalaya, Tripura and Mizoram. Which of the given above statements is/are correct? (a) 1 only (b) 2 only (c) Both 1 and 2 (d) Neither 1 nor 2 Ans: (c) Explanation: • S1: Part 10 of the Indian Constitution entails the provisions related to Scheduled and Tribal Areas with Articles 244 – 244 A. Both the Centre and the State have their roles to play in the administration of the Scheduled areas. • S2: About the Sixth Schedule: • It protects tribal populations and provides autonomy to the communities through creation of autonomous development councils that can frame laws on land, public health, agriculture and others. • As of now, 10 autonomous councils exist in Assam, Meghalaya, Tripura and Mizoram. • This special provision is provided under Article 244(2) and Article 275(1) of the Constitution. • Key provisions: o The governor is empowered to organise and re-organise the autonomous districts. o If there are different tribes in an autonomous district, the governor can divide the district into several autonomous regions. o Composition: Each autonomous district has a district council consisting of 30 members, of whom four are nominated by the governor and the remaining 26 are elected on the basis of adult franchise. o Term: The elected members hold office for a term of five years (unless the council is dissolved earlier) and nominated members hold office during the pleasure of the governor. o Each autonomous region also has a separate regional council. o Powers of councils: The district and regional councils administer the areas under their jurisdiction. They can make laws on certain specified matters like land, forests, canal water, shifting cultivation, village administration, inheritance of property, marriage and divorce, social customs and so on. But all such laws require the assent of the governor. o Village councils: The district and regional councils within their territorial jurisdictions can constitute village councils or courts for trial of suits and cases between the tribes. They hear appeals from them. The jurisdiction of high court over these suits and cases is specified by the governor. Refer: https://www.insightsonindia.com/2021/03/24/sixth-schedule-areas/ 202. With reference to the Citizenship (Amendment) Act, consider the following statements: 1. It prohibits illegal migrants from acquiring Indian citizenship. 2. It provides that the central government may cancel the registration of Overseas Citizenship of India (OCIs) on certain grounds. 3. It will not apply to the areas under the Inner Line Permit under the Bengal Eastern Frontier Regulation, 1873.

Telegram: https://t.me/insightsIAStips 107 Youtube: https://www.youtube.com/channel/UCpoccbCX9GEIwaiIe4HLjwA

Revision Through MCQs (RTM) Compilation (March 2021)

Which of the given above statements is/are correct? (a) 1 and 2 only (b) 2 and 3 only (c) 1 and 3 only (d) 1, 2 and 3 Ans: (b) Explanation: • S1: The Citizenship Act, 1955 prohibits illegal migrants from acquiring Indian citizenship. The Citizenship (Amendment) Act provide that the following minority groups will not be treated as illegal migrants: Hindus, Sikhs, Buddhists, Jains, Parsis and Christians from Afghanistan, Bangladesh and Pakistan. However, to get this benefit, they must have also been exempted from the Foreigners Act, 1946 and the Passport (Entry into India) Act, 1920 by the central government. • S2: The Act provides that the central government may cancel the registration of OCIs on certain grounds. • S3: The Act does not apply to tribal areas of Tripura, Mizoram, Assam and Meghalaya because of being included in the 6th Schedule of the Constitution. o Also areas that fall under the Inner Limit notified under the Bengal Eastern Frontier Regulation, 1873, will also be outside the Act’s purview. Refer: https://www.insightsonindia.com/2021/03/24/citizenship-amendment-act-2019-caa/ 203. Due to slowdown in various sectors, several states are asking for longer period of GST compensation. Which of the following statements is/are correct regarding GST compensation? 1. The Goods and Services Tax (Compensation to States) Act, 2017 provides for 100% compensation to the states for a period of only three years for the loss of revenue arising on account of implementation of GST. 2. For the purpose of calculating the compensation amount payable, the financial year ending 31st March, 2016, shall be taken as the base year. 3. For providing compensation to states, Centre uses the funds specifically collected as compensation cess levied on products considered to be ‘sin’ or luxury goods. Which of the above statements is/are correct? (a) 1 and 2 only (b) 3 only (c) 2 and 3 only (d) 1, 2 and 3 Ans: (c) Explanation: • Goods and Services Tax (Compensation to States) Act, 2017 provide for compensation to the States for the loss of revenue arising on account of implementation of the goods and services tax in pursuance of the provisions of the Constitution (One Hundred and First Amendment) Act, 2016. • Compensation cess was introduced as relief for States for the loss of revenues arising from the implementation of GST. States, in lieu of giving up their powers to collect taxes on goods and services after local levies were subsumed under the GST, were guaranteed a 14 per cent tax revenue growth in the first five years after GST implementation by the Central government. States’ tax revenue as of FY 2016 is considered as the base year for the calculation of this 14 per cent growth. Any shortfall against it is supposed to be compensated by the Centre using the funds specifically collected as compensation cess. • Compensation cess is levied on products considered to be ‘sin’ or luxury goods. Refer: https://www.insightsonindia.com/2021/03/24/gst-compensation-2/ 204. Consider the following statements about the Indus Water Treaty: 1. The treaty was signed by the then Indian Prime Minister Jawaharlal Nehru and Pakistan’s President Ayub Khan in 1956.

Telegram: https://t.me/insightsIAStips 108 Youtube: https://www.youtube.com/channel/UCpoccbCX9GEIwaiIe4HLjwA

Revision Through MCQs (RTM) Compilation (March 2021)

2. It was brokered by the World Bank (International Bank for Reconstruction and Development). 3. The treaty gives control over the waters of the three "eastern rivers" to India. Which of the given above statements is/are correct? (a) 1 and 2 only (b) 2 and 3 only (c) 1 and 3 only (d) 1, 2 and 3 Ans: (b) Explanation: About the Indus Water Treaty: • It is a Water-Distribution Treaty, signed in Karachi on 1960, between India (Pm Jawaharlal Nehru) and Pakistan (President Ayub Khan), brokered by the World Bank. • Under the treaty, India has control over water flowing in the eastern rivers– Beas, Ravi and . • Pakistan has control over the western rivers– Indus, Chenab and Jhelum. • As per the treaty, the water commissioners of Pakistan and India are required to meet twice a year and arrange technical visits to projects’ sites and critical river head works. • Both the sides share details of the water flow and the quantum of water being used under the treaty. Refer: https://www.insightsonindia.com/2021/03/24/indus-water-panel-holds-meeting/

205. Consider the following statements with reference to Indus River: 1. It is the western most River system in the subcontinent. 2. It originates from the Bokhar Chu (glacier) in northern slopes of Mt. Kailash. 3. Jhelum, Chenab, Ravi, Beas and Satluj are its main tributaries. Which of the given above statements is/are correct? (a) 1 and 2 only (b) 2 and 3 only (c) 1 and 3 only (d) 1, 2 and 3 Ans: (d) Explanation: • The Indus is the western most River system in the subcontinent. Jhelum, Chenab, Ravi, Beas and Satluj are its main tributaries. The Indus originates from the Bokhar Chu (glacier) in northern slopes of Mt. Kailash (6714m). more>>

• Refer: https://www.insightsonindia.com/2021/03/24/indus-water-panel-holds-meeting/

Telegram: https://t.me/insightsIAStips 109 Youtube: https://www.youtube.com/channel/UCpoccbCX9GEIwaiIe4HLjwA

Revision Through MCQs (RTM) Compilation (March 2021)

206. Consider the following statements about the North Atlantic Treaty Organization (NATO): 1. It is an intergovernmental military alliance between 30 European and North American countries. 2. It constitutes a system of collective defence whereby its independent member states agree to mutual defence in response to an attack by any external party. 3. It headquarters are located in Washington, D.C.,United States of America. Which of the given above statements is/are correct? (a) 1 and 2 only (b) 2 and 3 only (c) 1 and 3 only (d) 1, 2 and 3 Ans: (a) Explanation: • NATO is an intergovernmental military alliance between 30 European and North American countries. • The organization implements the North Atlantic Treaty that was signed on 4 April 1949. • NATO constitutes a system of collective defence whereby its independent member states agree to mutual defence in response to an attack by any external party. NATO's Headquarters are located in Haren, Brussels, Belgium, while the headquarters of Allied Command Operations is near Mons, Belgium. Refer: https://www.insightsonindia.com/2021/03/24/u-s-peace-plan/ 207. Consider the following statements about the United Nations Human Rights Council (UNHRC): 1. It is a specialised agency of the United Nations. 2. It has more than 50 members elected for staggered three-year terms on a regional group basis. 3. The headquarters of UNHRC is in Geneva, Switzerland. Which of the given above statements is/are correct? (a) 1 and 2 only (b) 2 and 3 only (c) 1 and 3 only (d) 1, 2 and 3 Ans: (c) Explanation: About UNHRC: • UNHRC was reconstituted from its predecessor organisation, the UN Commission on Human Rights to help overcome the “credibility deficit” of the previous organisation. • Headquartered in Geneva, Switzerland. • Composition: o The UNHRC has 47 members serving at any time with elections held to fill up seats every year, based on allocations to regions across the world to ensure geographical representation. o Each elected member serves for a term of three years. o Countries are disallowed from occupying a seat for more than two consecutive terms. • Functions: • The UNHRC passes non-binding resolutions on human rights issues through a periodic review of all 193 UN member states called the Universal Periodic Review (UPR). • It oversees expert investigation of violations in specific countries (Special Procedures). Refer: https://www.insightsonindia.com/2021/03/24/un-human-rights-council-6/ 208. With reference to India’s Nilgiri Biosphere Reserve, which of the following statements are correct? 1. It was the first biosphere reserve in India established in the year 1986. 2. It was declared a World Heritage Site by UNESCO in 2012.

Telegram: https://t.me/insightsIAStips 110 Youtube: https://www.youtube.com/channel/UCpoccbCX9GEIwaiIe4HLjwA

Revision Through MCQs (RTM) Compilation (March 2021)

3. It is located in the Western Ghats. Select the correct answer using the code below: (a) 1 and 2 only (b) 2 and 3 only (c) 1 and 3 only (d) 1, 2 and 3 Ans: (d) Explanation: • The Nilgiri Biosphere Reserve was the first biosphere reserve in India established in the year 1986. It is located in the Western Ghats and includes 2 of the 10 biogeographical provinces of India. Wide ranges of ecosystems and species diversity are found in this region. Thus, it was a natural choice for the premier biosphere reserve of the country. • The Nilgiri Sub-Cluster is a part of the Western Ghats, which was declared a World Heritage Site by UNESCO in 2012. Refer: https://en.wikipedia.org/wiki/Nilgiri_Biosphere_Reserve 209. Consider the following statements about Mukurthi National Park: 1. It is a part of Nilgiri Biosphere Reserve. 2. It is the second largest national park in Kerala. 3. It was created to protect “lion-tailed macaque”, one of the keystone species of this park. Which of the given above statements is/are correct? (a) 1 and 2 only (b) 1 only (c) 2 and 3 only (d) 1, 2 and 3 Ans: (b) Explanation: • Mukurthi National Park (MNP) is located in the northwest corner of Tamil Nadu in the Western Ghats. • It is a part of Nilgiri Biosphere Reserve along with Mudumalai Wildlife Sanctuary, Bandipur National Park, Nagarhole National Park, Wayanad Wildlife Sanctuary and Silent Valley. • Keystone Species: The park was created to protect its keystone species, the Nilgiri Tahr. Refer: https://en.wikipedia.org/wiki/Mukurthi_National_Park#Geography 210. From the ecological point of view, which one of the following assumes importance in being a good link between the Eastern Ghats and the Western Ghats? (a) Sathyamangalam Tiger Reserve (b) Nallamala Forest (c) Nagarhole National Park (d) Seshachalam Biosphere Reserve Ans: (a) Explanation: • Sathyamangalam forest range is a significant wildlife corridor in the Nilgiri Biosphere Reserve between the Western Ghats and the rest of the Eastern Ghats and a genetic link between the four other protected areas which it adjoins, including the Billigiriranga Swamy Temple Wildlife Sanctuary, Sigur Plateau, Mudumalai National Park and Bandipur National Park.

Telegram: https://t.me/insightsIAStips 111 Youtube: https://www.youtube.com/channel/UCpoccbCX9GEIwaiIe4HLjwA

Revision Through MCQs (RTM) Compilation (March 2021)

• Refer: UPSC CSE 2017

RTM- REVISION THROUGH MCQS – 25th-Mar-2021

211. With reference to Attorney General of India, consider the following statements: 1. The Attorney General (AG) is appointed by the president. 2. The Constitution does not contain the procedure and grounds for his removal. 3. The AG’s consent is mandatory when a private citizen wants to initiate a case of contempt of court against a person. Which of the given above statements is/are correct? (a) 1 and 2 only (b) 2 and 3 only (c) 1 and 3 only (d) 1, 2 and 3 Ans: (d) Explanation: • S1: The Attorney General (AG) is appointed by the president. He must be a person who is qualified to be appointed a judge of the Supreme Court. In other words, he must be a citizen of India and he must have been a judge of some high court for five years or an advocate of some high court for ten years or an eminent jurist, in the opinion of the president. • S2: The term of office of the AG is not fixed by the Constitution. Further, the Constitution does not contain the procedure and grounds for his removal. He holds office during the pleasure of the president. This means that he may be removed by the president at any time. He may also quit his office by submitting his resignation to the president. Conventionally, he resigns when the government (council of ministers) resigns or is replaced, as he is appointed on its advice. • S3: The AG’s consent is mandatory when a private citizen wants to initiate a case of contempt of court against a person. o However, when the court itself initiates a contempt of court case the AG’s consent is not required. o This is because the court is exercising its inherent powers under the Constitution to punish for contempt and such Constitutional powers cannot be restricted because the AG declined to grant consent. Refer: https://www.insightsonindia.com/2021/03/25/a-g-says-no-to-contempt-proceedings/

Telegram: https://t.me/insightsIAStips 112 Youtube: https://www.youtube.com/channel/UCpoccbCX9GEIwaiIe4HLjwA

Revision Through MCQs (RTM) Compilation (March 2021)

212. Consider the following statements: 1. The Supreme Court of India was inaugurated on January 28, 1950. 2. The judges of the Supreme Court are appointed by the president. 3. The chief justice is appointed by the outgoing Chief Justice of India on the day of his (or her) retirement. 4. At present, the Supreme Court consists of thirty-one judges. Which of the given above statements is/are correct? (a) 1 and 2 only (b) 1, 2 and 3 only (c) 2 and 3 only (d) 2, 3 and 4 only Ans: (a) Explanation: • S2 & S3: The judges of the Supreme Court are appointed by the president. The chief justice is appointed by the president after consultation with such judges of the Supreme Court and high courts as he deems necessary. The other judges are appointed by president after consultation with the chief justice and such other judges of the Supreme Court and the high courts as he deems necessary. The consultation with the chief justice is obligatory in the case of appointment of a judge other than Chief justice. • S1: The Supreme Court of India was inaugurated on January 28, 1950. It succeeded the Federal Court of India, established under the Government of India Act of 1935. • S4: There are currently 29 judges (including the Chief Justice of India) and maximum possible strength is 34. Refer: https://www.insightsonindia.com/2021/03/25/how-supreme-court-chooses-the-chief-justice- of-india-2/ 213. Predatory pricing policy is designed to (a) drive competitors out of business (b) maximise profits (c) encourage entrants into the market (d) attain least cost output Ans: (a) Explanation: • Predatory Pricing – the pricing of goods or services at such a low level that other firms cannot compete and are forced to leave the market. • Predatory pricing is a short-term strategy, adopted by some of the market giants with deep pockets to sustain short-term losses and reduce the prices of their products below the average variable costs. • This may lead to wiping out competition from the market and could be detrimental to the consumers in the long run. Refer: https://www.insightsonindia.com/2021/03/25/consumer-protection-e-commerce-rules-2020/ 214. With reference to India’s decision to levy an equalization tax of 6% on online advertisement services offered by non-resident entities, which of the following statements is/are correct? 1. It is introduced as a part of the Income Tax Act. 2. Non-resident entities that offer advertisement services in India can claim a tax credit in their home country under the “Double Taxation Avoidance Agreements”. Select the correct answer using the code given below: (a) 1 only (b) 2 only (c) Both 1 and 2 (d) Neither 1 nor 2 Ans: (d) Explanation:

Telegram: https://t.me/insightsIAStips 113 Youtube: https://www.youtube.com/channel/UCpoccbCX9GEIwaiIe4HLjwA

Revision Through MCQs (RTM) Compilation (March 2021)

• S1: It is incorrect. Equalisation Levy is a tax on business transaction for online marketing in which any Indian pays a sum of more than Rs.1 lakh to non-residents entities such Google and Facebook etc. It is part of the Finance act. • S2: It is incorrect. Since Equalisation Levy is outside the scope of tax treaties entered into by India with other countries, the foreign company cannot claim a tax credit in its home country. • The new amendment, effective from April 1, 2020, expands the equalization levy from online advertising to nearly all online commerce activities done in India by businesses that do not have taxable presence in India through applicability of 2% on its revenues. • Specifically, it is levied on consideration receivable by the e-commerce operator for supply or services or facilitation of supply or service to – Person resident in India, Non-resident under specified circumstances such as through sale of data collected from a person resident in India, and Person who buys goods or services through an IP address located in India. Refer: https://www.insightsonindia.com/2021/03/25/no-digital-tax-if-goods-sold-via-india-arm/ 215. In the context of the developments in Bioinformatics, the term ‘transcriptome’, sometimes seen in the news, refers to (a) a range of enzymes used in genome editing (b) the full range of mRNA molecules expressed by an organism (c) the description of the mechanism of gene expression (d) a mechanism of genetic mutations taking place in cells Ans: (b) Explanation: • A transcriptome is the full range of messenger RNA, or mRNA, molecules expressed by an organism. • The term “transcriptome” can also be used to describe the array of mRNA transcripts produced in a particular cell or tissue type. • In contrast with the genome, which is characterized by its stability, the transcriptome actively changes. • In fact, an organism’s transcriptome varies depending on many factors, including stage of development and environmental conditions. • http://www.nature.com/scitable/definition/transcriptome-296 Refer: https://www.insightsonindia.com/2021/03/25/double-mutant-virus-variant-found/ 216. With reference to Energy Efficiency Services Limited (EESL), consider the following statements: 1. It was formed under India's Ministry of New and Renewable Energy to facilitate energy efficiency projects. 2. It is the world's largest public Energy service company (ESCO). Which of the given above statements is/are correct? (a) 1 only (b) 2 only (c) Both 1 and 2 (d) Neither 1 nor 2 Ans: (b) Explanation: • Energy Efficiency Services Limited (EESL) is an energy service company (ESCO) of the Government of India and is the world's largest public ESCO. • It is 100% government-owned, a joint venture of state-owned NTPC Limited, Power Finance Corporation, REC Limited and POWERGRID. • EESL was formed under India's Ministry of Power to facilitate energy efficiency projects. Innovative business and implementation models can significantly reduce consumption and costs. EESL also acts as the resource center for capacity building of state electricity distribution companies, electricity regulatory commissions (ERCs), state-designated agencies (SDAs), upcoming ESCOs, financial institutions, etc.

Telegram: https://t.me/insightsIAStips 114 Youtube: https://www.youtube.com/channel/UCpoccbCX9GEIwaiIe4HLjwA

Revision Through MCQs (RTM) Compilation (March 2021)

• Gram Ujala: Under this program Convergence Energy Services Limited (CESL), a wholly owned subsidiary of Energy Efficiency Services Limited (EESL), will distribute high quality LED bulbs, at an affordable cost of 10 rupees per bulb in rural areas. Refer: https://www.insightsonindia.com/2021/03/25/gram-ujala/ 217. The Pakal Dul Hydro Electric Project is proposed on the: (a) Marusudar river (b) Chenab river (c) Tawi river (d) Jhelum river Ans: (a) Explanation: • The Pakal Dul Hydro Electric Project (1,000 MW) is proposed on the Marusudar river, a tributary of the Chenab river, in Kishtwar district in Jammu and Kashmir.

• Refer: Facts for Prelims: https://www.insightsonindia.com/2021/03/25/insights-daily-current- affairs-pib-summary-25-march-2021/ 218. With reference to India’s Dachigam National Park, which of the following statements are correct? 1. It is one of the natural habitats of Hangul. 2. It is the only national park in India that is north of the Himalayas. 3. There is no metalled road in Dachigam National Park. Which of the given above statements is/are correct? (a) 1 only (b) 2 and 3 only (c) 1 and 2 only (d) 1, 2 and 3 Ans: (a) Explanation: • Dachigam National park is located in the Zabarwan Range of the western Himalayas. It was initially created to ensure clean drinking water supply for the city of Srinagar. It was upgraded and declared a National Park in the year 1981. • It is popular as the home of the rare and critically endangered Hangul or Kashmir stag. • S2 and S3: are related to Hemis National Park. Refer: https://en.wikipedia.org/wiki/Dachigam_National_Park 219. Consider the following statements with respect to Hemis national park: 1. It is the only national park in India that is north of the Himalayas. 2. It lies on the bank of river Indus. 3. The park contains metalled or motorable roads. Select the correct answer using the code below:

Telegram: https://t.me/insightsIAStips 115 Youtube: https://www.youtube.com/channel/UCpoccbCX9GEIwaiIe4HLjwA

Revision Through MCQs (RTM) Compilation (March 2021)

(a) 1 and 3 only (b) 1, 2 and 3 (c) 1 and 2 only (d) 2 and 3 only Ans: (c) Explanation: • It is a high altitude national park in Ladakh, India. Globally famous for its snow leopards, it is believed to have the highest density of them in any protected area in the world. • It is the only national park in India that is north of the Himalayas, the largest notified protected area in India (largest National park) and is the second largest contiguous protected area, after the Nanda Devi Biosphere Reserve and surrounding protected areas. • The park is bounded on the north by the banks of the Indus River. • No metalled or motorable roads traverse the park. Refer: https://en.wikipedia.org/wiki/Hemis_National_Park 220. ‘Afloat on the waters of the Loktak Lake in Manipur, this park is the world’s only floating wildlife sanctuary. Known for the patches and rings of biomass called phumdis, the park is a wetland ecosystem’. The above given passage describes which of the following national park or wildlife sanctuary? (a) Sirohi National Park (b) Bunning Wildlife Sanctuary (c) Zeilad Lake Sanctuary (d) Keibul-Lamjao National Park Ans: (d) Explanation: • The Keibul Lamjao National Park is a national park in the Bishnupur district of the state of Manipur in India. • It is the only floating park in the world, located in North East India, and an integral part of Loktak Lake. • The national park is characterized by floating decomposed plant material locally called phumdi. It was created in 1966 as a wildlife sanctuary to preserve the natural habitat of the endangered Eld's deer (Cervus eldi eldi). In 1977, it was gazetted as national park. • It is also the world’s sole habitat for the sangai, or brow-antlered deer, in the wild. It’s the state animal of Manipur. Refer: https://en.wikipedia.org/wiki/Keibul_Lamjao_National_Park

RTM- REVISION THROUGH MCQS – 26th-Mar-2021

221. With reference to parliamentary proceedings in India, the power of adjournment as well as adjournment sine die lies with the: (a) Speaker of the Lok Sabha (b) Chairman of the Rajya Sabha (c) President of India (d) Either A or B in their respective houses Ans: (d) Explanation: • A sitting of Parliament can be terminated by adjournment or adjournment sine die or prorogation or dissolution (in the case of the Lok Sabha). • Adjournment: It suspends the work in a sitting for a specified time, which may be hours, days or weeks.

Telegram: https://t.me/insightsIAStips 116 Youtube: https://www.youtube.com/channel/UCpoccbCX9GEIwaiIe4HLjwA

Revision Through MCQs (RTM) Compilation (March 2021)

• Adjournment sine die: It means terminating a sitting of Parliament for an indefinite period. • In other words, when the House is adjourned without naming a day for reassembly. • The power of adjournment as well as adjournment sine die lies with the presiding officer (Speaker or Chairman) of the House. Refer: https://www.insightsonindia.com/2021/03/26/termination-of-session-2/ 222. With reference to parliament of India, consider the following statements: 1. The President can prorogue the House while in session. 2. Only the Lok Sabha is subject to dissolution. 3. The Lower House can be dissolved earlier by the President on the advice of the Prime Minister. Which of the given above statements is/are correct? (a) 1 and 2 only (b) 2 and 3 only (c) 1 and 3 only (d) 1, 2 and 3 Ans: (d) Explanation: • In India, the Lok Sabha has a five-year term, but can be dissolved earlier. According to Article 83(2) of the Constitution, completion of five years from the first day of its meeting amounts to dissolution of the Lower House. In this case, an election is held to elect the new Members of Parliament. The Lower House can also be dissolved earlier by the President on the advice of the Prime Minister. It can also be dissolved if the President feels that no viable government can be formed after the resignation or fall of a regime. • Prorogation: The President issues a notification for prorogation of the session after the business of a session is completed and the presiding officer declares the House adjourned sine die. The President can also prorogue the House while in session. Refer: https://www.insightsonindia.com/2021/03/26/termination-of-session-2/ 223. Consider the following statements: 1. Article 128 of the Constitution of India talks about “attendance of a retired judge” as the judge of the Supreme Court. 2. Under Article 224A of the Constitution of India, even retired high court judges can be appointed as ad-hoc judges to high courts. Which if the given above statements is/are correct? (a) 1 only (b) 2 only (c) Both 1 and 2 (d) Neither 1 nor 2 Ans: (c) Explanation: • S1: Article 128 talks about “attendance of a retired judge” as the judge of the Supreme Court. o It states that the Chief Justice of India at any time, with the previous consent of the President, may request any person who has held the office of a judge of the Supreme Court or the high court to sit and act as a judge of the Supreme Court. • S2: The appointment of ad-hoc judges was provided for in the Constitution under Article 224A. o Procedure to be followed: ▪ Under the Article, the Chief Justice of a High Court for any State may at any time, with the previous consent of the President, request any person who has held the office of judge of that court or of any other High Court to sit and act as a judge of the High Court for that State.

Telegram: https://t.me/insightsIAStips 117 Youtube: https://www.youtube.com/channel/UCpoccbCX9GEIwaiIe4HLjwA

Revision Through MCQs (RTM) Compilation (March 2021)

▪ Such a judge is entitled to such allowances as the president may determine. He will also enjoy all the jurisdiction, powers and privileges of a judge of the Supreme Court. But, he will not otherwise be deemed to be a judge of the Supreme Court. Refer: https://www.insightsonindia.com/2021/03/26/sc-suggests-posting-retired-judges-to-clear- backlog-in-hcs/ 224. The Representation of the People Act, 1951 deals with which of the following matters? 1. Registration of Political parties 2. Corrupt practices and Electoral offences 3. Election Disputes 4. By-elections Select the correct answer using the code below: (a) 1 , 2 and 3 only (b) 2, 3 and 4 only (c) 1, 2 and 4 only (d) 1, 2, 3 and 4 Ans: (d) Explanation: Representation of the People Act, 1951: • This act provides for the actual conduct of elections in India. It deals with the following matters: o Details like Qualification and Disqualification of members of both the Houses of Parliament and the State Legislatures, o Administrative machinery for conducting elections, o Registration of Political parties, o Conduct of Elections, o Election Disputes, o Corrupt practices & Electoral offences, & o By-elections. Refer: https://www.insightsonindia.com/2021/03/26/what-are-postal-ballots/ 225. The causative agent for Covid19 is the SARS-CoV-2 virus, it is a/an/the: (a) DNA virus (b) RNA virus (c) Influenza A virus (d) Megavirus Ans: (b) Explanation: How RT-PCR is used for detecting Covid-19? • The causative agent for Covid19 is the SARS-CoV-2 virus. It is an RNA virus, that means it infiltrates a healthy cell to multiply and survive. • Thus, the RT-PCR test is for the identification of SARS-CoV-2 RNA. In this, the RNA is converted to DNA through a process called ‘reverse transcription’ for detecting viruses. • How it is carried out? o The SARS-CoV-2 RNA is generally detectable in respiratory specimens during the acute phase of infection. o For that upper and lower respiratory specimens (such as nasal, nasopharyngeal) are collected. o This sample is treated with several chemical solutions that remove substances, such as proteins and fats, and extracts only the RNA present in the sample. o Real-time RT-PCR setup usually goes through 35 cycles, which means that by the end of the process, around 35 billion new copies of the sections of viral DNA are created from each strand of the virus present in the sample. o As new copies of the viral DNA sections are built, the marker labels attach to the DNA strands and then release a fluorescent dye, which is measured by the machine’s computer and presented in real-time on the screen. The computer

Telegram: https://t.me/insightsIAStips 118 Youtube: https://www.youtube.com/channel/UCpoccbCX9GEIwaiIe4HLjwA

Revision Through MCQs (RTM) Compilation (March 2021)

tracks the amount of fluorescence in the sample after each cycle. When the amount goes over a certain level of fluorescence, this confirms that the virus is present. Refer: https://www.insightsonindia.com/2021/03/26/rt-pcr/ 226. Consider the following statements: 1. DNA contains the sugar deoxyribose, while RNA contains the sugar ribose. 2. DNA is a double-stranded molecule, while RNA is a single-stranded molecule. 3. DNA is not stable under alkaline conditions, while RNA is stable. Which of the given above statements is/are correct? (a) 1 and 2 only (b) 2 Only (c) 1 and 3 only (d) 1, 2 and 3 Ans: (a) Explanation: • DNA stands for deoxyribonucleic acid, while RNA is ribonucleic acid. Although DNA and RNA both carry genetic information, there are quite a few differences between them. • This is a comparison of the differences between DNA versus RNA, including a quick summary and a detailed table of the differences. o DNA contains the sugar deoxyribose, while RNA contains the sugar ribose. The only difference between ribose and deoxyribose is that ribose has one more - OH group than deoxyribose, which has -H attached to the second (2′) carbon in the ring. o DNA is a double-stranded molecule, while RNA is a single-stranded molecule. o DNA is stable under alkaline conditions, while RNA is not stable. o DNA and RNA perform different functions in humans. DNA is responsible for storing and transferring genetic information, while RNA directly codes for amino acids and acts as a messenger between DNA and ribosomes to make proteins. o DNA and RNA base pairing is slightly different since DNA uses the bases adenine, thymine, cytosine, and guanine; RNA uses adenine, uracil, cytosine, and guanine Refer: https://www.insightsonindia.com/2021/03/26/rt-pcr/ 227. Consider the following statements: 1. A cess is levied on the tax payable and not on the taxable income. 2. A cess can be levied on both direct and indirect taxes. Which of the given above statements is/are correct? (a) 1 only (b) 2 only (c) Both 1 and 2 (d) Neither 1 nor 2 Ans: (c) Explanation: What is a cess? • A cess is levied on the tax payable and not on the taxable income. In a sense, for the taxpayer, it is equivalent to a surcharge on tax. • A cess can be levied on both direct and indirect taxes. The revenue obtained from income tax, corporation tax, and indirect taxes can be allocated for various purposes. • The proceeds of all taxes and cesses are credited in the Consolidated Fund of India (CFI), an account of the Government of India. Refer: https://www.insightsonindia.com/2021/03/26/state-welfare-boards-for-building-and-other- construction-workers-bocw/ 228. Arrange the following geographical locations in the direction of North to South: 1. Paracel Islands

Telegram: https://t.me/insightsIAStips 119 Youtube: https://www.youtube.com/channel/UCpoccbCX9GEIwaiIe4HLjwA

Revision Through MCQs (RTM) Compilation (March 2021)

2. Spratly Islands 3. Natuna Islands Select the correct answer using the code below: (a) 1-2-3 (b) 2-3-1 (c) 3-1-2 (d) 3-2-1 Ans: (a) Explanation:

• Refer: https://www.insightsonindia.com/2021/03/26/south-china-sea-2/

229. Consider the following statements regarding the Isthmus of Suez in eastern Egypt: 1. It separates the Mediterranean Sea and the Red Sea. 2. It connects the African continent and Asia. Which of the given above statements is/are correct? (a) 1 only (b) 2 only (c) Both 1 and 2 (d) Neither 1 nor 2 Ans: (c) Explanation: • An isthmus is a narrow strip of land that connects two larger landmasses and separates two bodies of water. • The Isthmus of Suez in eastern Egypt connects the continents of Africa and Asia, and separates the Mediterranean and Red Seas.

• Refer: facts for prelims: https://www.insightsonindia.com/2021/03/26/insights-daily-current- affairs-pib-summary-26-march-2021/

Telegram: https://t.me/insightsIAStips 120 Youtube: https://www.youtube.com/channel/UCpoccbCX9GEIwaiIe4HLjwA

Revision Through MCQs (RTM) Compilation (March 2021)

230. The ‘Central Scrutiny Centre’ (CSC) was in news recently, is an initiative of the: (a) Reserve Bank of India (b) Ministry of Finance (c) Securities and Exchange Board of India (d) Ministry of Corporate Affairs Ans: (d) Explanation: Central Scrutiny Centre (CSC): • It is an initiative of the Corporate Affairs Ministry to scrutinise the filings by users under straight through processes. • The objective is to ensure that data quality is uncompromised and free from flaws. • CSC will primarily scrutinise the filings made by users under straight through processes, identify data quality issues and irregularities, and communicate the same to the concerned Registrar of Companies so that corrective steps can be taken to restore authenticity and correctness of data and it can be seamlessly shared with other regulators, if required. Refer: facts for prelims: https://www.insightsonindia.com/2021/03/26/insights-daily-current- affairs-pib-summary-26-march-2021/

RTM- REVISION THROUGH MCQS – 27th-Mar-2021

231. The Battle of Saraighat was fought between: (a) Mughals and Ahoms (b) Marathas and Mughals (c) English East India Company and Marathas (d) None of the above Ans: (a) Explanation: • Who was Lachit Borphukan? o He was a commander in the Ahom kingdom. o Known for his leadership in the 1671 Battle of Saraighat that thwarted a drawn- out attempt by Mughal forces under the command of Ramsingh I to take over Ahom kingdom. o The battle of Saraighat was fought on the banks of the Brahmaputra in Guwahati. o The National Defence Academy (NDA), ever since 1999 has been conferring the best passing out cadet with the Lachit Borphukan gold medal. • Background: o During the last phase of the Battle of Saraighat, when the Mughals attacked the Assamese forces through the river in Saraighat, many Assamese soldiers began losing their will to fight. It was Lachit’s clarion call to all the soldiers that made them fight till their last breath, ultimately resulting in the defeat of the Mughals. Refer: https://www.insightsonindia.com/2021/03/27/lachit-borphukan-2/ 232. With reference to Electoral Bonds Scheme, consider the following statements: 1. Electoral bonds are interest-free bearer instruments used to donate money anonymously to political parties. 2. There is no limit on the number of bonds an individual or company can purchase. 3. If a party hasn’t enchased any bonds within 15 days, State Bank of India can deposits these into the Prime Minister’s Relief Fund. Which of the given above statements is/are correct?

Telegram: https://t.me/insightsIAStips 121 Youtube: https://www.youtube.com/channel/UCpoccbCX9GEIwaiIe4HLjwA

Revision Through MCQs (RTM) Compilation (March 2021)

(a) 1 and 3 only (b) 2 and 3 only (c) 1 and 2 only (d) 1, 2 and 3 Ans: (d) Explanation: What are electoral bonds? • Announced in the 2017 Union Budget, electoral bonds are interest-free bearer instruments used to donate money anonymously to political parties. • A bearer instrument does not carry any information about the buyer or payee. • The holder of the instrument (which is the political party) is presumed to be its owner. • The bonds are sold in multiples of Rs 1,000, Rs 10,000, Rs 1 lakh, Rs 10 lakh, and Rs 1 crore, and State Bank of India is the only bank authorised to sell them. • Donors can buy and subsequently donate bonds to a political party, which can encash the bonds through its verified account within 15 days. • There is no limit on the number of bonds an individual or company can purchase. • If a party hasn’t enchased any bonds within 15 days, SBI deposits these into the Prime Minister’s Relief Fund. Refer: https://www.insightsonindia.com/2021/03/27/how-the-electoral-bonds-scheme-has-worked- so-far-and-why-it-has-been-challenged-in-sc/ 233. Consider the following places and the features associated with them. 1. Khajuraho A. Chaitya Hall 2. Sanchi B. The Stupa 3. Karle C. Kandariya Temple 4. Deogarh D. Dashavatara Temple Select the correct answer using the code below: (a) 1C, 2B, 3A, 4D (b) 1D, 2C, 3B, 4A (c) 1A, 2D, 3C, 4B (d) 1B, 2C, 3D, 4A Ans: (a) Explanation: • The Kandariya Mahadeva Temple, one of the best examples of temples preserved from the medieval period in India, is the largest of the western group of temples in the Khajuraho complex which was built by the Chandela rulers. • Karla Cells are a complex of ancient Indian Buddhist rock-cut cave shrines located in Maharashtra. The caves house a Buddhist monastery dating back to the 2nd century BC. • The Dashavatara temple is one of the earliest Hindu stone temples still surviving today. Vishnu Temple shows the ornate beauty seen in Gupta style architecture. Refer: https://www.insightsonindia.com/2021/03/27/development-of-iconic-tourist-destinations- scheme/ 234. Consider the following statements about Khajuraho temples. 1. They were developed by Chandela rulers. 2. The temples were made of sandstone. 3. Khajuraho temples has the influence of Dravidian style of temple architecture. Which of the above statements is/are correct? (a) 1 and 3 only (b) 2 and 3 only (c) 1 and 2 only (d) 1, 2 and 3 Ans: (c) Explanation:

Telegram: https://t.me/insightsIAStips 122 Youtube: https://www.youtube.com/channel/UCpoccbCX9GEIwaiIe4HLjwA

Revision Through MCQs (RTM) Compilation (March 2021)

• In the central India, the Chandela rulers developed a distinct style of temple making of their own – known as Khajuraho school. The temples are famous for their nagara-style architectural symbolism and their erotic sculptures. • In these temples, both the interior and exterior walls were lavishly decorated with carvings. • The temples were made of sandstone. • Panchayatan style of temple making was followed. The temples were built on relatively high platform and belong to Hindu as well as Jain religion. Refer: https://www.insightsonindia.com/2021/03/27/development-of-iconic-tourist-destinations- scheme/ 235. Consider the following statements regarding NISAR mission: 1. It is a joint project between NASA and ISRO. 2. It is primarily a technology demonstration mission, designed to study ocean circulation and sea surface elevation. Which of the given above statements is/are correct? (a) 1 only (b) 2 only (c) Both 1 and 2 (d) Neither 1 nor 2 Ans: (a) Explanation: • About NISAR: o The satellite will be launched in 2022 from the Satish Dhawan Space Center in Sriharikota, India, into a near-polar orbit. o It will scan the globe every 12 days over the course of its three-year mission of imaging the Earth’s land, ice sheets and sea ice to give an “unprecedented” view of the planet. o It will detect movements of the planet’s surface as small as 0.4 inches over areas about half the size of a tennis court. o NASA will provide one of the radars for the satellite, a high-rate communication subsystem for science data, GPS receivers and a payload data subsystem. o ISRO will provide the spacecraft bus, the second type of radar (called the S-band radar), the launch vehicle and associated launch services. o NISAR will be equipped with the largest reflector antenna ever launched by NASA and its primary goals include tracking subtle changes in the Earth’s surface, spotting warning signs of imminent volcanic eruptions, helping to monitor groundwater supplies and tracking the rate at which ice sheets are melting. • Synthetic aperture radar: o The name NISAR is short for NASA-ISRO-SAR. SAR here refers to the synthetic aperture radar that NASA will use to measure changes in the surface of the Earth. o Essentially, SAR refers to a technique for producing high-resolution images. Because of the precision, the radar can penetrate clouds and darkness, which means that it can collect data day and night in any weather. Refer: https://www.insightsonindia.com/2021/03/27/what-is-nisar-the-joint-earth-observing- mission-of-nasa-and-isro/ 236. The terms ‘Ethereum, Stellar, Cardano’ sometimes mentioned news recently are related to (a) Exoplanets (b) Crypto currency (c) Cyber attacks (d) Mini satellites Ans: (b) Explanation: What are Cryptocurrencies?

Telegram: https://t.me/insightsIAStips 123 Youtube: https://www.youtube.com/channel/UCpoccbCX9GEIwaiIe4HLjwA

Revision Through MCQs (RTM) Compilation (March 2021)

• Cryptocurrencies are digital currencies in which encryption techniques are used to regulate the generation of units of currency and verify the transfer of funds, operating independently of a central bank. • Examples: Bitcoin, Ethereum etc. Refer: https://www.insightsonindia.com/2021/03/27/why-centre-wants-cryptocurrency-holdings- mandatorily-disclosed-in-roc-filings/ 237. In which one of the following States is Satkosia Tiger Reserve located? (a) Jharkhand (b) Chhattisgarh (c) Odisha (d) Madhya Pradesh Ans: (c) Explanation: What is the Satkosia Tiger Reserve and why was it chosen? • Located in Odisha. • Satkosia falls under reserves where “there is a potential for increasing tiger populations”. • Declared as a Tiger Reserve in 2007, Satkosia had a population of 12 tigers then. The numbers reduced to two in 2018. • The purpose of the relocation was to repopulate tigers in the reserve areas. Refer: https://www.insightsonindia.com/2021/03/27/what-was-the-tiger-relocation-project/ 238. Consider the following statements: 1. India’s first inter-state tiger translocation project was initiated in 2008. 2. Kanha National Park is nestled in the Maikal range of Satpuras in Madhya Pradesh. Which of the given above statements is/are correct? (a) 1 only (b) 2 only (c) Both 1 and 2 (d) Neither 1 nor 2 Ans: (b) Explanation: • India’s first inter-state tiger translocation project was initiated in 2018 wherein two big cats, a male (Mahavir) from Kanha Tiger Reserve and a female (Sundari) from Bandhavgarh from Madhya Pradesh were relocated to Satkosia Tiger Reserve in Odisha, to shore up the tiger population in the state. • Kanha National Park is nestled in the Maikal range of Satpuras in Madhya Pradesh, the heart of India that forms the central Indian highlands. Refer: https://www.insightsonindia.com/2021/03/27/what-was-the-tiger-relocation-project/ 239. The Shigmo is a spring festival celebrated in the Indian state of: (a) Karnataka (b) Maharashtra (c) Gujarat (d) Goa Ans: (d) Explanation: • Shigmo is the celebration of a ‘rich, golden harvest of paddy’ by the tribal communities of Goa. It is a vibrant celebration full of colour, song and dance rooted in Goan culture and traditions. • Agricultural communities including the Kunbis, Gawdas and Velips celebrate the festival that also marks the onset of spring. • Folk dances like Ghodemodini (a dance of equestrian warriors), Gopha and Phugadi are among the many dances performed by the participating communities. Refer: facts for prelims: https://www.insightsonindia.com/2021/03/27/insights-daily-current- affairs-pib-summary-27-march-2021/

Telegram: https://t.me/insightsIAStips 124 Youtube: https://www.youtube.com/channel/UCpoccbCX9GEIwaiIe4HLjwA

Revision Through MCQs (RTM) Compilation (March 2021)

240. Consider the following statements with reference to Kaziranga National Park: 1. It hosts two-thirds of the world's great one-horned rhinoceroses. 2. It is home to the highest density of tigers among protected areas in India. 3. Two of the largest snakes in the world, as well as the longest venomous snake in the world, inhabit this park. Which of the given above statements is/are correct? (a) 1 and 2 only (b) 2 and 3 only (c) 1 and 3 only (d) 1, 2 and 3 Ans: (c) Explanation: • S1: The sanctuary, which hosts two-thirds of the world's great one-horned rhinoceroses, is a World Heritage Site. • S2: Uttarakhand's Corbett Tiger Reserve (CTR) has reported the highest tiger density among India's 50 reserves with 14 tigers per 100 sq km, followed by Kaziranga, Nagarhole and Orang tiger reserves, according to the 656-page report 'Status of Tigers Co-predators and Prey in India', released by Union environment minister. • S3: Two of the largest snakes in the world, the reticulated python and rock python, as well as the longest venomous snake in the world, the king cobra, inhabit the park. Other snakes found here include the Indian cobra, monocled cobra, Russell's viper, and the common krait. Refer: https://indianexpress.com/article/explained/explained-why-political-parties-are-invoking- rhinos-in-assam-7247802/

Telegram: https://t.me/insightsIAStips 125 Youtube: https://www.youtube.com/channel/UCpoccbCX9GEIwaiIe4HLjwA

Revision Through MCQs (RTM) Compilation (March 2021)

RTM- REVISION THROUGH MCQS – 30th-Mar-2021

241. Consider the following statements: 1. The happiness curriculum was first introduced by the Delhi government in 2018. 2. The curriculum is primarily based on the concept of the “Nai Talim” proposed by Mahatma Gandhi. Which of the given above statements is/are correct? (a) 1 only (b) 2 only (c) Both 1 and 2 (d) Neither 1 nor 2 Ans: (a) Explanation: What is Happiness Curriculum? • The happiness curriculum was first introduced by the Delhi government in 2018. • The curriculum calls for schools to promote development in cognition, language, literacy, numeracy and the arts along with addressing the wellbeing and happiness of students. • Framework: The curriculum is based on philosophies of thinkers and educationists such as Mahatma Gandhi, Rabindranath Tagore and Jiddu Krishnamurthy. It is primarily based on the concept of the "Happiness Triad" proposed by philosopher Agrahar Nagraj Sarman, according to which there are three components of happiness – momentary happiness, long term happiness and sustainable happiness. It is also based on Nagraj's concept of "coexistential thought" (Madhyasth Darshan), which is about simplicity, generosity, kindness and benevolence in the pursuit of happiness. The curriculum also includes concepts about self and relationships with family, society and nature. Refer: https://www.insightsonindia.com/2021/03/30/happiness-curriculum/ 242. Consider the following statements: 1. The Digital India Land Record Modernization Programme (DILRMP) was launched in 2015 by the Government of India. 2. The Unique Land Parcel Identification Number (ULPIN) System contains 14 digits - Alpha- numeric unique ID for each land parcel surveyed in India. Which of the given above statements is/are correct? (a) 1 only (b) 2 only (c) Both 1 and 2 (d) Neither 1 nor 2 Ans: (b) Explanation: • The Digital India Land Record Modernization Programme (DILRMP), previously known as the National Land Record Modernization Programme (NLRMP), was launched in 2008 by the Government of India with the purpose to digitize and modernize land records and develop a centralised land record management system. • Unique Land Parcel Identification Number (ULPIN) scheme: o Under the scheme, a 14-digit identification number will be issued to every plot of land in the country. o It is being described as “the Aadhaar for land” — a number that would uniquely identify every surveyed parcel of land and prevent land fraud, especially in rural India, where land records are outdated and disputed. o The identification will be based on the longitude and latitude of the land parcel, and is dependent on detailed surveys and geo-referenced cadastral maps. Refer: https://www.insightsonindia.com/2021/03/30/unique-land-parcel-identification-number- ulpin-scheme/

Telegram: https://t.me/insightsIAStips 126 Youtube: https://www.youtube.com/channel/UCpoccbCX9GEIwaiIe4HLjwA

Revision Through MCQs (RTM) Compilation (March 2021)

243. With reference to National Register of Citizens (NRC), consider the following statements: 1. It is an official record of all genuine Indian citizens. 2. The register was first prepared after the 1951 Census of India and since then it has been updated regularly. 3. At present, only Nagaland has such a register. Select the correct answer using the code below: (a) 1 and 3 only (b) 2 only (c) 2 and 3 only (d) 1 only Ans: (d) Explanation: About NRC: • At its core, the NRC is an official record of those who are legal Indian citizens. It includes demographic information about all those individuals who qualify as citizens of India as per the Citizenship Act, 1955. • The register was first prepared after the 1951 Census of India and since then it has not been updated until recently. • So far, such a database has only been maintained for the state of Assam. Refer: https://www.insightsonindia.com/2021/03/30/nrc-rejection-slips-to-be-issued-soon/ 244. Consider the following statements: 1. Jet Streams occur in the Northern Hemisphere only. 2. Only some cyclones develop an eye. 3. The temperature inside the eye of a cyclone is nearly 10 degree Celsius lesser than that of the surroundings. Which of the statements given above is/are correct? (a) 1 only (b) 2 and 3 only (c) 2 only (d) 1 and 3 only Ans: (c) Explanation: • S1: They occur in upper atmospheres of both hemispheres. • S2: Correct. Extra-tropical cyclones may not always have an eye, whereas mostly mature storms have well developed eye. Rapidly intensifying storms may develop an extremely small, clear, and circular eye, sometimes referred to as a pinhole eye. • S3: It is warmer and not colder for a tropical cyclone. The warmer temperature is what drives the storm. • What is a cyclone? o Tropical Cyclone is any large system of winds that circulates about a center of low atmospheric pressure in a counter-clockwise direction north of the Equator and in a clockwise direction to the south. • Cyclone formation: o Cyclone is the formation of very low-pressure system with very high-speed winds revolving around it. o Factors like wind speed, wind direction, temperature and humidity contribute to the development of cyclones. o Before cloud formation, water takes up heat from the atmosphere to change into vapour. When water vapour changes back to liquid form as raindrops, this heat is released to the atmosphere. o The heat released to the atmosphere warms the air around. The air tends to rise and causes a drop in pressure. More air rushes to the centre of the storm. This cycle is repeated.

Telegram: https://t.me/insightsIAStips 127 Youtube: https://www.youtube.com/channel/UCpoccbCX9GEIwaiIe4HLjwA

Revision Through MCQs (RTM) Compilation (March 2021)

Refer: https://www.insightsonindia.com/2021/03/30/odisha-suffered-losses-worth- %e2%82%b931945-cr-in-eight-cyclones/ 245. Consider the following statements regarding autonomous administrative councils (ADCs) in India: 1. The Fifth Schedule of the Constitution of India allows for the formation of autonomous administrative councils. 2. They have powers to form courts to hear cases where both parties are members of Scheduled Tribes. 3. 125th constitutional amendment bill seeks to increase the financial and executive powers of the autonomous administrative councils. Which of the given above statements is/are correct? (a) 1 and 2 only (b) 2 and 3 only (c) 3 only (d) 1, 2 and 3 Ans: (b) Explanation: • S1: As per the Sixth Schedule, the four states viz. Assam, Meghalaya, Tripura and Mizoram contain the Tribal Areas which are technically different from the Scheduled Areas. • S2: Autonomous district councils have powers to form courts to hear cases where both parties are members of Scheduled Tribes and the maximum sentence is less than 5 years in prison. • S3: Related- 125th amendment bill: o It seeks to increase the financial and executive powers of the 10 Autonomous Councils in the Sixth Schedule areas of the northeastern region. o The amendments provide for elected village municipal councils,ensuring democracy at the grassroot level. o Powers: The village councils will be empowered to prepare plans for economic development and social justice including those related to agriculture, land improvement, implementation of land reforms, minor irrigation, water management, animal husbandry, rural electrification, small scale industries and social forestry. o The Finance Commissionwill be mandated to recommend devolution of financial resources to them. o Finance: The Autonomous Councils now depend on grants from Central ministries and the State government for specific projects. o Reservations: At least one-third of the seats will be reserved for women in the village and municipal councils in the Sixth Schedule areas of Assam, Mizoram and Tripura after the amendment is approved. Refer: https://www.insightsonindia.com/2021/03/30/what-is-the-greater-tipraland-demand/ 246. Consider the following statements regarding the Lord Mountbatten- the last Viceroy of India. 1. He prepared a “Dickie Bird Plan” for India’s independence. 2. He proposed the “Heads of Agreement Deal” which gave Hyderabad the status of an autonomous dominion under India. 3. He supported the congress stand that the princely states must not be given the option of independence. Which of the given above statements is/are correct? (a) 3 only (b) 2 and 3 only (c) 1 and 3 only (d) 1, 2 and 3

Telegram: https://t.me/insightsIAStips 128 Youtube: https://www.youtube.com/channel/UCpoccbCX9GEIwaiIe4HLjwA

Revision Through MCQs (RTM) Compilation (March 2021)

Ans: (d) Explanation: • S1: Lord Mountbatten prepared a “Dickie Bird Plan” for India’s independence. This plan was prepared by a committee of General Sir Hastings Ismay, Sir George Abell and Lord Mountbatten himself. Plan Balkan was completed and presented on 15-16 April 1947 by Hastings Ismay to assembly of provincial governors in Delhi. Due to this, this plan was also called “Ismay Plan”. • S2: It was in June 1948 that Lord Mountbatten proposed the Heads of Agreement deal which gave Hyderabad the status of an autonomous dominion nation under India. India was ready to sign the deal and did so but the Nizam refused on the grounds that he wanted complete independence or the status of dominion under the British Commonwealth of Nations. More>> • S3: In the Mountbatten Plan it is stated that the Princely States would have the option to join either of the two dominions of India or Pakistan. • Patricia Mountbatten: o Patricia Edwina Victoria Mountbatten was the Second Countess Mountbatten of Burma and had a strong royal connection as the great-great-granddaughter of Queen Victoria and first cousin to Prince Philip, the 99-year-old husband of Britain’s Queen Elizabeth II. o She was the eldest daughter of Lord Louis Mountbatten — the last Viceroy of India. Refer: Facts for Prelims: https://www.insightsonindia.com/2021/03/30/insights-daily-current- affairs-pib-summary-30-march-2021/ 247. The object of the Butler Committee of 1927 was to? (a) Define the jurisdiction of the Central and Provincial Governments. (b) Define the powers of the Secretary of State for India. (c) Impose censorship on national press. (d) Improve the relationship between the Government of India and the Indian States. Ans: (d) Explanation: • It was also known as the Indian states committee. • Relations between the Indian Princes and the crown were not well defined because the extent of sovereignty of the Paramount power was not settled properly. • The Indian states committee (under the Chairmanship of Harcourt Butler) in 1927 was appointed to investigate and clarify the relationship between the paramount power and the Princes. Refer: http://tinyurl.com/yatpetz2 248. With reference to ‘Swamitva Yojana’, consider the following statements: 1. It is a Central Sector scheme. 2. The Department of Land Resources is the Nodal Department for implementation of the scheme. 3. It aims to provide an integrated property validation solution for rural India. Which of the given above statements is/are correct? (a) 1 and 2 only (b) 3 only (c) 1 and 3 only (d) 1, 2 and 3 Ans: (c) Explanation: • SVAMITVA Scheme is a Central Sector scheme launched by Hon’ble Prime Minister of India on National Panchayat Day i.e 24th April 2020. The Ministry of Panchayati Raj (MoPR) is the Nodal Ministry for implementation of the scheme. In the States, the Revenue Department / Land Records Department will be the Nodal Department and shall

Telegram: https://t.me/insightsIAStips 129 Youtube: https://www.youtube.com/channel/UCpoccbCX9GEIwaiIe4HLjwA

Revision Through MCQs (RTM) Compilation (March 2021)

carry out the scheme with support of State Panchayati Raj Department. Survey of India shall work as the technology partner for implementation. • The scheme aims to provide an integrated property validation solution for rural India. The demarcation of rural abadi areas would be done using Drone Surveying technology. This would provide the ‘record of rights’ to village household owners possessing houses in inhabited rural areas in villages which, in turn, would enable them to use their property as a financial asset for taking loans and other financial benefits from Bank • The scheme seeks to achieve the following objectives: - o To bring financial stability to the citizens in rural India by enabling them to use their property as a financial asset for taking loans and other financial benefits. o Creation of accurate land records for rural planning. o Determination of property tax, which would accrue to the GPs directly in States where it is devolved or else, add to the State exchequer. o Creation of survey infrastructure and GIS maps that can be leveraged by any department for their use. o To support in preparation of better-quality Gram Panchayat Development Plan (GPDP) by making use of GIS maps. o To reduce property related disputes and legal cases Refer: https://svamitva.nic.in/svamitva/

249. With reference to India’s Rajaji National Park, consider the following statements: 1. It is situated in the state of Himachal Pradesh. 2. It is most renowned for its elephants. 3. The Suswa river is the lifeline of Rajaji National Park. Which of the given above statements is/are correct? (a) 1 and 2 only (b) 2 and 3 only (c) 3 only (d) 1, 2 and 3 Ans: (b) Explanation: • S3: …. The two rivers after criss-crossing Dehradun city meet the Suswa river at Ramgarh at the Rajaji National Park. The Suswa finally merges with the Ganga. The Suswa, an important tributary of the Ganga, carries all dirt and garbage into the holy river….More>> • S2: The park is at the northwestern limit of distribution for both elephants and tigers in India. The park is most renowned for its elephants. The mountain goat, goral is another noteworthy resident. It is mainly confined to the precipitous pine-covered slopes. • S1: Rajaji National Park is situated along the hills and foothills of Shiwalik ranges in the Himalayan foothills and represent the Shiwalik eco-system. Combining three sanctuaries, namely Chilla, Motichur and Rajaji - Rajaji National Park is spread over the Pauri Garwal, Dehradun and Saharanpur districts of Uttarakhand. Refer: http://rajajinationalpark.co.in/1.about.htm 250. Which one of the following is a critically endangered species in India? (a) Malabar Civet (b) Asiatic lion (c) Bengal tiger (d) Nilgiri tahr Ans: (a) Explanation: • The Malabar Civet (Viverra civettina) is considered to be one of the world’s rarest mammals. It is endemic to India and was first reported from Travancore, Kerala. It is nocturnal in nature and found exclusively in the Western Ghats.

Telegram: https://t.me/insightsIAStips 130 Youtube: https://www.youtube.com/channel/UCpoccbCX9GEIwaiIe4HLjwA

Revision Through MCQs (RTM) Compilation (March 2021)

• Habitat: Wooded plains and hill slopes of evergreen rainforests. • Distribution: Western Ghats. • Threats: Deforestation and commercial plantations are major threats. • It is listed as Critically Endangered on the IUCN Red List as the population is estimated to number fewer than 250 mature individuals. Refer: http://www.kerenvis.nic.in/Database/CriticallyEndangeredAnimal_1094.aspx

Telegram: https://t.me/insightsIAStips 131 Youtube: https://www.youtube.com/channel/UCpoccbCX9GEIwaiIe4HLjwA

Revision Through MCQs (RTM) Compilation (March 2021)

RTM- REVISION THROUGH MCQS – 31th-Mar-2021

251. Consider following statements: 1. Under Indian law, only a “law” can be challenged as unconstitutional. 2. The word ‘law’ in Article 13 includes only ordinary laws and not the constitutional amendment acts (constituent laws). Which of the given above statements is/are correct? (a) 1 only (b) 2 only (c) Both 1 and 2 (d) Neither 1 nor 2 Ans: (c) Explanation: • The term ‘law’ in Article 13 has been given a wide connotation so as to include the following: (a) Permanent laws enacted by the Parliament or the state legislatures; (b) Temporary laws like ordinances issued by the president or the state governors; (c) Statutory instruments in the nature of delegated legislation (executive legislation) like order, bye-law, rule, regulation or notification; and (d) Non-legislative sources of law, that is, custom or usage having the force of law. Thus, not only a legislation but any of the above can be challenged in the courts as violating a Fundamental Right and hence, can be declared as void. • Further, Article 13 declares that a constitutional amendment is not a law and hence cannot be challenged. However, the Supreme Court held in the Kesavananda Bharati case (1973) that a Constitutional amendment can be challenged on the ground that it violates a fundamental right that forms a part of the ‘basic structure’ of the Constitution and hence, can be declared as void. Refer: https://www.insightsonindia.com/2021/03/31/freedom-of-religion/ 252. Consider the following statements: 1. A Uniform Civil Code is one that would provide for one law for the entire country, applicable to all religious communities in their personal matters. 2. Article 44 of the Constitution lays down that the state shall endeavour to secure a Uniform Civil Code for the citizens throughout the territory of India. 3. The duty of the state is greater in other directive principles than in Article 44. Which of the given above statements is/are correct? (a) 1 and 2 only (b) 2 and 3 only (c) 1 and 3 only (d) 1, 2 and 3 Ans: (d) Explanation: • A Uniform Civil Code is one that would provide for one law for the entire country, applicable to all religious communities in their personal matters such as marriage, divorce, inheritance, adoption etc. Article 44 of the Constitution lays down that the state shall endeavour to secure a Uniform Civil Code for the citizens throughout the territory of India. • Article 44 is one of the Directive Principles of State Policy. These, as defined in Article 37, are not justiciable (not enforceable by any court) but the principles laid down therein are fundamental in governance. • Fundamental Rights are enforceable in a court of law. While Article 44 uses the words “state shall endeavour”, other Articles in the ‘Directive Principles’ chapter use words such as “in particular strive”; “shall in particular direct its policy”; “shall be obligation of the state” etc.

Telegram: https://t.me/insightsIAStips 132 Youtube: https://www.youtube.com/channel/UCpoccbCX9GEIwaiIe4HLjwA

Revision Through MCQs (RTM) Compilation (March 2021)

• Article 43 mentions “state shall endeavour by suitable legislation”, while the phrase “by suitable legislation” is absent in Article 44. All this implies that the duty of the state is greater in other directive principles than in Article 44. Refer: https://www.insightsonindia.com/2021/03/31/uniform-civil-code-4/ 253. Consider the following statements: 1. Government of India launched the ‘Overseas Citizenship of India (OCI) Scheme’ by making amendments to Citizenship Act, 1955 in 2014. 2. As per the recent government notification, the Overseas Citizen of India (OCI) cardholders are required to carry their old passport with them to fly to India. 3. Individuals who do not have citizenship of any other country are not eligible to gain an OCI status. Which of the given above statements is/are correct? (a) 1 and 3 only (b) 2 and 3 only (c) 3 only (d) 1 and 2 only Ans: (c) Explanation: • S3: Anyone who is applying for OCI card should hold a valid Passport of another country. o Individuals who do not have citizenship of any other country are not eligible to gain an OCI status. o Individuals whose parents or grandparents hold citizenship of Pakistan and Bangladesh are not eligible to apply. • S2: People of Indian origin and the Indian diaspora having Overseas Citizens of India (OCI) cards will not have to carry their old, expired passports for travel to India, as was required earlier, according to a government notification. • S1: Government of India launched the ‘Overseas Citizenship of India (OCI) Scheme’ by making amendments to Citizenship Act, 1955 in 2005. o On 09 January 2015, the Government of India discontinued the PIO card and merged it with OCI card. Refer: https://www.insightsonindia.com/2021/03/31/overseas-citizens-of-india-oci-2/ 254. Consider the following statements with reference to Election Commission of India's Model Code of Conduct: 1. It came into force immediately on announcement of the election schedule by the commission. 2. It is applicable to a “caretaker” government on premature dissolution of a State Assembly. 3. It instructs that ministers and other government authorities cannot announce financial grants in any form. Which of the given above statements is/are correct? (a) 1 and 2 only (b) 2 and 3 only (c) 1 and 3 only (d) 1, 2 and 3 Ans: (d) Explanation: • S1 & S2: The code comes into force on the announcement of the poll schedule and remains operational till the process is concluded, as provided in the notification. It is also applicable to a “caretaker” government on premature dissolution of a State Assembly, as was the case in Telangana. • S3: According to the EC, the code states that the party in power — whether at the Centre or in the States — should ensure that it does not use its official position for campaigning. Ministers and other government authorities cannot announce financial grants in any form. No project or scheme which may have the effect of influencing the voter in

Telegram: https://t.me/insightsIAStips 133 Youtube: https://www.youtube.com/channel/UCpoccbCX9GEIwaiIe4HLjwA

Revision Through MCQs (RTM) Compilation (March 2021)

favour of the party in power can be announced, and Ministers cannot use official machinery for campaign purposes. Refer: https://www.insightsonindia.com/2021/03/31/model-code-of-conduct-2/ 255. With reference to New Development Bank (NDB), consider the following statements: 1. NDB has more than 50 member nation. 2. China is the largest shareholder in NDB. 3. The bank is headquartered in Shanghai, China. Which of the given above statements is/are correct? (a) 1 and 2 only (b) 3 only (c) 2 and 3 only (d) 1, 2 and 3 Ans: (b) Explanation: • S1 & S2: The Agreement on the New Development Bank entered into force in July 2015, with the official declaration of all five states that have signed it. The five founding members of the Bank include Brazil, Russia, India, China and South Africa. o Bank's Articles of Agreement specify that all members of the United Nations could be members of the bank, however the share of the BRICS nations can never be less than 55% of voting power.

• • S3: Headquartered in Shanghai, China. Refer: https://www.insightsonindia.com/2021/03/31/new-development-bank-ndb/ 256. “2020 Human Rights Report” released by which of the following? (a) Amnesty International (b) International Court of Justice (c) Human Rights Watch (d) None of the above Ans: (d) Explanation: • Released by the U.S. State Department. • The report, which is submitted each year to the U.S. Congress, is retrospective and contains a country-wise discussion of the state of human rights. Refer: https://www.insightsonindia.com/2021/03/31/2020-human-rights-report/ 257. Consider the following statements about the Sabarmati river: 1. It originates in the Pushkar valley of the Aravalli Range in Gujarat. 2. Ahmedabad city is located along the bank of this river. 3. This river crosses the Tropic of Cancer twice. Which of the given above statements is/are correct? (a) 1 and 3 only (b) 2 only

Telegram: https://t.me/insightsIAStips 134 Youtube: https://www.youtube.com/channel/UCpoccbCX9GEIwaiIe4HLjwA

Revision Through MCQs (RTM) Compilation (March 2021)

(c) 1 and 2 only (d) 1, 2 and 3 Ans: (b) Explanation: About the Sabarmati river: • It is one of the west flowing rivers along with Narmada and Tapti. • It Originates from Aravalli hills near Tepur village in Udaipur Dist of state Rajasthan. • Mouth of the river: Gulf of Cambey ( Khambhat). • Ahmedabad city is located along the bank of this river. Refer: facts for prelims: 258. Which of the following are in Agasthyamala Biosphere Reserve? (a) Neyyar, Peppara and Shendurney Wildlife Sanctuaries; and Kalakad Mundanthurai Tiger Reserve (b) Mudumalai, Sathyamangalam and Wayanad Wildlife Sanctuaries; and Silent Valley National Park (c) Kaundinya, Gundla Brahme-swaram and Papikonda Wildlife Sanctuaries; and Mukurthi National Park (d) Kawal and Sri Venkateswara Wildlife Sanctuaries; and Nagarjunasagar-Srisailam Tiger Reserve Ans: (a) Explanation: • The Agasthyamala Biosphere Reserve is a biosphere reserve in India established in 2001, located in the southernmost end of the Western Ghats • It encompass the wildlife santiaries Shendurney Wildlife Sanctuary, Peppara Wildlife Sanctuary, Neyyar Wildlife Sanctuary, and Kalakkad Mundanthurai Tiger Reserve. • Agasthyamalai Biosphere Reserve became part of World Network of Biosphere Reserves in 2016. Is also under UNESCO's world list of biosphere reserve. • Nacaduba sinhala ramaswamii Sadasivan: o It is a new butterfly species discovered in the Agasthyamalai in the Western Ghats. Refer: facts for prelims: https://www.insightsonindia.com/2021/03/31/insights-daily-current- affairs-pib-summary-31-march-2021/ 259. With reference to Biosphere reserves in India, consider the following statements: 1. Presently, there are 12 notified biosphere reserves in India. 2. Simlipal is the smallest biosphere reserve in India. 3. The Biosphere Reserves in India are declared by the State Government through nomination under the UNESCO’S Man & Biosphere (MAB) Programme. Which of the given above statements is/are correct? (a) 1 and 2 only (b) 3 only (c) 2 and 3 only (d) 1, 2 and 3 Ans: (b) Explanation: • S3: The Biosphere Reserves in India are declared by the State or Central Government through Nomination under the UNESCO’S Man & Biosphere (MAB) Programme. • S2: The Panna Biosphere Reserve covering a total area of about 542 km2 only is the smallest biosphere reserve in India. This reserve was established in the year 2011 and is a part of Panna and Chhatarpur District of Madhya Pradesh. • S1: Presently, there are 18 notified biosphere reserves in India. Refer: http://www.wiienvis.nic.in/Database/BiosphereReserves_8225.aspx

Telegram: https://t.me/insightsIAStips 135 Youtube: https://www.youtube.com/channel/UCpoccbCX9GEIwaiIe4HLjwA

Revision Through MCQs (RTM) Compilation (March 2021)

260. Consider the following pairs: Marine Protected Areas State 1. Coringa Andhra Pradesh 2. Haliday Island Goa 3. Sajnakhali West Bengal 4. Point Calimere Kerala Which of the given above pairs is/are correctly matched? (a) 1 and 3 only (b) 1, 2 and 3 only (c) 2, 3 and 4 only (d) 2 and 4 only Ans: (a) Explanation: • Point Calimere: Tamil Nadu • Haliday Island: West Bengal Refer: http://www.wiienvis.nic.in/Database/MPA_8098.aspx

Telegram: https://t.me/insightsIAStips 136 Youtube: https://www.youtube.com/channel/UCpoccbCX9GEIwaiIe4HLjwA